You are on page 1of 42

Title : G.R. No. L-2068 October 20, 1948 section 154 of Act No.

section 154 of Act No. 190, and as an auxiliary remedy, to have a writ of preliminary injunction issued so that the trial may be held pending until
further orders of this court.

DOMINADOR B. BUSTOS, petitioner, vs. ANTONIO G. LUCERO, Judge of First Instance of Pampanga, respondent. Petition was originally filed with CA
This petition was originally filed with the Court of Appeals, but was later certified to this court on the ground that the main basis of the petition is
Topic: Remedial Law section 49 of Republic Act No. 409, otherwise known as Revised Charter of the City of Manila, approved on June 18, 1949, and respondents
assail the constitutionality of said section in that it contravenes the constitutional provision that the rules of court "shall be uniform for all courts
The petitioner herein, an accused in a criminal case, filed a motion with the Court of First Instance of Pampanga after he had been bound over of the same grade . . . . (Section 13, Article VIII of the Constitution.).
to that court for trial, praying that the record of the case be remanded to the justice of the peace court of Masantol, the court of origin, in order
that the petitioner might cross-examine the complainant and her witnesses in connection with their testimony, on the strength of which warrant RTC:
was issued for the arrest of the accused.
Petitioner was charged before the Court of First Instance of Manila with two statutory offenses, namely, (1) with a violation of Commonwealth
RTC: Act No. 606, which was docketed as criminal case No. 18374, in that he knowingly chartered a vessel of Philippine registry to an alien without
The motion was denied the approval of the President of the Philippines and (2) with a violation of section 129 in relation to section 2713 of the Revised Administrative
Code, which was docketed as Criminal Case No. 18375, in that he failed to submit to the Collector of Customs the manifests and certain
SC authenticated documents for the vessel "Antarctic" and failed to obtain the necessary clearance from the Bureau of Customs prior to the
Denial is the subject matter of this proceeding. departure of said vessel for a foreign port.

Petioner’s Position: Before Trial of RTC petitioner filed motion:


On April 23, 1952, before the trial of said criminal cases, petitioner filed a motion praying that assessors be appointed to assist the court in
According to the memorandum submitted by the petitioner's attorney to the Court of First Instance in support of his motion, the accused, considering the questions of fact involved in said cases as authorized by section 49 of Republic Act No. 409, otherwise known as Revised
assisted by counsel, appeared at the preliminary investigation. In that investigation, the justice of the peace informed him of the charges and Charter of the City of Manila, which provides that "the aid of assessors in the trial of any civil or criminal action in the Municipal Court, or the
asked him if he pleaded guilty or not guilty, upon which he entered the plea of not guilty. "Then his counsel moved that the complainant present Court of First Instance, within the City, may be invoked in the manner provided in the Code of Civil Procedure." This motion was opposed by the
her evidence so that she and her witnesses could be examined and cross-examined in the manner and form provided by law." The fiscal and City Fiscal who appeared for the People of the Philippines.
the private prosecutor objected, invoking section 11 of rule 108, and the objection was sustained. "In view thereof, the accused's counsel
announced his intention to renounce his right to present evidence," and the justice of the peace forwarded the case to the court of first instance. RTC denied Motion :

Issue: On April 28, 1952, the court issued an order denying the motion holding in effect that with the promulgation of the Rules of Court by the
Supreme Court, which became effective on July 1, 1940, all rules concerning pleading, practice and procedure in all courts of the Philippines
Whether the accused, after renouncing his right to present evidence, and by reason of that waiver he was committed to the corresponding court previously existing were not only superseded but expressly repealed, that the Supreme Court, having been vested with the rule-making power,
for trial, is estopped. expressly omitted the portions of the Code of Civil Procedure regarding assessors in said Rules of Court, and that the reference to said statute
by section 49 of Republic Act No. 409 on the provisions regarding assessors should be deemed as a mere surplusage. Believing that this order
Ruling: is erroneous, petitioner now comes to this court imputing abuse of discretion to the respondent Judge.

No.

In Dequito and Saling Buhay vs. Arellano, G.R. No. L- 1336, r, in which case the respondent justice of the peace had allowed the accused, over Issues raised by the petioner:
the complaint's objection, to recall the complainant and her witnesses at the preliminary investigation so that they might be cross-examined, SC I. The right of the petitioner to a trial with the aid of assessors is an absolute substantive right, and the duty of the court to provide assessors is
sustained the justice of the peace's order. SC said that section 11 of Rule 108 does not curtail the sound discretion of the justice of the peace mandatory.
on the matter.SC said that "while section 11 of Rule 108 defines the bounds of the defendant's right in the preliminary investigation, there is II. The right to trial with the aid of assessors, being a substantive right, cannot be impaired by this court in the exercise of its rule-making power.
nothing in it or any other law restricting the authority, inherent in a court of justice, to pursue a course of action reasonably calculated to bring III. Section 154 of the Code of Civil Procedure and Section 2477 of the Old Charter of Manila, creating the right to trial with the aid of
out the truth." assessors, are substantive law and were not repealed by Rules of Court.
The "defendant can not, as a matter of right, compel the complaint and his witnesses to repeat in his presence what they had said at the IV. Granting without admitting that the provisions on assessors of the Code of Civil Procedure and the old Charter of Manila were impliedly
preliminary examination before the issuance of the order of arrest.The constitutional right of an accused to be confronted by the witnesses repealed, nevertheless, the same provisions were later reenacted by reference in section 49 of the Revised Charter of Manila, which is now the
against him does not apply to preliminary hearings' nor will the absence of a preliminary examination be an infringement of his right to confront source of the right to trial with the aid of assessors and which refers to the Code of Civil Procedure merely to indicate the procedure for
witnesses." As a matter of fact, preliminary investigation may be done away with entirely without infringing theconstitutional right of an accused appointing assessors.
under the due process clause to a fair trial. V. Section 49 of the Revised Charter of Manila is not invalid class legislation and does not violate the constitutional provision that the rules of
pleading, practice and procedure 'shall be uniform for all the courts of the same grade.
Issue raised by respondent:
In another resolution: GR No. L-2068, Mar 08, 1949 DOMINADOR B. BUSTOS v. ANTONIO G. LUCERO Respondents now contend that the right to trial with the aid of assessors, with all its concomitant provisions, cannot now be invoked because,
being procedural in nature, the same must be deemed to have been impliedly eliminated.
Issue:
It is contended that Section 11 of Rule 108 of the Rules of Court infringes Section 13, Article VIII, of the Constitution. It is said that the rule in Ruling: Petition Granted
question deals with substantive matters and impairs substantive rights.
This respondent’s claim would be correct if we were to hold that the right to trial with the aid of assessors is not substantive but procedural or
Ruling: adjective in nature. If it were merely procedural, not having been incorporated in the Rules of Court, the logical conclusion is that the rule-
making power has deemed wise to eliminate it.
SC disagrees . The court is of the opinion that Section 11 of Rule 108, like its predecessors, is an adjective law and not a substantive law or
substantive right. Substantive law creates substantive rights and the two terms in this respect may be said to be synonymous. Substantive
rights is a term which includes those rights which one enjoys under the legal system prior to the disturbance of normal relations. (60 C. J. 980.) The trial with the aid of assessors as granted by section 154 of the Code of Civil Procedure and section 2477 of the old Charter of Manila are
Substantive law is that part of the law which creates, defines and regulates rights, or which regulates the rights and duties which give rise to a parts of substantive law and as such are not embraced by the rule-making power of the Supreme Court. This is so because in said section 154
cause of action; that part of the law which courts are established to administer; as opposed to adjective or remedial law, which prescribes the this matter is referred to as a right given by law to a party litigant. Section 2477 of the Administrative Code of 1917 is couched is such a manner
method of enforcing rights or obtains redress for their invasion. (36 C. J. 27; 52 C. J. S. 1026.) that a similar right is implied when invoked by a party litigant. It says that the aid may be invoked in the manner provided in the Code of Civil
Procedure. And this right has been declared absolute and substantial by this Court in several cases where the aid of assessors had been
invoked (Berbari vs. Concepcion, et al., 40 Phil., 320; Colegio de San Jose vs. Sison, 54 Phil., 344.) Thus, it was there said that these
provisions "necessarily lead to the conclusion that the intervention of the assessors is not an empty formality which may be disregarded without
violating either the letter or the spirit of the law. It is another security given by the law to the litigants, and as such, it is a substantial right of
Title : G.R. No. L-6120 which they cannot be deprived without vitiating all the proceedings. Were we to agree that for one reason or another the trial by assessors may
be done away with, the same line of reasoning would force us to admit that the parties litigant may be deprived of their right to be represented
by counsel, to appear and be present at the hearings, and so on, to the extent of omitting the trial in a civil case, and thus set at naught the
June 30, 1953 essential rights granted by the law to the parties, with consequent nullity of the proceedings." (Colegio de San Jose vs. Sison, 54 Phil., 344,
349.)

CIPRIANO P. PRIMICIAS, petitioner,


vs.
FELICISIMO OCAMPO, as Judge-at-large presiding over Branch C of the Court of First Instance of Manila and EUGENIO ANGELES,
as City Fiscal of Manila, representing the PEOPLE OF THE PHILIPPINES, respondents.
Title: G.R. No. 141524 September 14, 2005

Topic: Remedial Law

This is a petition which seeks to prohibit respondent Judge from proceeding with the trial of two criminal cases which were then pending
against petitioner without the assistance of assessors in accordance with the provisions of section 49 of Republic Act No. 409 in relation to
DOMINGO NEYPES, LUZ FAUSTINO, ROGELIO FAUSTINO, LOLITO VICTORIANO, JACOB OBANIA AND DOMINGO CABACUNGAN, THE HONORABLE COURT OF APPEALS FINALLY ERRED IN FINDING THAT THE DECISION IN THE CASE OF DENSO, INC. V. IAC, 148
Petitioners, SCRA 280, IS APPLICABLE IN THE INSTANT CASE THEREBY IGNORING THE PECULIAR FACTS AND CIRCUMSTANCES OF THIS CASE
AND THE FACT THAT THE SAID DECISION WAS
RENDERED PRIOR TO THE ENACTMENT OF THE 1997 RULES OF CIVIL PROCEDURE.

vs.
HON. COURT OF APPEALS, HEIRS OF BERNARDO DEL MUNDO, namely: FE, CORAZON, JOSEFA, SALVADOR and CARMEN, Note:
all surnamed DEL MUNDO, LAND BANK OF THE PHILIPPINES AND HON. ANTONIO N. ROSALES, Presiding Judge, Branch 43, The foregoing issues essentially revolve around the period within which petitioners should have filed their notice of appeal.
Regional Trial Court, Roxas, Oriental Mindoro, Respondent.
Ruling:

Topic: Remedial Law Supreme Court Granted the petition .

Petitioners Domingo Neypes, Luz Faustino, Rogelio Faustino, Lolito Victoriano, Jacob Obania and Domingo Cabacungan filed an action for Based on the foregoing, an appeal should be taken within 15 days from the notice of judgment or final order appealed from. A final judgment or
annulment of judgment and titles of land and/or reconveyance and/or reversion with preliminary injunction before the Regional Trial Court, order is one that finally disposes of a case, leaving nothing more for the court to do with respect to it. It is an adjudication on the merits which,
Branch 43, of Roxas, Oriental Mindoro, against the Bureau of Forest Development, Bureau of Lands, Land Bank of the Philippines and the considering the evidence presented at the trial, declares categorically what the rights and obligations of the parties are; or it may be an order or
heirs of Bernardo del Mundo, namely, Fe, Corazon, Josefa, Salvador and Carmen. judgment that dismisses an action.

As already mentioned, petitioners argue that the order of July 1, 1998 denying their motion for reconsideration should be construed as the "final
RTC: order," not the February 12, 1998 order which dismissed their complaint. Since they received their copy of the denial of their motion for
In the course of the proceedings, the parties (both petitioners and respondents) filed various motions with the trial court. Among these were: (1) reconsideration only on July 22, 1998, the 15-day reglementary period to appeal had not yet lapsed when they filed their notice of appeal on
the motion filed by petitioners to declare the respondent heirs, the Bureau of Lands and the Bureau of Forest Development in default and (2) July 27, 1998.
the motions to dismiss filed by the respondent heirs and the Land Bank of the Philippines, respectively.
What therefore should be deemed as the "final order," receipt of which triggers the start of the 15-day reglementary period to appeal 3 ⁄4 the
February 12, 1998 order dismissing the complaint or the July 1, 1998 order dismissing the MR?
In an order dated May 16, 1997, the trial court, presided by public respondent Judge Antonio N. Rosales, resolved the foregoing motions as
follows: (1) the petitioners’ motion to declare respondents Bureau of Lands and Bureau of Forest Development in default was granted for their In the recent case of Quelnan v. VHF Philippines, Inc., the trial court declared petitioner Quelnan non-suited and accordingly dismissed his
failure to file an answer, but denied as against the respondent heirs of del Mundo because the substituted service of summons on them was complaint. Upon receipt of the order of dismissal, he filed an omnibus motion to set it aside. When the omnibus motion was filed, 12 days of the
improper; (2) the Land Bank’s motion to dismiss for lack of cause of action was denied because there were hypothetical admissions and 15-day period to appeal the order had lapsed. He later on received another order, this time dismissing his omnibus motion. He then filed his
matters that could be determined only after trial, and (3) the motion to dismiss filed by respondent heirs of del Mundo, based on notice of appeal. But this was likewise dismissed ― for having been filed out of time.
prescription, was also denied because there were factual matters that could be determined only after trial.
The court a quo ruled that petitioner should have appealed within 15 days after the dismissal of his complaint since this was the final order that
MR filed: was appealable under the Rules. We reversed the trial court and declared that it was the denial of the motion for reconsideration of an order of
dismissal of a complaint which constituted the final order as it was what ended the issues raised there.
The respondent heirs filed a motion for reconsideration of the order denying their motion to dismiss on the ground that the trial court could very
well resolve the issue of prescription from the bare allegations of the complaint itself without waiting for the trial proper. In de la Rosa v. Court of Appeals, we stated that, as a rule, periods which require litigants to do certain acts must be followed unless, under
exceptional circumstances, a delay in the filing of an appeal may be excused on grounds of substantial justice. There, we condoned the delay
RTC decision: incurred by the appealing party due to strong considerations of fairness and justice.
In an order dated February 12, 1998, the trial court dismissed petitioners ’ complaint on the ground that the action had already prescribed.
Petitioners allegedly received a copy of the order of dismissal on March 3, 1998 and, on the 15th day thereafter or on March 18, 1998, filed a In setting aside technical infirmities and thereby giving due course to tardy appeals, we have not been oblivious to or unmindful of the
motion for reconsideration. On July 1, 1998, the trial court issued another order dismissing the motion for reconsideration which petitioners extraordinary situations that merit liberal application of the Rules. In those situations where technicalities were dispensed with, our decisions
received on July 22, 1998. Five days later, on July 27, 1998, petitioners filed a notice of appeal and paid the appeal fees on August 3, 1998. were not meant to undermine the force and effectivity of the periods set by law. But we hasten to add that in those rare cases where procedural
rules were not stringently applied, there always existed a clear need to prevent the commission of a grave injustice. Our judicial system and the
Notice of appeal denied : courts have always tried to maintain a healthy balance between the strict enforcement of procedural laws and the guarantee that every litigant
On August 4, 1998, the court a quo denied the notice of appeal, holding that it was filed eight days late. This was received by petitioners on be given the full opportunity for the just and proper disposition of his cause.
July 31, 1998. Petitioners filed a motion for reconsideration but this too was denied in an order dated September 3, 1998.

CA : The Supreme Court may promulgate procedural rules in all courts.It has the sole prerogative to amend, repeal or even establish new rules for a
more simplified and inexpensive process, and the speedy disposition of cases. In the rules governing appeals to it and to the Court of Appeals,
Petitioners position: particularly Rules 42, 43 and 45, the Court allows extensions of time, based on justifiable and compelling reasons, for parties to file their
In the appellate court, petitioners claimed that they had seasonably filed their notice of appeal. They argued that the 15-day reglementary appeals. These extensions may consist of 15 days or more.
period to appeal started to run only on July 22, 1998 since this was the day they received the final order of the trial court denying their motion To standardize the appeal periods provided in the Rules and to afford litigants fair opportunity to appeal their cases, the Court deems it practical
for reconsideration. When they filed their notice of appeal on July 27, 1998, only five days had elapsed and they were well within the to allow a fresh period of 15 days within which to file the notice of appeal in the Regional Trial Court, counted from receipt of the order
reglementary period for appeal. dismissing a motion for a new trial or motion for reconsideration. Henceforth, this "fresh period rule" shall also apply to Rule 40 governing
appeals from the Municipal Trial Courts to the Regional Trial Courts; Rule 42 on petitions for review from the Regional Trial Courts to the Court
Ca Decision: of Appeals; Rule 43 on appeals from quasi-judicial agencies to the Court of Appeals and Rule 45 governing appeals by certiorari to the
Supreme Court. The new rule aims to regiment or make the appeal period uniform, to be counted from receipt of the order denying the motion
On September 16, 1999, the Court of Appeals (CA) dismissed the petition. It ruled that the 15-day period to appeal should have been reckoned for new trial, motion for reconsideration (whether full or partial) or any final order or resolution.
from March 3, 1998 or the day they received the February 12, 1998 order dismissing their complaint. According to the appellate court, the order
was the "final order" appealable under the Rules. SC holds that petitioners seasonably filed their notice of appeal within the fresh period of 15 days, counted from July 22, 1998 (the date of
receipt of notice denying their motion for reconsideration). This pronouncement is not inconsistent with Rule 41, Section 3 of the Rules which
MR filed. states that the appeal shall be taken within 15 days from notice of judgment or final order appealed from. The use of the disjunctive word "or"
Petitioners filed a motion for reconsideration of the aforementioned decision. This was denied by the Court of Appeals on January 6, 2000. signifies disassociation and independence of one thing from another. It should, as a rule, be construed in the sense in which it ordinarily
implies. Hence, the use of "or" in the above provision supposes that the notice of appeal may be filed within 15 days from the notice of
judgment or within 15 days from notice of the "final order," which we already determined to refer to the July 1, 1998 order denying the motion
Issue: for a new trial or reconsideration.

In this present petition for review under Rule 45 of the Rules, petitioners ascribe the following errors allegedly committed by the appellate court:
I
THE HONORABLE COURT OF APPEALS ERRED IN DISMISSING THE PETITIONERS’ PETITION FOR
CERTIORARI AND MANDAMUS AND IN AFFIRMING THE ORDER OF THE HON. JUDGE ANTONIO N. ROSALES WHICH DISMISSED THE Title :G.R. No. L-286 March 29, 1946
PETITIONERS’ APPEAL IN CIVIL CASE NO. C-36 OF THE REGIONAL TRIAL COURT, BRANCH 43, ROXAS, ORIENTAL MINDORO, EVEN
AFTER THE PETITIONERS HAD PAID THE APPEAL DOCKET FEES.
II
THE HONORABLE COURT OF APPEALS LIKEWISE ERRED IN RULING AND AFFIRMING THE DECISION OR ORDER OF THE FREDESVINDO S. ALVERO, petitioner,
RESPONDENT HON. ANTONIO M. ROSALES THAT PETITIONERS’ APPEAL WAS FILED OUT OF TIME WHEN PETITIONERS RECEIVED vs.
THE LAST OR FINAL ORDER OF THE COURT ON JULY 22, 1998 AND FILED THEIR NOTICE OF APPEAL ON JULY 27, 1998 AND PAID M.L. DE LA ROSA, Judge of First Instance of Manila, JOSE R. VICTORIANO, and MARGARITA VILLARICA, respondents.
THE APPEAL DOCKET FEE ON AUGUST 3, 1998.
III
THE HONORABLE COURT OF APPEALS FURTHER ERRED IN RULING THAT THE WORDS "FINAL ORDER" IN SECTION 3, RULE 41, OF Topic: Remedial Law
THE 1997 RULES OF CIVIL PROCEDURE WILL REFER TO THE [FIRST] ORDER OF RESPONDENT JUDGE HON. ANTONIO M.
MORALES DATED FEBRUARY 12, 1998 INSTEAD OF THE LAST AND FINAL ORDER DATED JULY 1, 1998 COPY OF WHICH WAS Facts:
RECEIVED BY PETITIONERS THROUGH COUNSEL ON JULY 22, 1998.
IV. This is an original petition for certiorari filed in this court.

RTC:
Agreement:
Respondent:
The record shows that, on June 25, 1945, respondent Jose R. Victoriano had filed a complaint, in the Court of First Instance of the City of 1. Javellana would pay ₱80,000.00 upon the execution of the deed and the balance of ₱80,000.00 upon the registration of the
Manila, against petitioner Fredesvindo S. Alvero and one Margarita Villarica, alleging two causes of action, to wit, (1) to declare in force the parcels of land under the Torrens System by Margarita;
contract of sale, made on October 1, 1940, between said Jose R. Victoriano and Margarita Villarica, of two (2) parcels of land in the Manotoc 2. Should Margarita become incapacitated, her son and attorney-in-fact, Juvenal M. Alma Jose (Juvenal), and her daughter,
subdivision, Balintawak, in the barrio of Calaanan, municipality of Caloocan, Province of Rizal, with a combined area of 480 square meters, petitioner Priscilla M. Alma Jose, would receive the payment of the balance and proceed with the application for registration.
which land was subsequently sold by said Villarica, in favor of petitioner Fredesvindo S. Alvero, on December 31, 1944, for the sum of
P100,000 in Japanese military notes; and (2) to declare said subsequent sale null and void.
After Margarita died and with Juvenal having predeceased Margarita without issue, the vendor’s undertaking fell on the shoulders of Priscilla,
On July 7, 1945, Margarita Villarica filed an answer to said complaint, expressly admitting having sold said land to Fresdesvindo S. Alvero, for being Margarita’s sole surviving heir.
P100,000, in December, 1944, due to the imperative necessity of raising funds with which to provide for herself and family, and that she did not
remember the previous sale; at the same time, offering to repurchase said land from Fredesvindo S. Alvero in the sum of P5,000, but that the Cause of the dispute:
latter refused to accept the offer. Priscilla did not comply with the undertaking to cause the registration of the properties under the Torrens System, and, instead, began to
Respondent Counterclaim: improve the properties by dumping filling materials therein with the intention of converting the parcels of land into a residential or industrial
On July 13, 1945, Fredesvindo S. Alvero, in answering said complaint, denied the allegations made therein, and claimed exclusive ownership subdivision.
of the land in question, and at the same time set up a counterclaim and crossclaim in his answer, demanding from Jose R. Victoriano a P200-
monthly rent on said property, beginning from February, 1945, plus P2,000 as damages. Case originally filed:
Javellana commenced on February 10, 1997 an action for specific performance, injunction, and damages against Priscilla in the RTC in
RTC Decision: Malolos, Bulacan; entitled Ramon C. Javellana, represented by Atty. Guillermo G. Blanco v. Priscilla Alma Jose.

Considering that Jose R. Victoriano's document was older than that of Fredesvindo S. Alvero, and that he had taken possession of said Plaintiff’s averments:
property, since October 1, 1940
The respondent judge rendered his decision in favor of Jose R. Victoriano, adjudging to him the title over the property in question, including all 1. Upon the execution of the deed of conditional sale, he had paid the initial amount of ₱80,000.00 and had taken possession of
the improvements existing thereon, and dismissed the counterclaim. the parcels of land;
2. That he had paid the balance of the purchase price to Juvenal on different dates upon Juvenal’s representation that Margarita
Subsequent Events: had needed funds for the expenses of registration and payment of real estate tax; and
On November 28, 1945, Fredesvindo S. Alvero was notified of said decision; and on December 27, 1945, he filed a petition for reconsideration 3. That in 1996, Priscilla had called to inquire about the mortgage constituted on the parcels of land; and
and new trial, which was denied on January 3, 1946; and of said order he was notified on January 7, 1946. 4. That he had told her then that the parcels of land had not been mortgaged but had been sold to him.
On January 8, 1946, Fredesvindo S. Alvero filed his notice of appeal and record on appeal simultaneously in the lower court, without filing the
P60-appeal bond.
On January 14, 1946, Jose R. Victoriano filed a petition to dismiss the appeal, and at the same time, asked for the execution of the judgment. Plaintiff’s prayer:
On January 15, 1946, Fredesvindo S. Alvero filed an opposition to said motion to dismiss, alleging that on the very same day, January 15,
1946, said appeal bond for P60 had been actually filed, and allege as an excuse, for not filing the said appeal bond, in due time, the illness of 1. Issuance of a TRO or writ of preliminary injunction to restrain Priscilla from dumping filling materials in the parcels of land; and
his lawyer's wife, who died on January 10, 1946, and buried the following day. 2. That Priscilla be ordered to institute registration proceedings and then to execute a final deed of sale in his favor.
On January 17, 1946, the respondent judge, Hon. Mariano L. de la Rosa, ordered the dismissal of the appeal, declaring that, although the
notice of appeal and record on appeal had been filed in due time, the P60-appeal bond was filed too late.
On January 23, 1946, Fredesvindo S. Alvero filed a petition for the reconsideration of the said order dated January 17, 1946, dismissing his Defendant’s answer:
appeal; and said petition for reconsideration was denied on January 29, 1946. Priscilla filed a motion to dismiss, stating that the complaint was already barred by prescription; and that the complaint did not state a cause of
action.

SC: RTC:

Petioner filed petition for certiorari. 1. The RTC initially denied Priscilla’s motion to dismiss.
2. Upon her MR, the RTC on June 24, 1999 reversed itself and granted the motion to dismiss.
Respondent: Basis/Reason: Javellana had no cause of action against her due to her not being bound to comply with the
On February 11, 1946, the respondents filed their answer to the petition for certiorari, alleging (1) that said petition is defective in form as well terms of the deed of conditional sale for not being a party thereto; that there was no evidence showing the payment of the
as in substance; (2) that there has been no excusable negligence, on the part of the petitioner, or grave abuse of discretion on the part of the balance; that he had never demanded the registration of the land from Margarita or Juvenal, or brought a suit for specific
respondent judge, in the instant case. performance against Margarita or Juvenal; and that his claim of paying the balance was not credible.
3. Javellana moved for reconsideration -- contentions:
a. that the presentation of evidence of full payment was not necessary at that stage of the proceedings
Issue: Whether petioner filed the appeal on time (Nonetheless, he attached to the motion for reconsideration the receipts showing the payments made to
Juvenal.);
Ruling: b. that in resolving a motion to dismiss on the ground of failure to state a cause of action, the facts alleged in the
complaint were hypothetically admitted and only the allegations in the complaint should be considered in
No. As already stated, the decision rendered by the respondent judge, Hon. Mariano L. de la Rosa, was dated November 16, 1945, of which resolving the motion; and
counsel for Fredesvindo S. Alvero was notified on November 28, 1945; that his motion for reconsideration and new trial was filed on December c. Priscilla could no longer succeed to any rights respecting the parcels of land because he had meanwhile
27, 1945, and denied on January 3, 1946, and that said counsel for Alvero was notified of said order on January 7, 1946; and that he filed his acquired absolute ownership of them; and that the only thing that she, as sole heir, had inherited from
notice of appeal and record on appeal the following day, to wit, January 8, 1946, and that the P60-appeal bond was filed only on January 15, Margarita was the obligation to register them under the Torrens System.
1946. 4. On June 21, 2000, the RTC denied the motion for reconsideration.
According to the computation erroneously made by the court, the last day for filing and perfecting the appeal, in this case, was January 8, 1946, Basis/reason: Lack of any reason to disturb the order of June 24, 1999.
or which date, Fredesvindo S. Alvero should have filed his (1) notice of appeal, (2) record on appeal, and (3) appeal bond. But the P60-appeal 5. Javellana filed a notice of appeal.
bond was filed only on January 15, 1946.
Failure to perfect the appeal, within the time prescribed by the rules of court, will cause the judgment to become final, and the certification of
the record on appeal thereafter, cannot restore the jurisdiction which has been lost. (Roman Catholic Bishop of Tuguegarao vs. Director of CA:
Lands, 34 Phil., 623; Estate of Cordoba and Zarate vs. Alabado, 34 Phil., 920; and Bermudez vs. Director of Lands, 36 Phil., 774.)
1. Priscilla countered that the June 21, 2000 order was not appealable; that the appeal was not perfected on time (Priscilla insists
Rules of courts, promulgated by authority of law, have the force and effect of law; and rules of court prescribing the time within which certain that Javellana filed his notice of appeal out of time. She points out that he received a copy of the June 24, 1999 order on July
acts must be done, or certain proceedings taken, are considered absolutely indispensable to the prevention of needless delays and to the 9, 1999, and filed his motion for reconsideration on July 21, 1999 (or after the lapse of 12 days); that the RTC denied his
orderly and speedy discharge of judicial business. (Shioji vs. Harvey, 43 Phil., 333.) motion for reconsideration through the order of June 21, 2000, a copy of which he received on July 13, 2000; that he had only
Strict compliance with the rules of court has been held mandatory and imperative, so that failure to pay the docket fee in the Supreme Court, three days from July 13, 2000, or until July 16, 2000, within which to perfect an appeal; and that having filed his notice of
within the period fixed for that purpose, will cause the dismissal of the appeal. (Salaveria vs. Albindo, 39Phil., 922.) In the same manner, on appeal on July 19, 2000, his appeal should have been dismissed for being tardy by three days beyond the expiration of the
failure of the appellant in a civil case to serve his brief, within the time prescribed by said rules, on motion of the appellee and notice to the reglementary period).; and that Javellana was guilty of forum shopping. (It appears that pending the appeal, Javellana also
appellant, or on its own motion, the court may dismiss the appeal. (Shioji vs. Harvey, 43 Phil., 333.) filed a petition for certiorari in the CA to assail the June 24, 1999 and June 21, 2000 orders dismissing his complaint.)
2. On August 6, 2001, however, the CA dismissed the petition for certiorari.
Held: Petition for certiorari dismissed.
Basis/reason: RTC did not commit grave abuse of discretion in issuing the orders, and holding that it only
committed, at most, an error of judgment correctible by appeal in issuing the challenged orders.

Jose v. Javellana, G.R. No. 158239, January 25, 2012 3. CA remanded the case back to the RTC for further proceedings.
4. On May 9, 2003, the CA denied the motion for reconsideration stating that it decided to give due course to the appeal even if
TOPIC: Nature and purpose of procedural law filed out of time because Javellana had no intention to delay the proceedings, as in fact he did not even seek an extension of
time to file his appellant’s brief; that current jurisprudence afforded litigants the amplest opportunity to present their cases free
FACTS: from the constraints of technicalities, such that even if an appeal was filed out of time, the appellate court was given the
On September 8, 1979, Margarita Marquez Alma Jose (Margarita) sold for ₱160,000.00 to respondent Ramon Javellana by deed of conditional discretion to nonetheless allow the appeal for justifiable reasons.
sale two parcels of land located in Barangay Mallis, Guiguinto, Bulacan.
SC: On 11 August 1997, petitioner filed a Notice of Appeal without paying the necessary docket fees. Immediately thereafter, respondents
Priscilla then brought this appeal, averring that the CA thereby erred in not outrightly dismissing Javellana’s appeal because: (a) the June 21, filed a Motion to Dismiss Appeal on the ground of nonpayment of docket fees.
2000 RTC order was not appealable XXXXXXX
In its Opposition,7 petitioner alleged that its counsel was not yet familiar with the revisions of the Rules of Court that became effective only on 1
July 1997. Its representative was likewise not informed by the court personnel that docket fees needed to be paid upon the filing of the Notice
On his part, Javellana countered that the errors being assigned by Priscilla involved questions of fact not proper for the Court to review through of Appeal. Furthermore, it contended that the requirement for the payment of docket fees was not mandatory. It therefore asked the RTC for a
petition for review on certiorari; that the June 21, 2000 RTC order, being a final order, was appealable; that his appeal was perfected on time; liberal interpretation of the procedural rules on appeals.

On 29 September 1997, the RTC issued an Order 8 dismissing the appeal citing Sec. 4 of Rule 41 9 of the Revised Rules of Court.
ISSUE
WHETHER OR NOT: THE JUNE 21, 2000 RTC ORDER WAS NO APPEALABLE Petitioner thereafter moved for a reconsideration of the Order 10 alleging that the trial court lost jurisdiction over the case after the former had
filed the Notice of Appeal. Petitioner also alleged that the court erred in failing to relax procedural rules for the sake of substantial justice.

RULING On 25 November 1997, the RTC denied the Motion.


YES IT IS APPEALABLE AND THE Appeal was made on time pursuant to Neypes v. CA
Section 3 of Rule 41 of the Rules of Court provides: CA:
Section 3. Period of ordinary appeal. — The appeal shall be taken within fifteen (15) days from notice of the judgment or final order appealed On 28 January 1998, petitioner filed with the Court of Appeals (CA) a Petition for Certiorari and Mandamus under Rule 65 alleging that
from. Where a record on appeal is required, the appellant shall file a notice of appeal and a record on appeal within thirty (30) days from notice the RTC had no jurisdiction to dismiss the Notice of Appeal, and that the trial court had acted with grave abuse of discretion when it
of the judgment or final order. strictly applied procedural rules.

The period of appeal shall be interrupted by a timely motion for new trial or reconsideration. No motion for extension of time to file a motion for On 29 November 2000, the CA rendered its Decision 12 on the Petition. It held that while the failure of petitioner to pay the docket and
new trial or reconsideration shall be allowed. (n) other lawful fees within the reglementary period was a ground for the dismissal of the appeal pursuant to Sec. 1 of Rule 50 of the
Revised Rules of Court, the jurisdiction to do so belonged to the CA and not the trial court. Thus, appellate court ruled that the RTC
The seemingly correct insistence of Priscilla cannot be upheld, however, considering that the Court meanwhile adopted the fresh period rule in committed grave abuse of discretion in dismissing the appeal and set aside the latter’s assailed Order dated 29 September 1997.
Neypes v. Court of Appeals, by which an aggrieved party desirous of appealing an adverse judgment or final order is allowed a fresh period of
15 days within which to file the notice of appeal in the RTC reckoned from receipt of the order denying a motion for a new trial or motion for Thereafter, respondents filed their respective Motions for Reconsideration.
reconsideration, to wit: It appears that prior to the promulgation of the CA’s Decision, this Court issued Administrative Matter (A.M.) No. 00-2-10-SC which took effect
on 1 May 2000, amending Rule 4, Sec. 7 and Sec. 13 of Rule 41 of the 1997 Revised Rules of Court. The circular expressly provided that
The Supreme Court may promulgate procedural rules in all courts. It has the sole prerogative to amend, repeal or even establish new rules for trial courts may, motu proprio or upon motion, dismiss an appeal for being filed out of time or for nonpayment of docket and other
a more simplified and inexpensive process, and the speedy disposition of cases. In the rules governing appeals to it and to the Court of lawful fees within the reglementary period. Subsequently, Circular No. 48-2000 13 was issued on 29 August 2000 and was addressed to
Appeals, particularly Rules 42, 43 and 45, the Court allows extensions of time, based on justifiable and compelling reasons, for parties to file all lower courts.
their appeals. These extensions may consist of 15 days or more.
By virtue of the amendment to Sec. 41, the CA upheld the questioned Orders of the trial court by issuing the assailed Amended Decision 14 in the
The fresh period rule may be applied to this case, for the Court has already retroactively extended the fresh period rule to "actions pending and present Petition granting respondents’ Motion for Reconsideration.
undetermined at the time of their passage and this will not violate any right of a person who may feel that he is adversely affected, inasmuch as
there are no vested rights in rules of procedure." 27According to De los Santos v. Vda. de Mangubat:28 The CA’s action prompted petitioner to file a Motion for Reconsideration alleging that SC Circular No. 48-2000 should not be given
retroactive effect. It also alleged that the CA should consider the case as exceptionally meritorious. Petitioner’s counsel, Atty. Rexes
Procedural law refers to the adjective law which prescribes rules and forms of procedure in order that courts may be able to V. Alejano, explained that he was yet to familiarize himself with the Revised Rules of Court, which became effective a little over a
administer justice. Procedural laws do not come within the legal conception of a retroactive law, or the general rule against the month before he filed the Notice of Appeal. He was thus not aware that the nonpayment of docket fees might lead to the dismissal of
retroactive operation of statues ― they may be given retroactive effect on actions pending and undetermined at the time of their the case.
passage and this will not violate any right of a person who may feel that he is adversely affected, insomuch as there are no vested
rights in rules of procedure. On 30 May 2002, the CA issued the assailed Resolution 15 denying petitioner’s Motion for Reconsideration.
Xxxxxxxx
WHEREFORE, the Court DENIES the petition for review on certiorari; AFFIRMS the decision promulgated on November 20, 2002; and Hence, this Petition.
ORDERS the petitioner to pay the costs of suit.
SC:

ISSUE
W/N the appeal was dismissible for failure to pay docket fees, substantial justice demands that procedural rules be relaxed in this
Panay Railways, Inc. v. Heva Management, case.

TOPIC: Retroactive application of procedural rules


RULING
FACTS: The Petition has no merit.
Agreement: Statutes and rules regulating the procedure of courts are considered applicable to actions pending and unresolved at the time of
On 20 April 1982, petitioner Panay Railways Inc., a government-owned and controlled corporation, executed a Real Estate Mortgage Contract their passage. Procedural laws and rules are retroactive in that sense and to that extent. The effect of procedural statutes and rules
covering several parcels of lands, including Lot No. 6153, in favor of Traders Royal Bank (TRB) to secure ₱ 20 million worth of loan and credit on the rights of a litigant may not preclude their retroactive application to pending actions. This retroactive application does not
accommodations. Petitioner excluded certain portions of Lot No. 6153: that already sold to Shell Co., Inc. referred to as 6153-B, a road referred violate any right of a person adversely affected. Neither is it constitutionally objectionable. The reason is that, as a general rule, no
to as 6153-C, and a squatter area known as 6153-D vested right may attach to or arise from procedural laws and rules. It has been held that "a person has no vested right in any
Cause of the dispute: particular remedy, and a litigant cannot insist on the application to the trial of his case, whether civil or criminal, of any other than the
Petitioner failed to pay its obligations to TRB, prompting the bank to extra-judicially foreclose the mortgaged properties including Lot No. 6153. existing rules of procedure.”
On 20 January 1986, a Certificate of Sale was issued in favor of the bank as the highest bidder and purchaser. Consequently, the sale of Lot
No. 6153 was registered with the Register of Deeds on 28 January 1986 and annotated at the back of the transfer certificates of title (TCT) More so when, as in this case, petitioner admits that it was not able to pay the docket fees on time. Clearly, there were no substantive rights to
covering the mortgaged properties. speak of when the RTC dismissed the Notice of Appeal.
Thereafter, TRB caused the consolidation of the title in its name on the basis of a Deed of Sale and an Affidavit of Consolidation after petitioner
failed to exercise the right to redeem the properties. The corresponding TCTs were subsequently issued in the name of the bank. As early as 1932, in Lazaro v. Endencia, 17 we have held that the payment of the full amount of the docket fees is an indispensable step for the
perfection of an appeal. The Court acquires jurisdiction over any case only upon the payment of the prescribed docket fees. 18
Case originally filed:
On 12 February 1990, TRB filed a Petition for Writ of Possession against petitioner. During the proceedings, petitioner, through its duly Moreover, the right to appeal is not a natural right and is not part of due process. It is merely a statutory privilege, which may be exercised only
authorized manager and officer-in-charge and with the assistance of counsel, filed a Manifestation and Motion to Withdraw Motion for in accordance with the law. 19
Suspension of the Petition for the issuance of a writ of possession
It was only in 1994 that petitioner realized that the extrajudicial foreclosure included some excluded properties in the mortgage contract. Thus, We have repeatedly stated that the term "substantial justice" is not a magic wand that would automatically compel this Court to
on 19 August 1994, it filed a Complaint for Partial Annulment of Contract to Sell and Deed of Absolute Sale with Addendum; Cancellation of suspend procedural rules. Procedural rules are not to be belittled or dismissed simply because their non-observance may result in
Title No. T-89624; and Declaration of Ownership of Real Property with Reconveyance plus Damages. 3 prejudice to a party’s substantive rights. Like all other rules, they are required to be followed, except only for the most persuasive of
It then filed an Amended Complaint4 on 1 January 1995 and again filed a Second Amended Complaint 5 on 8 December 1995. reasons when they may be relaxed to relieve litigants of an injustice not commensurate with the degree of their thoughtlessness in
not complying with the procedure prescribed. 20
Defendant’s answer:
Meanwhile, respondents filed their respective Motions to Dismiss on these grounds: (1) petitioner had no legal capacity to sue; (2) We cannot consider counsel’s failure to familiarize himself with the Revised Rules of Court as a persuasive reason to relax the
there was a waiver, an abandonment and an extinguishment of petitioner’s claim or demand; (3) petitioner failed to state a cause of application of the Rules. It is well-settled that the negligence of counsel binds the client. This principle is based on the rule that any
action; and (4) an indispensable party, namely TRB, was not impleaded. act performed by lawyers within the scope of their general or implied authority is regarded as an act of the client. Consequently, the
mistake or negligence of the counsel of petitioner may result in the rendition of an unfavorable judgment against it.
RTC:
On 18 July 1997, the RTC issued an Order 6 granting the Motion to Dismiss of respondents. It held that the Manifestation and Motion filed
by petitioner was a judicial admission of TRB’s ownership of the disputed properties. The trial court pointed out that the Manifestation was
executed by petitioner’s duly authorized representative with the assistance of counsel. This admission thus operated as a waiver barring Tan v. Court of Appeals, G.R. No. 136368, January 16, 2002
petitioner from claiming otherwise.
TOPIC: Exceptions to the retroactive application of procedural rules
The clerk of court of the appellate court entered in the Book of Entries of Judgement the decision in CA-G.R. CV No. 33657 and issued the
FACTS: corresponding Entry of Judgment which, on its face, stated that the said decision "has on October 21, 1995 become final and executory." 6
This is a petition for review of the Decision of the Court of Appeals dated July 15, 1998 1 and its Resolution dated November 9, 1998 2 denying
petitioner's motion for reconsideration in CA-G.R. SP-41738. The respondents Magdangal filed in the trial court a Motion for Consolidation and Writ of Possession. 7 They alleged that the 120-day period of
redemption of the petitioner has expired. They reckoned that the said period began 15 days after October 5, 1995, the date when the finality of
Agreement: the judgment of the trial court as affirmed by the appellate court commenced to run.
"Involved in this case is a parcel of land, designated as Lot No. 645-C, with an area of 34,829 square meters, more or less, situated in
Bunawan, Davao City. The lot was once covered by TCT No. T-72067 of the Registry of Deeds of Davao City in the name of the late Jaime C. On the other hand, petitioner filed on March 27, 1996 a motion for execution in the appellate court praying that it "direct the court a quo to issue
Tan (Tan, for short) married to Praxedes V. Tan. the corresponding writ of execution in Civil Case No. 19049-88." 8 On April 17, 1996, petitioner deposited with the clerk of court the repurchase
On January 22, 1981, Tan, for a consideration of P59,200.00, executed a deed of absolute sale over the property in question in favor of price of the lot plus interest as ordered by the decision.
spouses Jose Magdangal and Estrella Magdangal. Simultaneous with the execution of this deed, the same contracting parties entered into
another agreement whereunder Tan given one (1) year within which to redeem or repurchase the property. On June 10, 1996, the trial court allowed the petitioner to redeem the lot in question. It ruled that the 120-day redemption period should be
Tan failed to redeem the property until his death on January 4, 1988. reckoned from the date of Entry of Judgment in the appellate court or from March 13, 1996. 9 The redemption price was deposited on April 17,
1996. As aforestated, the Court of Appeals set aside the ruling of the trial court.

From 1991-1996, the years relevant to the case at bar, the rule that governs finality of judgment is Rule 51 of the Revised Rules of Court. Its
sections 10 and 11 provide:
Cause of the dispute: "SEC. 10. Entry of judgments and final resolutions. - If no appeal or motion for new trial or reconsideration is filed within the time provided in
and these Rules, the judgment or final resolution shall forthwith be entered by the clerk in the book of entries of judgments. The date when the
Case originally filed: judgment or final resolution becomes executory shall be deemed as the date of its entry. The record shall contain the dispositive part of the
On May 2, 1988, Tan's heirs filed before the Regional Trial Court at Davao City a suit against the Magdangals for reformation of instrument. judgment or final resolution and shall be signed by the clerk, with a certificate that such judgment or final resolution has become final and
Docketed as CIVIL CASE NO. 19049-88, the complaint alleged that, while Tan and the Magdangals denominated their agreement as deed of executory. (2a, R36)
absolute sale, their real intention was to conclude an equitable mortgage.
Barely hours after the complaint was stamped 'received,' the Magdangals were able to have Tan's title over the lot in question canceled and to SEC. 11. Execution of judgment. - Except where the judgment or final order or resolution, or a portion thereof, is ordered to be immediately
secure in their names TCT No. T-134470. This development prompted the heirs of Tan, who were to be later substituted by Jaime V. Tan, Jr. executory, the motion for its execution may only be filed in the proper court after its entry.
(Tan, Jr.) as plaintiff, to file a supplemental complaint.
In original actions in the Court of Appeals, its writ of execution shall be accompanied by a certified true copy of the entry of judgment or final
RTC: resolution and addressed to any appropriate officer for its enforcement.
On June 4, 1991, Branch 11 of the Regional Trial Court of Davao City rendered judgment:
1. The Deed of Absolute Sale (Exhibits B, B-1) is, in accordance with the true intention of the parties, hereby declared and reformed an In appealed cases, where the motion for execution pending appeal is filed in the Court of Appeals at a time that it is in possession of the
equitable mortgage; original record or the record on appeal, the resolution granting such motion shall be transmitted to the lower court from which the case
2. The plaintiff is ordered to pay the defendants within 120 days after the finality of this decision P59,200 plus interest at the rate of 12% per originated, together with a certified true copy of the judgment or final order to be executed, with a directive for such court of origin to issue the
annum from May 2, 1988, the date the complaint was filed, until paid; proper writ for its enforcement."
3. In order to avoid multiplicity of suits and to fully give effect to the true intention of the parties, upon the payment of the aforesaid amount, TCT
No. T-134470 in the name of defendants Jose Magdangal and Estrella Magdangal (Exh. 13) and shall be deemed canceled and null and void It is evident that if we apply the old rule on finality of judgment, petitioner redeemed the subject property within the 120-day period of
and TCT No. T-72067 in the name of Jaime C. Tan and Praxedes Valles Tan (Exh. A) be reinstated). redemption reckoned from the appellate court's entry of judgment. The appellate court, however, did not apply the old rule but the 1997
From the above, the Magdangals appealed to this Court in CA Revised Rules of Civil Procedure. In fine, it applied the new rule retroactively and we hold that given the facts of the case at bar this is an error.

CA: There is no dispute that rules of procedure can be given retroactive effect. This general rule, however, has well-delineated
Affirmed in toto the appealed decision of the lower court. Copy of this affirmatory judgment was each received by the Magdangals and Tan, Jr. exceptions. We quote author Agpalo
on October 5, 1995. ****Procedural laws are adjective laws which prescribe rules and forms of procedure of enforcing rights or obtaining redress for their
On March 13, 1996, the Clerk of this Court entered in the Book of Entries of Judgment the Decision and issued the corresponding Entry of invasion; they refer to rules of procedure by which courts applying laws of all kinds can properly administer justice. They include
Judgment which, on its face, stated that the said Decision 'has become final and executory' rules of pleadings, practice and evidence. As applied to criminal law, they provide or regulate the steps by which one who commits a
crime is to be punished.
On March 21, 1996, the Magdangals filed in the lower court a MOTION FOR CONSOLIDATION AND WRIT OF POSSESSION, therein alleging
that they did not appeal from the aforesaid decision of this Court, adding '[T]hat the appealed judgment of the Court of Appeals has become The general rule that statutes are prospective and not retroactive does not ordinarily apply to procedural laws. It has been held that
final and executory 15 days from October 5, 1995 or up to October 20, 1995, which the 120 days redemption period commences. And noting "a retroactive law, in a legal sense, is one which takes away or impairs vested rights acquired under laws, or creates a new obligation
that the redemption period has expired without Tan, Jr. exercising his option, the Magdangals thus prayed that the title 'in the name of Jaime C. and imposes a new duty, or attaches a new disability, in respect of transactions or considerations already past. Hence, remedial
Tan and Praxedes Tan be consolidated and confirmed in the name of the (Magdangals) x x x and pending such issuance, a writ of possession statutes or statutes relating to remedies or modes of procedure, which do not create new or take away vested rights, but only
be ordered issued operate in furtherance of the remedy or confirmation of rights already existing, do not come within the legal conception of a
retroactive law, or the general rule against the retroactive operation of statutes." The general rule against giving statutes retroactive
In opposition to this motion, Tan, Jr. alleged, among other things, that until an entry of judgment has been issued by the Court of Appeals and operation whose effect is to impair the obligations of contract or to disturb vested rights does not prevent the application of statutes
copy thereof furnished the parties, the appealed decision of the court a quo in this case cannot be considered final and executory. Pressing the to proceedings pending at the time of their enactment where they neither create new nor take away vested rights. A new statute
point, Tan, Jr., citing Cueto vs. Collantes, infra., would then assert that the period of redemption on his part commenced to run from receipt of which deals with procedure only is presumptively applicable to all actions - those which have accrued or are pending.
entry of judgment in CA-G.R. CV No. 33657.
Statutes regulating the procedure of the courts will be construed as applicable to actions pending and undetermined at the time of
Meanwhile, Tan, Jr. via a motion for execution dated March 27, 1996, which he filed directly with this court, prayed this court to direct the their passage. Procedural laws are retroactive in that sense and to that extent. The fact that procedural statutes may somehow affect
court a quo to issue the corresponding writ of execution in Civil Case No. 19049-88. In a related move, Tan, Jr. filed on April 16, 1996, a the litigants' rights may not preclude their retroactive application to pending actions. The retroactive application of procedural laws
MANIFESTATION AND MOTION therein advising the court a quo of his intention to redeem the property in question and of the fact that, on is not violative of any right of a person who may feel that he is adversely affected. Nor is the retroactive application of procedural
such date, he has deposited with its clerk of court the repurchase price, plus interest, as required by its original decision. By way of relief, Tan, statutes constitutionally objectionable. The reason is that as a general rule no vested right may attach to, nor arise from, procedural
Jr. prayed that the Magdangals be ordered to claim the amount thus deposited and the Register of Deeds of Davao City, to reinstate the title of laws. It has been held that "a person has no vested right in any particular remedy, and a litigant cannot insist on the application to
Jaime Tan and Praxedes Tan. the trial of his case, whether civil or criminal, of any other than the existing rules of procedure."

Jointly acting on the aforementioned MOTON FOR CONSOLIDATION AND WRIT OF POSSESION of the Magdangals, MANIFESTATION Thus, the provision of Batas Bilang 129 in Section 39 thereof prescribing that "no record on appeal shall be required to take an
AND MOTION of Tan, Jr. he court a quo presided by the respondent judge, came out with the first challenged order of June 10, 1996: appeal" is procedural in nature and should therefore be applied retroactively to pending actions. Hence, the question as to whether
The Motion for Consolidation and a Writ of Possession is hereby DENIED for lack of merit. an appeal from an adverse judgment should be dismissed for failure of appellant to file a record on appeal within thirty days as
The deposit of the amount of P116,032.00 made by plaintiff with the Office of Court s hereby considered full payment of the redemption price required under the old rules, which question is pending resolution at the time Batas Bilang 129 took effect, became academic upon
and the Clerk of Court is hereby ordered to deliver said amount to herein defendants. the effectivity of said law because the law no longer requires the filing of a record on appeal and its retroactive application removed
the legal obstacle to giving due course to the appeal. A statute which transfers the jurisdiction to try certain cases from a court to a
The Register of Deeds of Davao City x x x is hereby directed to cancel TCT No. T-134470 in the name of Jose Magdangal and Estrella quasi-judicial tribunal is a remedial statute that is applicable to claims that accrued before its enactment but formulated and filed
Magdangal and, thereafter, to reinstate TCT No. 72067 in the name of Jaime C. Tan and Praxedes Valles Tan and to submit her compliance after it took effect, for it does not create new nor take away vested rights. The court that has jurisdiction over a claim at the time it
thereto within ten (10) days from receipt of this Order. accrued cannot validly try the claim where at the time the claim is formulated and filed the jurisdiction to try it has been transferred
by law to a quasi-judicial tribunal, for even actions pending in one court may be validly taken away and transferred to another and no
Explaining her action, the respondent judge wrote in the same order: litigant can acquire a vested right to be heard by one particular court.
'Following the ruling of the Supreme Court in Cueto vs. Collantes, et al., 97 Phil. 325, the 120 days period for plaintiff to pay the amount of
P59,200.00 plus interest x x x should be reckoned from the date of Entry of Judgment x x x which was March 13, 1996. The plaintiff made a ****Exceptions to the rule:****
deposit on April 17, 1996 well within the 120-day period mandated by the decision of this Court.' The rule that procedural laws are applicable to pending actions or proceedings admits certain exceptions. The rule does not apply
where the statute itself expressly or by necessary implication provides that pending actions are excepted from its operation, or
SC: where to apply it to pending proceedings would impair vested rights. Under appropriate circumstances, courts may deny the
ISSUE retroactive application of procedural laws in the event that to do so would not be feasible or would work injustice. Nor may
ISSUE: w/n the section 1, Rule 39 of the 1997 Revised Rules of Procedure should not be given retroactive effect in this case procedural laws be applied retroactively to pending actions if to do so would involve intricate problems of due process or impair the
independence of the courts."

RULING We hold that section 1, Rule 39 of the 1997 Revised Rules of Procedure should not be given retroactive effect in this case as it would
NO result in great injustice to the petitioner. Undoubtedly, petitioner has the right to redeem the subject lot and this right is a substantive
right. Petitioner followed the procedural rule then existing as well as the decisions of this Court governing the reckoning date of the
period of redemption when he redeemed the subject lot. Unfortunately for petitioner, the rule was changed by the 1997 Revised Rules process, minimize and/or rectify any error of judgment. While we aim to resolve cases with dispatch and to have judgments of courts become
of Procedure which if applied retroactively would result in his losing the right to redeem the subject lot. It is difficult to reconcile the final at some definite time, we likewise aspire to deliver justice fairly.
retroactive application of this procedural rule with the rule of fairness. Petitioner cannot be penalized with the loss of the subject lot
when he faithfully followed the laws and the rule on the period of redemption when he made the redemption. In this case, the new period of 15 days eradicates the confusion as to when the 15-day appeal period should be counted – from receipt of
notice of judgment (March 3, 1998) or from receipt of notice of "final order" appealed from (July 22, 1998).
Petitioner fought to recover this lot from 1988. To lose it because of a change of procedure on the date of reckoning of the period of
redemption is inequitous. The manner of exercising the right cannot be changed and the change applied retroactively if to do so will To recapitulate, a party litigant may either file his notice of appeal within 15 days from receipt of the RTC’s decision or file it within 15
defeat the right of redemption of the petitioner which is already vested days from receipt of the order (the “final order”) denying his motion for new trial or motion for reconsideration. Obviously, the new
15-day period may be availed of only if either motion is filed; otherwise, the decision becomes final and executory after the lapse of
the original appeal period provided in Rule 41, Section 3.
Neypes v. Court of Appeals, G.R. No. 141524, September 14, 2005 Petitioners here filed their notice of appeal on July 27, 1998 or five days from receipt of the order denying their motion for reconsideration on
July 22, 1998. Hence, the notice of appeal was well within the fresh appeal period of 15 days, as already discussed.
TOPIC: Fresh-Period Rule
Note: The “FRESH PERIOD RULE” do not apply to Rule 64 (Review of Judgments and Final Orders or Resolutions of the Commission on
FACTS: Elections and the Commission on Audit) because Rule 64 is derived from the Constitution
Petitioners filed an action for annulment of judgment and titles of land and/or reconveyance and/or reversion with preliminary injunction before
the RTC against the private respondents

RTC: Phil. Carpet Manufacturing v. Tagyamon, G.R. No. 191475, December 11, 2013
Later, in an order, the trial court dismissed petitioners’ complaint on the ground that the action had already prescribed. Petitioners allegedly
received a copy of the order of dismissal on March 3, 1998 and, on the 15th day thereafter or on March 18, 1998, filed a motion for TOPIC: Philippine Courts; Nature
reconsideration.
On July 1, 1998, the trial court issued another order dismissing the motion for reconsideration which petitioners received on July 22, 1998. Five FACTS:
days later, on July 27, 1998, petitioners filed a notice of appeal and paid the appeal fees on August 3, 1998. Petitioner Philippine Carpet Manufacturing Corporation (PCMC) is a corporation registered in the Philippines engaged in the business of
On August 4, 1998, the court a quo denied the notice of appeal, holding that it was filed eight days late. This was received by petitioners on manufacturing wool and yarn carpets and rugs. 4 Respondents were its regular and permanent employees, but were affected by petitioner’s
July 31, 1998. Petitioners filed a motion for reconsideration but this too was denied in an order dated September 3, 1998. retrenchment and voluntary retirement programs.
Agreement:
CA:
Cause of the dispute:
Via a petition for certiorari and mandamus under Rule 65, petitioners assailed the dismissal of the notice of appeal before the On March 15, 2004, Tagyamon,5 Luna,6 Badayos,7 Dela Cruz,8 and Comandao9 received a uniformly worded Memorandum of dismissal, to wit:
CA. In the appellate court, petitioners claimed that they had seasonably filed their notice of appeal. This is to inform you that in view of a slump in the market demand for our products due to the un-competitiveness of our price, the company is
constrained to reduce the number of its workforce. The long-term effects of September 11 and the war in the Middle East have greatly affected
They argued that the 15-day reglementary period to appeal started to run only on July 22, 1998 since this was the day they the viability of our business and we are left with no recourse but to reorganize and downsize our organizational structure.
received the final order of the trial court denying their motion for reconsideration. When they filed their notice of appeal on July
27, 1998, only five days had elapsed and they were well within the reglementary period for appeal. We wish to inform you that we are implementing a retrenchment program in accordance with Article 283 of the Labor Code of the Philippines,
as amended, and its implementing rules and regulations.
On September 16, 1999, the CA dismissed the petition. It ruled that the 15-day period to appeal should have been reckoned
from March 3, 1998 or the day they received the February 12, 1998 order dismissing their complaint. According to the appellate In this connection, we regret to advise you that you are one of those affected by the said exercise, and your employment shall be terminated
court, the order was the “final order” appealable under the Rules. effective at the close of working hours on April 15, 2004.

As to Marcos, Ilao, and Nemis, they claimed that they were dismissed effective March 31, 2004, together with fifteen (15) other employees on
SC: the ground of lack of market/slump in demand. 11 PCMC, however, claimed that they availed of the company’s voluntary retirement program and,
ISSUE in fact, voluntarily executed their respective Deeds of Release, Waiver, and Quitclaim.
(1) Whether or not receipt of a final order triggers the start of the 15-day reglmentary period to appeal, the February 12, 1998 order dismissing
the complaint or the July 1, 1998 order dismissing the Motion for Reconsideration. Case originally filed:
(2) Whether or not petitioners file their notice of appeal on time. Claiming that they were aggrieved by PCMC’s decision to terminate their employment, respondents filed separate complaints for illegal
dismissal against PCMC, Pacific Carpet Manufacturing Corporation, Mr. Patricio Lim and Mr. David Lim. These cases were later consolidated

RULING Plaintiff’s (respondents) averments:


(1) The July 1, 1998 order dismissing the motion for reconsideration should be deemed as the final order. Respondents primarily relied on the Supreme Court’s decision in Philippine Carpet Employees Association (PHILCEA) v. Hon. Sto. Tomas
In the case of Quelnan v. VHF Philippines, Inc., the trial court declared petitioner non-suited and accordingly dismissed his complaint. Upon (Philcea case),13 as to the validity of the company’s retrenchment program. They further explained that PCMC did not, in fact, suffer losses
receipt of the order of dismissal, he filed an omnibus motion to set it aside. When the omnibus motion was filed, 12 days of the 15-day period to shown by its acts prior to and subsequent to their termination. 14
appeal the order had lapsed. He later on received another order, this time dismissing his omnibus motion. He then filed his notice of appeal.
But this was likewise dismissed ― for having been filed out of time. The court a quo ruled that petitioner should have appealed within 15 days Plaintiff’s prayer:
after the dismissal of his complaint since this was the final order that was appealable under the Rules. The SC reversed the trial court and They also insisted that their acceptance of separation pay and signing of quitclaim is not a bar to the pursuit of illegal dismissal case.
declared that it was the denial of the motion for reconsideration of an order of dismissal of a complaint which constituted the final order as it
was what ended the issues raised there. Defendant’s (petitioner) answer:
PCMC, for its part, defended its decision to terminate the services of respondents being a necessary management prerogative. It pointed out
This pronouncement was reiterated in the more recent case of Apuyan v. Haldeman et al. where the SC again considered the order denying that as an employer, it had no obligation to keep in its employ more workers than are necessary for the operation of his business. Thus, there
petitioner’s motion for reconsideration as the final order which finally disposed of the issues involved in the case. Based on the aforementioned was an authorized cause for dismissal. Petitioners also stressed that respondents belatedly filed their complaint as they allowed almost three
cases, the SC sustained petitioners’ view that the order dated July 1, 1998 denying their motion for reconsideration was the final order years to pass making the principle of laches applicable. Considering that respondents accepted their separation pay and voluntarily executed
contemplated in the Rules. deeds of release, waiver and quitclaim, PCMC invoked the principle of estoppel on the part of respondents to question their separation from the
service. Finally, as to Marcos, Ilao and Nemis, PCMC emphasized that they were not dismissed from employment, but in fact they voluntarily
(2) YES. retired from employment to take advantage of the company’s program
To standardize the appeal periods provided in the Rules and to afford litigants fair opportunity to appeal their cases, the Court deems it practical
to allow a fresh period of 15 days within which to file the notice of appeal in the Regional Trial Court, counted from receipt of the order Labor Arbiter:
dismissing a motion for a new trial or motion for reconsideration.
On August 23, 2007, Labor Arbiter (LA) Donato G. Quinto, Jr. rendered a Decision dismissing the complaint for lack of merit. 17 The LA found no
Henceforth, this "fresh period rule" shall also apply to Rule 40 governing appeals from the Municipal Trial Courts to the Regional Trial Courts; flaw in respondents’ termination as they voluntarily opted to retire and were subsequently re-employed on a contractual basis then regularized,
Rule 42 on petitions for review from the Regional Trial Courts to the Court of Appeals; Rule 43 on appeals from quasi-judicial agencies to the terminated from employment and were paid separation benefits. 18 In view of respondents’ belated filing of the complaint, the LA concluded that
Court of Appeals and Rule 45 governing appeals by certiorari to the Supreme Court. The new rule aims to regiment or make the appeal period such action is a mere afterthought designed primarily for respondents to collect more money, taking advantage of the 2006 Supreme Court
uniform, to be counted from receipt of the order denying the motion for new trial, motion for reconsideration (whether full or partial) or any final decision
order or resolution.
NCLR:
The SC thus held that petitioners seasonably filed their notice of appeal within the fresh period of 15 days, counted from July 22, 1998 (the date On appeal, the National Labor Relations Commission (NLRC) sustained the LA decision. 20 In addition to the LA ratiocination, the NLRC
of receipt of notice denying their motion for reconsideration). This pronouncement is not inconsistent with Rule 41, Section 3 of the Rules emphasized the application of the principle of laches for respondents’ inaction for an unreasonable period.
which states that the appeal shall be taken within 15 days from notice of judgment or final order appealed from. The use of the
disjunctive word “or” signifies disassociation and independence of one thing from another. It should, as a rule, be construed in the Respondents elevated the matter to the CA in a petition for certiorari.
sense in which it ordinarily implies. Hence, the use of “or” in the above provision supposes that the notice of appeal may be filed CA:
within 15 days from the notice of judgment or within 15 days from notice of the “final order,” which we already determined to refer to the In reversing the earlier decisions of the LA and the NLRC, the CA refused to apply the principle of laches, because the case was instituted prior
July 1, 1998 order denying the motion for a new trial or reconsideration. to the expiration of the prescriptive period set by law which is four years. It stressed that said principle cannot be invoked earlier than the
Neither does this new rule run counter to the spirit of Section 39 of BP 129 which shortened the appeal period from 30 days to 15 expiration of the prescriptive period. 21 Citing the Court’s decision in the Philcea case, the CA applied the doctrine of stare decisis, in view of the
days to hasten the disposition of cases. The original period of appeal (in this case March 3-18, 1998) remains and the requirement for strict similar factual circumstances of the cases. As to Ilao, Nemis and Marcos, while acknowledging their voluntary resignation, the CA found the
compliance still applies. The fresh period of 15 days becomes significant only when a party opts to file a motion for new trial or motion for same not a bar to the illegal dismissal case because they did so on the mistaken belief that PCMC was losing money. 22 With the foregoing
reconsideration. In this manner, the trial court which rendered the assailed decision is given another opportunity to review the case and, in the findings, the CA ordered that respondents be reinstated with full backwages less the amounts they received as separation pay. In case of
impossibility of reinstatement, the CA ordered PCMC to pay respondents backwages and in lieu of reinstatement, separation pay equal to one same employees.42 And while it is true that petitioner paid them separation pay, the payment was in the nature of separation and not retirement
month pay or ½ month pay for every year of service whichever is higher, plus moral damages pay. In other words, payment was made because of the implementation of the retrenchment program and not because of retirement. 43 As their
application for availing of the company’s voluntary retirement program was based on the wrong premise, the intent to retire was not clearly
SC: established, or rather that the retirement is involuntary. Thus, they shall be considered discharged from employment. 44 Consequently, they shall
Priscilla then brought this appeal, averring that the CA thereby erred in not outrightly dismissing Javellana’s appeal because: (a) the June 21, be treated as if they are in the same footing as the other respondents herein and the union members in the Philcea case.
2000 RTC order was not appealable XXXXXXX
RAYOS VS CITY OF MANILA
On his part, Javellana countered that the errors being assigned by Priscilla involved questions of fact not proper for the Court to review through
CITATION : G.R. No. 196063, December 14, 2011
petition for review on certiorari; that the June 21, 2000 RTC order, being a final order, was appealable; that his appeal was perfected on time;
FACTS : This petition, captioned as a petition for review on Certiorari and declaratory
SC:
relief, assails the Order of 6 January 2011of the Regional Trial Court of Manila, Branch 49, denying reconsideration of the trial courts Order of
IN RENDERING ITS DISPUTED DECISION AND RESOLUTION, THE COURT A QUO HAS DECIDED A QUESTION OF SUBSTANCE NOT
11 March 2010which denied the motion to dismiss filed by petitioners Orlando A. Rayos, Fe A. Rayos Dela Paz, and Engr. Manuel A.
IN ACCORD WITH LAW AND/OR ESTABLISHED JURISPRUDENCE.
Rayos.The present case originated from a complaint for eminent domain filed by respondent City of Manila against Remedios V.
De Caronongan, Patria R. Serrano, Laureano M. Reyes, Paz B. Sison, Teofila B. Sison, Leticia R. Ventanilla, Rosalinda
Petitioners contend that the Philcea case decided by this Court and relied upon by the CA in the assailed decision was based on erroneous
R. Barrozo (defendants), docketed as Civil Case No. 03108154.
factual findings, inapplicable financial statement, as well as erroneous analysis of such financial statements. 25 They, thus, implore the Court to
revisit the cited case in order to dispense with substantial justice. 26 They explain that the Court made conclusions based on erroneous
information. Petitioners also insist that the doctrines of res judicata and law of the case are not applicable, considering that this case does not
involve the same parties as the Philcea case.27 They likewise point out that not all respondents were involuntarily separated on the ground of In its Complaint,the City of Manila alleged that it passed Ordinance No. 7949 authorizing the City Mayor to acquire by expropriation,
redundancy as some of them voluntarily availed of the company’s Voluntary Separation Program. 28They further contend that respondents are negotiation or by any other legal means the parcel of land co-owned by defendants, which is covered by TCT No. 227512 and with an area of
guilty not only of laches but also of estoppel in view of their inaction for an unreasonable length of time to assail the alleged illegal dismissal 1,182.20 square meters. The City of Manila offered to purchase the property at P1,000.00 per square meter.
and in voluntarily executing a release, quitclaim and waiver In their Answer,defendants conveyed their willingness to sell the property to the City of Manila, but at the price of P50,000.00 per square
meter which they claimed was the fair market value of the land at the time.
On 11 March 2010, the trial court denied the motion to dismiss. The trial court ruled that the motion to dismiss did not show any compelling
ISSUE reason to convince the court that the doctrine of stare decisis applies. Petitioners failed to demonstrate how or why the facts in this case are
The main issue sought to be determined in this case is the validity of respondents’ dismissal from employment. similar with the cited cases in order that the issue in this case be resolved in the same manner.

Petitioners contend that they either voluntarily retired from the service or terminated from employment based on an authorized cause. The LA ISSUE : Is Motion to dismiss appealable?
and the NLRC are one in saying that the dismissal was legal. The CA, however, no longer discussed the validity of the ground of termination.
Rather, it applied the Court’s decision in the Philcea case where the same ground was thoroughly discussed. In other words, the appellate court RULING : An order denying a motion to dismiss is interlocutory and not appealable. An order denying a motion to dismiss
applied the doctrine of stare decisis and reached the same conclusion as the earlier case. does not finally dispose of the case, and in effect, allows the case to proceed until the final adjudication thereof by the court. As such, it is
merely interlocutory in nature and thus, not appealable.
Under the doctrine of stare decisis, when a court has laid down a principle of law as applicable to a certain state of facts, it will adhere to that Clearly, no appeal, under Rule 45 of the Rules of Court, may be taken from an interlocutory order. In case of denial of an
principle and apply it to all future cases in which the facts are substantially the same, even though the parties may be different. 36 Where the interlocutory order, the immediate remedy available to the aggrieved party is to file a special civil action for certiorari under Rule 65 of the Rules
facts are essentially different, however, stare decisis does not apply, for a perfectly sound principle as applied to one set of facts might be of Court.
entirely inappropriate when a factual variant is introduced. 37 In this case, since the trial courts order denying the motion to dismiss is not appealable, petitioners should
have filed a petition for certiorari under Rule 65 to assail such order, and not a petition for review on certiorari under Rule 45 of the Rules of
The question, therefore, is whether the factual circumstances of this present case are substantially the same as the Philcea case. Court. For being a wrong remedy, the present petition deserves outright dismissal.
Even if the Court treats the present petition as a petition for certiorari under Rule 65, which is the proper remedy to challenge
the order denying the motion to dismiss, the same must be dismissed for violation of the principle of hierarchy of courts. This well-settled
RULING principle dictates that petitioners should file the petition for certiorari with the Court of Appeals, and not directly with this Court.
We answer in the affirmative.
RATIONALE : This Courts original jurisdiction to issue writs of certiorari is not exclusive. It is shared by this Court with
This case and the Philcea case involve the same period which is March to April 2004; the issuance of Memorandum to employees informing Regional Trial Courts and with the Court of Appeals. This concurrence of jurisdiction is not,however, to be taken as according to parties seeking
them of the implementation of the cost reduction program; the implementation of the voluntary retirement program and retrenchment program, any of the writs an absolute, unrestrained freedom of choice of the court to which application therefor will be directed. There is after all a
except that this case involves different employees; the execution of deeds of release, waiver, and quitclaim, and the acceptance of separation hierarchy of courts. That hierarchy is determinative of the venue of appeals, and also serves as a general determinant of the appropriate
pay by the affected employees. forum for petitions for the extraordinary writs. A becoming regard for that judicial hierarchy most certainly indicates that petitions for the
issuance of extraordinary writs against first level (inferior) courts should be filed with the Regional Trial Court, and those against the latter, with
The illegality of the basis of the implementation of both voluntary retirement and retrenchment programs of petitioners had been thoroughly the Court of Appeals. A direct invocation of the Supreme Courts original jurisdiction to issue these writs should be allowed only when
ruled upon by the Court in the Philcea case. It discussed the requisites of both retrenchment and redundancy as authorized causes of there are special and important reasons therefor, clearly and specifically set out in the petition. This is [an] established policy. It is a
termination and that petitioners failed to substantiate them. In ascertaining the bases of the termination of employees, it took into consideration policy necessary to prevent inordinate demands upon the Courts time and attention which are better devoted to those matters within its
petitioners’ claim of business losses exclusive jurisdiction, and to prevent further over-crowding of the Courts docket.

In arriving at its conclusions, the Court took note of petitioners’ net sales, gross and net profits, as well as net income. The Court, thus, reached
the conclusion that the retrenchment effected by PCMC is invalid due to a substantive defect.
UNITED CLAIMANTS ASSOC. VS. NEA
The retrenchment effected by respondent Corporation is invalid due to a substantive defect, non-compliance with the substantial requirements
to effect a valid retrenchment; it necessarily follows that the termination of the employment of petitioner Union's members on such ground is,
likewise, illegal. As such, they (petitioner Union's members) are entitled to reinstatement with full backwages. CITATION : G.R. No. 187107, JANUARY 31. 2012

We find no reason to depart from the above conclusions which are based on the Court’s examination of the evidence presented by the parties
therein. As the respondents here were similarly situated as the union members in the Philcea case, and considering that the questioned
dismissal from the service was based on the same grounds under the same circumstances, there is no need to relitigate the issues presented FACTS : This is an original action for Injunction to restrain and/or prevent the
herein. In short, we adopt the Court’s earlier findings that there was no valid ground to terminate the employees. implementation of Resolution Nos. 46 and 59, dated July 10, 2003 and September 3, 2003, respectively, otherwise known as the National
Electrification Administration (NEA) Termination Pay Plan, issued by respondent NEA Board of Administrators (NEA Board).
A closer look at petitioners’ arguments would show that they want the Court to re-examine our decision in the Philcea case allegedly on the
ground that the conclusions therein were based on erroneous interpretation of the evidence presented. Petitioners are former employees of NEA who were terminated from their employment with the implementation of the assailed
resolutions.
Indeed, in Abaria v. National Labor Relations Commission, 39 although the Court was confronted with the same issue of the legality of a strike
that has already been determined in a previous case, the Court refused to apply the doctrine of stare decisis insofar as the award of Respondent NEA is a government-owned and/or controlled corporation created in accordance with Presidential Decree No. (PD) 269
backwages was concerned because of the clear erroneous application of the law. We held therein that the Court abandons or overrules issued on August 6, 1973. Under PD 269, Section 5(a)(5), the NEA Board is empowered to organize or reorganize NEAs staffing structure.
precedents whenever it realizes that it erred in the prior decision. 40 The Court’s pronouncement in that case is instructive:
The doctrine though is not cast in stone for upon a showing that circumstances attendant in a particular case override the great
benefits derived by our judicial system from the doctrine of stare decisis, the Court is justified in setting it aside. For the Court, as Thereafter, in order to enhance and accelerate the electrification of the whole country, including the privatization of the National Power
the highest court of the land, may be guided but is not controlled by precedent. Thus, the Court, especially with a new membership, Corporation, Republic Act No. (RA) 9136, otherwise known as the Electric Power Industry Reform Act of 2001 (EPIRA Law), was enacted,
is not obliged to follow blindly a particular decision that it determines, after re-examination, to call for a rectification . taking effect on June 26, 2001. The law imposed upon NEA additional mandates in relation to the promotion of the role of rural electric
(TOPIC: Philippine Courts; Nature) cooperatives to achieve national electrification.

The Abaria case, however, is not applicable in this case.1âwphi1 There is no reason to abandon the Court’s ruling in the Philcea case. Meanwhile, on August 28, 2002, former President Gloria Macapagal- Arroyo issued Executive Order No. 119 directing the NEA
Do we apply the aforesaid decision to all the respondents herein? Again, we answer in the affirmative. Board to submit a reorganization plan. Thus, the NEA Board issued the assailed resolutions.

Just like the union members in the Philcea case, respondents Tagyamon, Luna, Badayos, Dela Cruz, and Comandao received similarly worded On September 17, 2003, the Department of Budget and Management approved the NEA Termination Pay Plan.
memorandum of dismissal effective April 15, 2004 based on the same ground of slump in the market demand for the company’s products. As
such, they are similarly situated in all aspects as the union members. With respect to respondents Marcos, Nemis and Ilao, although they
applied for voluntary retirement, the same was not accepted by petitioner. Instead, it issued notice of termination dated March 6, 2004 to these
Thereafter, the NEA implemented an early retirement program denominated as the Early Leavers Program, giving incentives to acquired, continues until the case is finally terminated; that since its original Complaint, Amended and Supplemental Complaint and Second
those who availed of it and left NEA before the effectivity of the reorganization plan. The other employees of NEA were terminated effective Amended Complaint all primarily sought the nullification of the Deed of Assignment and Conveyance (DAC) transferring the Asset Pool in favor
December 31, 2003. of petitioner Home Guaranty Corporation (HGC), the subject matter of the case is clearly one which is incapable of pecuniary estimation; and,
that the court erred in holding that the case was a real action and that it evaded the payment of the correct docket fees computed on the basis
ISSUE : Whether or Not the Court has jurisdiction over the instance case. of the assessed value of the realties in the Asset Pool.
The record shows that, with the raffle of R-II Builders’ complaint before Branch 24 of the
RULING : YES. THE COURT HAS JURISDICTION OVER THE CASE. Manila RTC and said court’s grant of the application for temporary restraining order incorporated therein, HGC sought a preliminary hearing of
its affirmative defenses which included, among other grounds, lack of jurisdiction and improper venue. It appears that, at said preliminary
Evidently, the instant petition should have been filed with the RTC. However, as an exception to this general rule, the principle of hierarchy of hearing, it was established that R-II Builders’ complaint did not involve an intra-corporate dispute and that, even if it is, venue was improperly
courts may be set aside for special and important reasons. Such reason exists in the instant case involving as it does the employment of the laid since none of the parties maintained its principal office in Manila. While it is true, therefore, that R-II Builders had no hand in the raffling of
entire plantilla of NEA, more than 700 employees all told, who were effectively dismissed from employment in one swift stroke. This to the mind the case, it cannot be gainsaid that Branch 24 of the RTC Manila had no jurisdiction over the case. Rather than ordering the dismissal of the
of the Court entails its attention. complaint, however, said court issued the 2 January 2008 order erroneously ordering the re-raffle of the case. In Atwel v. Concepcion
Progressive Association, Inc. and Reyes v. Hon. Regional Trial Court of Makati, Branch 142 which involved SCCs trying and/or deciding cases
Moreover, the Court has made a similar ruling in National Power Corporation Drivers and Mechanics Association (NPC-DAMA) v. which were found to be civil in nature, this Court significantly ordered the dismissal of the complaint for lack of jurisdiction instead of simply
National Power Corporation (NPC).In that case, the NPC-DAMA also filed a petition for injunction directly with this Court assailing NPC Board directing the re-raffle of the case to another branch.
Resolution Nos. 2002-124 and 2002-125, both dated November 18, 2002, directing the termination of all employees of the NPC on January 31,
2003. Despite such apparent disregard of the principle of hierarchy of courts, the petition was given due course. We perceive no compelling ISSUE : Whether or not the Court had no jurisdiction over the instant case.
reason to treat the instant case differently. RULING : Even then, the question of the Manila RTC's jurisdiction over the case is tied up with R-II Builder's payment of
the correct docket fees which should be paid in full upon the filing of the pleading or other application which initiates an action or
proceeding. While it is, consequently, true that jurisdiction, once acquired, cannot be easily ousted, it is equally settled that a court acquires
ERNESTO DY VS. HON. PALAMOS jurisdiction over a case only upon the payment of the prescribed filing and docket fees. Already implicit from the filing of the complaint in the
City of Manila where the realties comprising the Asset Pool are located, the fact that the case is a real action is evident from the allegations of
CITATION : G.R. No. 187107, JANUARY 31. 2012 R-II Builders’ original Complaint, Amended and Supplemental Complaint and Second Amended Complaint which not only sought the
nullification of the DAC in favor of HGC but, more importantly, prayed for the transfer of possession of and/or control of the properties in the
FACTS : This petition for Certiorari under Rule 65 of the 1997 Revised Rules of Civil Asset Pool. Its current protestations to the contrary notwithstanding, no less than R-II Builders – in its opposition to HGC’s motion to dismiss –
Procedure questions the December 13, 2010 and March 7, 2011Ordersof the Regional Trial Court of Makati, Branch 64 (RTC), in Civil Case admitted that the case is a real action as it affects title to or possession of real property or an interest therein. Having only paid docket fees
No. 92-2311, granting the motion for execution of petitioner, but denying his prayer for the return of his cargo vessel in the condition when the corresponding to an action where the subject matter is incapable of pecuniary estimation, R-II Builders cannot expediently claim that jurisdiction
possession thereof was seized from him. over the case had already attached.
In De Leon v. Court of Appeals, this Court had, of course, ruled that a case for rescission or annulment of contract is not susceptible
Petitioner Ernesto Dy (petitioner) and his wife, Lourdes Dy (Lourdes), were the proprietors of Limchia Enterprises which was engaged of pecuniary estimation although it may eventually result in the recovery of real property. Taking into consideration the allegations and the
in the shipping business. In 1990, Limchia Enterprises, with Lourdes as co-maker, obtained a loan from Orix Metro Leasing and Finance nature of the relief sought in the complaint in the subsequent case of Serrano v. Delica, however, this Court determined the existence of a real
Corporation (respondent) to fund its acquisition of M/V Pilar-I, a cargo vessel. As additional security for the loan, Limchia Enterprises executed action and ordered the payment of the appropriate docket fees for a complaint for cancellation of sale which prayed for both permanent and
the Deed of Chattel Mortgage over M/V Pilar-I. preliminary injunction aimed at the restoration of possession of the land in litigation is a real action. In discounting the apparent conflict in said
rulings, the Court went on to rule as follows in Ruby Shelter Builders and Realty Development Corporation v. Hon. Pablo C, Formaran, to wit:
The Court x x x does not perceive a contradiction between Serrano and the Spouses De Leon. The Court calls attention to the
Due to financial losses suffered when M/V Pilar-I was attacked by pirates, Spouses Dy failed to make the scheduled payments as following statement in Spouses De Leon: "A review of the jurisprudence of this Court indicates that in determining whether an action is one the
required in their promissory note. After receiving several demand letters from respondent, Spouses Dy applied for the restructuring of their loan. subject matter of which is not capable of pecuniary estimation, this Court has adopted the criterion of first ascertaining the nature of the
Meanwhile, Lourdes issued several checks to cover the remainder of their loan but the same were dishonored by the bank, prompting principal action or remedy sought." Necessarily, the determination must be done on a case-to-case basis, depending on the facts and
respondent to institute a criminal complaint for violation of the Bouncing Checks Law. Lourdes appealed to respondent with a new proposal to circumstances of each. What petitioner conveniently ignores is that in Spouses De Leon, the action therein that private respondents instituted
update their outstanding loan obligations. before the RTC was "solely for annulment or rescission" of the contract of sale over a real property.

Consequently, on August 17, 2010, petitioner filed a motion for execution of judgment with the RTC. In the intervening period,
Colorado filed its Manifestation/Motion, dated July 29, 2010, informing the RTC that M/V Pilar-I, which was in its possession, had sustained
severe damage and deterioration and had sunk in its shipyard because of its exposure to the elements. For this reason, it sought permission DAVAO LIGHT VS. COURT OF APPEALS
from the court to cut the sunken vessel into pieces, sell its parts and deposit the proceeds in escrow.In his Comment/Objection, petitioner
insisted that he had the right to require that the vessel be returned to him in the same condition that it had been at the time it was wrongfully
seized by respondent or, should it no longer be possible, that another vessel of the same tonnage, length and beam similar to that of M/V Pilar-I CITATION : G.R. NO. 111685, AUGUST 20, 2001
be delivered. Colorado, however, responded that the vessel had suffered severe damage and deterioration that refloating or restoring it to its
former condition would be futile, impossible and very costly; and should petitioner persist in his demand that the ship be refloated, it should be FACTS : The Davao Light and Power Co., Inc. ("Davao Light") filed a collection suit
done at the expense of the party adjudged by the court to pay the same. against Queensland Hotel ("Queensland") and Teodorico Adarna ("Adarna") with an ex parte application for a writ of preliminary attachment. On
3 May 1989, the trial court issued an Order of Attachment, and the corresponding Writ of Attachment on 11 May 1989. On 12 May 1989, the
ISSUE : Whether petitioner was justified in resorting directly to this Court via a petition for certiorari under Rule 65. summons, a copy of the complaint, and the writ of attachment was served upon Queensland and Adarna. Queensland and Adarna filed a
motion to discharge the attachment on the ground that at the time the Order of Attachment and Writ of Attachment were issued, the trial court
has yet to acquire jurisdiction over the cause of action and over the persons of the defendants.
RULING : Hierarchy of Courts; Direct ResortTo The Supreme Court Justified. This Order of September 19, 1989 was successfully
Petitioner argues that his situation calls for the direct invocation of this Court’s jurisdiction in the interest of justice. challenged by Queensland and Adarna in a special civil action of certiorari instituted by them in the Court of Appeals. The Order was, as
Moreover, as pointed out by the RTC, what is involved is a judgment of the Court which the lower courts cannot modify. aforestated, annulled by the Court of Appeals in its Decision of May 4, 1990.
Hence, petitioner deemed it proper to bring this case immediately to the attention of this Court. Lastly, petitioner claims that
the present case involves a novel issue of law – that is, whether in an action to recover, a defendant in wrongful possession ISSUE : The question is whether or not a writ of preliminary attachment may issue ex
of the subject matter in litigation may be allowed to return the same in a deteriorated condition without any liability. parte against a defendant before acquisition of jurisdiction of the latter's person by service of summons or his voluntary submission to the
Court's authority.
RATIO : Under the principle of hierarchy of courts, direct recourse to this Court is improper because the Supreme Court
is a court of last resort and must remain to be so in order for it to satisfactorily perform its constitutional functions, thereby allowing it to devote RULING : The Court rules that the question must be answered in the affirmative and that
its time and attention to matters within its exclusive jurisdiction and preventing the overcrowding of its docket. 16 Nonetheless, the invocation of consequently, the petition for review will have to be granted.
this Court’s original jurisdiction to issue writs of certiorari has been allowed in certain instances on the ground of special and important reasons The court may validly issue a writ of preliminary injunction prior to the acquisition of jurisdiction over the person of the
clearly stated in the petition, such as,(1) when dictated by the public welfare and the advancement of public policy; (2) when demanded by the defendant. There is an appreciable period of time between the commencement of the action (takes place upon the filing of an initiatory
broader interest of justice; (3) when the challenged orders were patent nullities; or (4) when analogous exceptional and compelling pleading) and the service of summons to the defendant. In the meanwhile, there are a number of actions which the plaintiff or the court may
circumstances called for and justified the immediate and direct handling of the case. 17 validly take, including the application for and grant of the provisional remedy of preliminary attachment. There is nothing in the law which
This case falls under one of the exceptions to the principle of hierarchy of courts. Justice demands that this Court take cognizance of this case prohibits the court from granting the remedy prior to the acquisition of jurisdiction over the person of the defendant. In fact, Rule 57 of the Rules
to put an end to the controversy and resolve the matter which has been dragging on for more than twenty (20) years. Moreover, in light of the of Court allows the granting of a writ of preliminary injunction at the commencement of the suit. In the cases of Toledo v. Burgos and Filinvest
fact that what is involved is a final judgment promulgated by this Court, it is but proper for petitioner to call upon its original jurisdiction and seek Credit Corporation v. Relova, it was held that notice and hearing are not prerequisites to the issuance of a writ of preliminary attachment.
final clarification. Further, in the case of Mindanao Savings & Loan Association, Inc. v. Court of Appeals, it was ruled that giving notice to the defendant would
defeat the purpose of the remedy by affording him or her the opportunity to dispose of his properties before the writ can be issued.
A preliminary attachment may be discharged with the same ease as obtaining it. In any case, the ease of availing the provisional remedy
HOME GUARANTY CORP. VS. R-11 BUILDERS of preliminary attachment is matched by the ease with which it can be remedied by either the posting of a counterbond, or by a showing of its
improper or irregular issuance. The second means of defeating a preliminary attachement, however, may not be availed of if the writ was
CITATION : G.R. NO. 192649, MARCH 9, 2011 issued upon a ground which is at the same time the applicant's cause of action.
Preliminary attachment not binding until jurisdiction over the person of the defendant is acquired. The writ of preliminary attachment,
however, even though validly issued, is not binding upon the defendant until jurisdiction over his person is first acquired.
FACTS : In urging the reversal of the Court's decision, R-II Builders argues that it filed
its complaint with the Manila RTC which is undoubtedly vested with jurisdiction over actions where the subject matter is incapable of pecuniary
estimation; that through no fault of its own, said complaint was raffled to Branch 24, the designated Special Commercial Court (SCC) tasked to NOCUM VS. LUCIO TAN
hear intra-corporate controversies; that despite the determination subsequently made by Branch 24 of the Manila RTC that the case did not CITATION : G.R. N0. 145022, SEPTEMBER 23, 2005
involve an intra-corporate dispute, the Manila RTC did not lose jurisdiction over the same and its Executive Judge correctly directed its re- FACTS : On September 27, 1998, Lucio Tan filed a complaint against
raffling to Branch 22 of the same Court; that the re-raffle and/or amendment of pleadings do not affect a court's jurisdiction which, once reporter Armand Nocum, Capt. Florendo Umali, ALPAP and Inquirer with the Regional Trial Court of Makati, docketed as Civil
Case No. 98-2288, seeking moral and exemplary damages for the alleged malicious and defamatory imputations contained in Jurisdiction is a matter of substantive law; venue, of procedural law; (c) Jurisdiction establishes a relation between the court and the
a news article. subject matter; venue, a relation between plaintiff and defendant, or petitioner and respondent; and, (d) Jurisdiction is fixed by law
and cannot be conferred by the parties; venue may be conferred by the act or agreement of the parties.
INQUIRER and NOCUM filed their joint answer, dated October 27, 1998, wherein they alleged that: (1) the complaint
failed to state a cause of action; (2) the defamatory statements alleged in the complaint were general conclusions without In the case at bar, the additional allegations in the Amended Complaint that the article and the caricature were printed and first published in the
factual premises; (3) the questioned news report constituted fair and true report on the matters of public interest concerning a City of Makati referred only to the question of venue and not jurisdiction. These additional allegations would neither confer jurisdiction on the
public figure and therefore, was privileged in nature; and (4) malice on their part was negated by the publication in the same RTC nor would respondent’s failure to include the same in the original complaint divest the lower court of its jurisdiction over the case.
article of plaintiffs or PALs side of the dispute with the pilots union. Respondent’s failure to allege these allegations gave the lower court the power, upon motion by a party, to dismiss the complaint on the ground
that venue was not properly laid.
ALPAP and UMALI likewise filed their joint answer, dated October 31, 1998, and alleged therein that: (1) the complaint
stated no cause of action; (2) venue was improperly laid; and (3) plaintiff Lucio Tan was not a real party in interest. It appeared In Laquian v. Baltazar, this Court construed the term "jurisdiction" in Article 360 of the Revised Penal Code as referring to the place where
that the complaint failed to state the residence of the complainant at the time of the alleged commission of the offense and the actions for libel shall be filed or "venue."
place where the libelous article was printed and first published.
We note that the amended complaint or amendment to the complaint was not intended to vest jurisdiction to the lower court, where originally it
Thus, the Regional Trial Court of Makati issued an Order dated February 10, 1999, dismissing the complaint without had none. The amendment was merely to establish the proper venue for the action. It is a well-established rule that venue has nothing
prejudice on the ground of improper venue. to do with jurisdiction, except in criminal actions. Assuming that venue were properly laid in the court where the action was
instituted, that would be procedural, not a jurisdictional impediment. In fact, in civil cases, venue may be waived.
Aggrieved by the dismissal of the complaint, respondent Lucio Tan filed an Omnibus Motion dated February 24, 1999,
seeking reconsideration of the dismissal and admission of the amended complaint. In par. 2.01.1 of the amended complaint, it Consequently, by dismissing the case on the ground of improper venue, the lower court had jurisdiction over the case. Apparently, the herein
is alleged that This article was printed and first published in the City of Makati (p. 53, Rollo, CA-G.R. SP No. 55192), and in petitioners recognized this jurisdiction by filing their answers to the complaint, albeit, questioning the propriety of venue, instead of a motion to
par. 2.04.1, that This caricature was printed and first published in the City of Makati (p. 55, id.). dismiss.
The lower court, after having the case dismissed for improper venue, admitted the amended complaint and deemed
set aside the previous order of dismissal. We so hold that dismissal of the complaint by the lower court was proper considering that the complaint, indeed, on its face, failed to allege
neither the residence of the complainant nor the place where the libelous article was printed and first published. Nevertheless, before the
finality of the dismissal, the same may still be amended as in fact the amended complaint was admitted, in view of the court a quo’s jurisdiction,
of which it was never divested. In so doing, the court acted properly and without any grave abuse of discretion.
ISSUE : The question to be resolved is: Did the lower court acquire jurisdiction over the civil case
upon the filing of the original complaint for damages?
SPOUSES JESUS FAJARDO and EMER FAJARDO vs. ANITA R. FLORES, assisted by her husband, BIENVENIDO FLORES
G.R. No. 167891 January 15, 2010
RULING : The Court rule in the affirmative.
It is settled that jurisdiction is conferred by law based on the facts alleged in the complaint since the latter comprises a concise statement
Topic: Doctrine of primary jurisdiction
of the ultimate facts constituting the plaintiff's causes of action. [11] In the case at bar, after examining the original complaint, we find that the RTC
acquired jurisdiction over the case when the case was filed before it. From the allegations thereof, respondents cause of action is for damages
Facts:
arising from libel, the jurisdiction of which is vested with the RTC. Article 360 of the Revised Penal Code provides that it is a Court of First
Leopoldo delos Reyes owned a parcel of land located in Barangay Sumandig in Hacienda Buenavista, San Ildefonso, Bulacan. In 1963, he
Instance.that is specifically designated to try a libel case
allowed petitioner Jesus Fajardo to cultivate said land. The net harvests were divided equally between the two until 1975 when the relationship
was converted to leasehold tenancy. From the time petitioner cultivated the land, he was allowed by Leopoldo delos Reyes to erect a house for
Petitioners are confusing jurisdiction with venue. A former colleague, the Hon. Florenz D. Regalado, differentiated jurisdiction and venue
his family on the stony part of the land, which is the subject of controversy.
as follows: (a) Jurisdiction is the authority to hear and determine a case; venue is the place where the case is to be heard or tried; (b)
Jurisdiction is a matter of substantive law; venue, of procedural law; (c) Jurisdiction establishes a relation between the court and the subject
On January 26, 1988, Leopoldo delos Reyes died. His daughter and sole heir, herein respondent Anita Flores, inherited the property. On June
matter; venue, a relation between plaintiff and defendant, or petitioner and respondent; and, (d) Jurisdiction is fixed by law and cannot be
28, 1991, Anita Flores and Jesus Fajardo executed an agreement, denominated as “KASUNDUAN NG PAGHAHATI NG LUPA AT
conferred by the parties; venue may be conferred by the act or agreement of the parties.
PAGTATALAGA NG DAAN UKOL SA MAGKABILANG PANIG followed by another agreement, “KASUNDUAN SA HATIAN SA LUPA,” executed
on July 10, 1991, wherein the parties agreed to deduct from Lot No. 2351 an area of 10,923 sq m, allotting the same to petitioner. Apparently,
In the case at bar, the additional allegations in the Amended Complaint that the article and the caricature were printed and first
there was a conflict of claims in the interpretation of the Kasunduan between Anita Flores and Jesus Fajardo, which was referred to the DAR,
published in the City of Makati referred only to the question of venue and not jurisdiction. These additional allegations would neither confer
Provincial Agrarian Reform Office, Baliuag, Bulacan. The case was referred to the Department of Agrarian Reform Adjudication Board
jurisdiction on the RTC nor would respondents failure to include the same in the original complaint divest the lower court of its jurisdiction over
(DARAB), Malolos, Bulacan.
the case. Respondents failure to allege these allegations gave the lower court the power, upon motion by a party, to dismiss the complaint on
the ground that venue was not properly laid.
On December 22, 2000, a complaint for ejectment was filed by herein respondent Anita Flores, assisted by her husband Bienvenido Flores,
against petitioners with the Municipal Trial Court (MTC), San Ildefonso, Bulacan. Petitioners filed a Motion to Dismiss, alleging that Lot No.
2351, was agricultural land; that they had been continuously, uninterruptedly, and personally cultivating the same since 1960 up to the present;
that the MTC had no jurisdiction over the case, considering that the dispute between the parties, regarding the Kasunduan, was referred to the
ARMAND NOCUM and THE PHILIPPINE DAILY INQUIRER, INC. vs. LUCIO TAN DARAB; and that the assumption by the DARAB of jurisdiction over the controversy involving the lot in question therefore precluded the MTC
from exercising jurisdiction over the case.
G.R. No. 145022 September 23, 2005
Resolving the Motion to Dismiss, the MTC ruled that, while at first glance, the court did not have jurisdiction over the case, considering that it
Topic: Jurisdiction vs. Venue was admitted that petitioner was allowed to cultivate the land, a closer look at the Kasunduan, however, revealed that what was divided was
only the portion being tilled. By contrast, the subject matter of the complaint was the stony portion where petitioners’ house was erected. Thus,
Facts: the court ruled that it had jurisdiction over the subject matter. On April 25, 2001, the MTC rendered judgment in favor of respondent.
Lucio Tan filed a complaint against reporter Armand Nocum, Capt. Florendo Umali, ALPAP and Inquirer with the Regional Trial Court of Makati,
seeking moral and exemplary damages for the alleged malicious and defamatory imputations contained in a news article. INQUIRER and On appeal, the Regional Trial Court (RTC), Branch 16, Third Judicial Region, Malolos, Bulacan, affirmed the MTC Decision in toto upon a
NOCUM filed their joint answer, dated October 27, 1998. ALPAP and UMALI likewise filed their joint answer and alleged therein that the venue finding that no reversible error was committed by the court a quo in its Decision dated August 29, 2002. On motion for reconsideration,
was improperly laid, among others. It appeared that the complaint failed to state the residence of the complainant at the time of the alleged however, the RTC issued an Order on December 10, 2002, reversing its decision dated August 29, 2002. The RTC found that the issue
commission of the offense and the place where the libelous article was printed and first published. Thus, the Regional Trial Court of Makati involved appeared to be an agrarian dispute, which fell within the contemplation of Republic Act (R.A.) No. 6657, otherwise known as the
dismissed the complaint without prejudice on the ground of improper venue. Comprehensive Agrarian Reform Law of 1988, and thus ordered the dismissal of the case for lack of jurisdiction.

Aggrieved, respondent Lucio Tan filed an Omnibus Motion seeking reconsideration of the dismissal and admission of the amended complaint. A petition for review was then filed by respondents with the CA to annul the Order of the RTC dated December 10, 2002. On October 28, 2004,
In par. 2.01.1 of the amended complaint, it is alleged that "This article was printed and first published in the City of Makati". the CA rendered the assailed decision, which reinstated the MTC decision. It disagreed with the findings of the RTC and ruled that the part of
Lot No. 2351 where petitioners’ house stood was stony and residential in nature, one that may not be made to fall within the ambit of the
RTC admitted the amended complaint and deemed set aside the previous order of dismissal stating that the mistake or deficiency in the operation of Philippine agrarian laws, owing to its non-agriculture character. The subsequent motion for reconsideration was denied; hence, this
original complaint appears now to have been cured in the Amended Complaint. Besides, there is no substantial amendment in the Amended petition.
Complaint which would affect the defendants’ defenses and their Answers as the Amendment is merely formal.
Issue:
Dissatisfied, petitioners appealed the RTC decision to the Court of Appeals. Two petitions for certiorari were filed, one filed by petitioners and Whether or not it is MTC or the DARAB which has jurisdiction over the case.
the other by defendants. The two petitions were consolidated. CA affirmed the decision of the RTC. The motions for reconsideration filed by
petitioners and by defendants Umali and ALPAP were likewise denied in a resolution dated 15 September 2000. Hence, this petition. Ruling:
We agree with the RTC when it clearly pointed out in its Order dated December 10, 2002 that the resolution of this case hinges on the correct
Issue: Whether or not the lower court acquire jurisdiction over the civil case upon the filing of the original complaint for damages. interpretation of the contracts executed by the parties. The issue of who has a better right of possession over the subject land cannot be
determined without resolving first the matter as to whom the subject property was allotted. Thus, this is not simply a case for unlawful detainer,
Ruling: but one that is incapable of pecuniary estimation, definitely beyond the competence of the MTC.
It is settled that jurisdiction is conferred by law based on the facts alleged in the complaint since the latter comprises a concise statement of the
ultimate facts constituting the plaintiff's causes of action. In the case at bar, after examining the original complaint, we find that the RTC More importantly, the controversy involves an agricultural land, which petitioners have continuously and personally cultivated since the 1960s.
acquired jurisdiction over the case when the case was filed before it. From the allegations thereof, respondent’s cause of action is for damages In the Kasunduan, it was admitted that Jesus Fajardo was the tiller of the land. Being agricultural lessees, petitioners have a right to a home lot
arising from libel, the jurisdiction of which is vested with the RTC. Article 360 of the Revised Penal Code provides that it is a Court of First and a right to exclusive possession thereof by virtue of Section 24, R.A. No. 3844 of the Agricultural Land Reform Code. Logically, therefore,
Instance that is specifically designated to try a libel case. the case involves an agrarian dispute, which falls within the contemplation of R.A. No. 6657, or the Comprehensive Agrarian Reform Law.

Petitioners are confusing jurisdiction with venue. A former colleague, the Hon. Florenz D. Regalado, differentiated jurisdiction and venue as An agrarian dispute refers to any controversy relating to tenurial arrangements, whether leasehold, tenancy, stewardship, or otherwise, over
follows: (a) Jurisdiction is the authority to hear and determine a case; venue is the place where the case is to be heard or tried; (b) lands devoted to agriculture, including disputes concerning farmworkers’ associations or representation of persons in negotiating, fixing,
maintaining, changing, or seeking to arrange terms or conditions of such tenurial arrangements. It includes any controversy relating to
compensation of lands acquired under this Act and other terms and conditions of transfer of ownership from landowner to farmworkers, tenants,
and other agrarian reform beneficiaries, whether the disputants stand in the proximate relation of farm operator and beneficiary, landowner and
tenant, or lessor and lessee. It relates to any controversy relating to, inter alia, tenancy over lands devoted to agriculture.

Undeniably, the instant case involves a controversy regarding tenurial arrangements. The contention that the Kasunduans, which allegedly
terminated the tenancy relationship between the parties and, therefore, removed the case from the ambit of R.A. No. 6657, is untenable. There
still exists an agrarian dispute because the controversy involves the home lot of petitioners, an incident arising from the landlord-tenant
relationship.

In the case at bar, petitioners’ claim that the tenancy relationship has been terminated by the Kasulatan is of no moment. As long as the subject
matter of the dispute is the legality of the termination of the relationship, or if the dispute originates from such relationship, the case is
cognizable by the DAR, through the DARAB. The severance of the tenurial arrangement will not render the action beyond the ambit of an
agrarian dispute.

Furthermore, the records disclose that the dispute between the parties, regarding the interpretation of the Kasunduan, was, in fact, raised and
referred to the DAR, which in turn referred the case to the DARAB. In view of the foregoing, we reiterate Hilario v. Prudente, that:
The doctrine of primary jurisdiction precludes the courts from resolving a controversy over which jurisdiction has initially
been lodged with an administrative body of special competence. For agrarian reform cases, jurisdiction is vested in the
Department of Agrarian Reform (DAR); more specifically, in the Department of Agrarian Reform Adjudication Board (DARAB).
TOPIC: ADHERENCE OF JURISDICTION Motion for Partial Reconsideration was filed of the decision alleging loss of jurisdiction on the part of the trial court over the
complaint by virtue of the passage of Executive Order No. 209 (as amended by Executive Order No. 227), otherwise known as
the Family Code of the Philippines which took effect on August 3, 1988. This motion was denied by the lower court in the
JOSE E. ARUEGO VS. COURT OF APPEALS Order, dated January 14, 1993.

CITATION: G.R. NO. 112193 MARCH 13, 1996 Petitioners interposed an appeal but the lower court refused to give it due course on the ground that it was filed out of time.

CASE: On March 7, 1983, a Complaint for Compulsory Recognition and Enforcement of Successional Rights was filed before Branch 30 of the CA
Regional Trial Court of Manila by the minors, private respondent Antonia F. Aruego and her alleged sister Evelyn F. Aruego, represented by
their mother and natural guardian, Luz M. Fabian. Named defendants therein were Jose E. Aruego, Jr. and the five (5) minor children of the Petitioners filed a Petition for Prohibition and Certiorari with prayer for a Writ of Preliminary Injunction.
deceased Gloria A. Torres, represented by their father and natural guardian, Justo P. Torres, Jr.
Decision: CA dismissed the petition for lack of merit.
FACTS: The late Jose M. Aruego, Sr., a married man, had an amorous relationship with Luz M. Fabian in 1959 until his death on March 30,
1982. Out of this relationship were born Antonia F. Aruego and Evelyn F. Aruego on October 5, 1962 and September 3, 1963, respectively. A motion for reconsideration was filed. It was denied by CA in a minute resolution dated Oct 13, 1993.

Jose M. Aruego, Sr acknowledged and recognized the herein private respondents as his children verbally among respondent’s and their ISSUE: WON RTC lost jurisdiction of the case on the ground of prescription.
mother's family friends, as well as by myriad different paternal ways, including but not limited to the following:
RULING:
1. Regular support and educational expenses;
2. Allowance to use his surname; NO.
3. Payment of maternal bills;
4. Payment of baptismal expenses and attendance therein; As ruled in Tayag vs. Court of Appeals:
5. Taking them to restaurants and department stores on occasions of family rejoicing;
6. Attendance to school problems of plaintiffs; Under the circumstances obtaining in the case at bar, we hold that the right of action of the minor child has been vested by the
7. Calling and allowing plaintiffs to his office every now and then; filing of the complaint in court under the regime of the Civil Code and prior to the effectivity of the Family Code. We herein
8. Introducing them as such children to family friends. adopt our ruling in the recent case of Republic of the Philippines vs. Court of Appeals, et. al. 7 where we held that the fact of
filing of the petition already vested in the petitioner her right to file it and to have the same proceed to final adjudication in
The respondents are thus, in continuous possession of the status of (illegitimate) children of the deceased Jose M. Aruego who showered accordance with the law in force at the time, and such right can no longer be prejudiced or impaired by the enactment of a new
them, with the continuous and clear manifestations of paternal care and affection as above outlined. law.

Basis of Respondent’s action for compulsory recognition as an illegitimate child: xxx xxx xxx

Book I, Title VIII of the Civil Code on PERSONS, specifically Article 285 thereof, which state the manner by Accordingly, Article 175 of the Family Code finds no proper application to the instant case since it will ineluctably affect
which illegitimate children may prove their filiation, to wit: adversely a right of private respondent and, consequentially, of the minor child she represents, both of which have been vested
with the filing of the complaint in court. The trial court is, therefore, correct in applying the provisions of Article 285 of the Civil
Art. 285. The action for the recognition of natural children may be brought only during the lifetime of the Code and in holding that private respondent's cause of action has not yet prescribed.
presumed parents, except in the following cases:
Tayag applies four-square with the case at bench. The action brought by private respondent Antonia Aruego for compulsory recognition and
1. If the father or mother died during the minority of the child, in which case the latter may file the action enforcement of successional rights which was filed prior to the advent of the Family Code, must be governed by Article 285 of the Civil Code
before the expiration of four years from the attainment of his majority; . . . . and not by Article 175, paragraph 2 of the Family Code. The present law cannot be given retroactive effect insofar as the instant case is
concerned, as its application will prejudice the vested right of private respondent to have her case decided under Article 285 of the
Civil Code. The right was vested to her by the fact that she filed her action under the regime of the Civil Code. Prescinding from this,
the conclusion then ought to be that the action was not yet barred, notwithstanding the fact that it was brought when the putative father was
already deceased, since private respondent was then still a minor when it was filed, an exception to the general rule provided under Article 285
of the Civil Code. Hence, the trial court, which acquired jurisdiction over the case by the filing of the complaint, never lost jurisdiction
Petitioner’s argument: over the same despite the passage of E.O. No. 209, also known as the Family Code of the Philippines.

With the advent of the New Family Code on August 3, 1988, the trial court lost jurisdiction over the complaint of private respondent on the The jurisdiction of a court, whether in criminal or civil cases, once attached cannot be ousted by subsequent happenings or events, although of
ground of prescription, considering that under Article 175, paragraph 2, in relation to Article 172 of the New Family Code, it is provided that an a character which would have prevented jurisdiction from attaching in the first instance, and it retains jurisdiction until it finally disposes of the
action for compulsory recognition of illegitimate filiation, if based on the "open and continuous possession of the status of an illegitimate child," case.
must be brought during the lifetime of the alleged parent without any exception, otherwise the action will be barred by prescription.

Basis of Petitioners’ argument:


TOPIC: ADHERENCE OF JURISDICTION
Art. 172. The filiation of legitimate children is established by any of the following:

1. The record of birth appearing in the civil register or a final judgment; or LUCIA BARRAMEDA VS. RURAL BANK OF CANAMAN
2. An admission of legitimate filiation in a public document or a private handwritten instrument and signed by the parent
concerned. CITATION: G.R. NO. 176260 NOVEMBER 24, 2010

In the absence of the foregoing evidence, the legitimate filiation shall be proved by: CASE originally filed: On March 17, 2000, petitioner Lucia Barrameda Vda. DeBallesteros filed a complaint for Annulment of Deed of
Extrajudicial Partition, Deed of Mortgage and Damages with prayer for Preliminary Injunction against her children, Roy, Rito, Amy, Arabel, Rico,
1. The open and continuous possession of the status of a legitimate child; or Abe, Ponce Rex and Adden, all surnamed Ballesteros, and the Rural Bank of Canaman, Inc., Baao Branch (RBCI) before the RTC-Irig.
2. Any other means allowed by the Rules of Court and special laws.

Art. 175. Illegitimate children may establish their illegitimate filiation in the same way and on the same evidence as legitimate FACTS:
children.
Lucia’s deceased husband, Eugenio, left two (2) parcels of land located in San Nicolas, Baao, Camarines Sur, each with an area of 357 square
The action must be brought within the same period specified in Article 173 [during the lifetime of the child], except when the meters. On March 6, 1995, without her knowledge and consent, her children executed a deed of extrajudicial partition and waiver of the estate
action is based on the second paragraph of Article 172, in which case the action may be brought during the lifetime of the of her husband wherein all the heirs, including Lucia, agreed to allot the two parcels to Rico Ballesteros. Still, without her knowledge and
alleged parent. consent, Rico mortgaged Parcel B of the estate in favor of RBCI which mortgage was being foreclosed for failure to settle the loan secured by
the lot; and that Lucia was occupying Parcel B and had no other place to live.
Petitioners point out that, since the complaint of private respondent and her alleged sister was filed on March 7, 1983, or almost one (1) year
after the death of their presumed father on March 30, 1982, the action has clearly prescribed under the new rule as provided in the Family She prayed that the deed of extrajudicial partition and waiver, and the subsequent mortgage in favor of RBCI be declared null and void having
Code. Petitioners, further, maintain that even if the action was filed prior to the effectivity of the Family Code, this new law must be applied to been executed without her knowledge and consent. She also prayed for damages.
the instant case pursuant to Article 256 of the Family Code which provides:
RBCI claimed that in 1979, Lucia sold one of the two parcels to Rico which represented her share in the estate of her husband. The
This Code shall, have retroactive effect insofar as it does not prejudice or impair vested of acquired rights in accordance with extrajudicial partition, waiver and mortgage were all executed with the knowledge and consent of Lucia although she was not able to sign the
the Civil Code or other laws. document. RBCI further claimed that Parcel B had already been foreclosed way back in 1999 which fact was known to Lucia through the
auctioning notary public.

RTC Attorney’s fees were pleaded as counterclaim.

Decision: The court declared Antonia Aruego as illegitimate daughter of Jose Aruego and Luz Fabian. The court further During the pre-trial, RBCIs counsel filed a motion to withdraw after being informed that Philippine Deposit Insurance Corporation (PDIC) would
declared that Evelyn Fabian is not an illegitimate daughter of Jose Aruego with Luz Fabian handle the case as RBCI had already been closed and placed under the receivership of the PDIC. Consequently, on February 4, 2002, the
lawyers of PDIC took over the case of RBCI.
On November 29, 1997, the Iligan City RTC rendered a Decision, holding the MSU liable for damages amounting to P2,726,189.90. The Court
On May 9, 2003, RBCI, through PDIC, filed a motion to dismiss on the ground that the RTC-Iriga has no jurisdiction over the subject matter of of Appeals affirmed the Iligan City RTC decision and the CA decision subsequently lapsed to finality. On January 19, 2009, Entry of Judgment
the action. RBCI stated that pursuant to Section 30, Republic Act No. 7653, otherwise known as the New Central Bank Act, the RTC-Makati, was made.
already constituted itself, per its Order dated August 10, 2001, as the liquidation court to assist PDIC in undertaking the liquidation of RBCI.
Thus, the subject matter of Civil Case No. IR-3128 fell within the exclusive jurisdiction of such liquidation court. On March 10, 2009, the Iligan City RTC issued a writ of execution. The MSU, however, failed to comply with the writ; thus, on March 24, 2009,
Sheriff Gerard Peter Gaje served a Notice of Garnishment on the MSUs depository bank, the Land Bank of the Philippines ( LBP), Marawi City
Lucia opposed the motion. Branch.

Argument: Lucia contends that the RTC-Iriga is vested with jurisdiction over Civil Case No. 3128, the constitution of the Office of the Solicitor General opposed the motion for execution. The Iligan City RTC denied the opposition. MSU responded to the denial by
liquidation court notwithstanding. She contends that: filing on April 1, 2009 a petition with the Marawi City RTC, for prohibition and mandamus with an application for the issuance of a temporary
restraining order (TRO) and/or preliminary injunction against the LBP and Sheriff Gaje. The petition of MSU was raffled to the RTC, Marawi
Since the RTC-Iriga has already obtained jurisdiction over the case it should continue exercising such jurisdiction until the final City, Branch 8, presided by respondent Judge.
termination of the case. The jurisdiction of a court once attached cannot be ousted by subsequent happenings or events,
although of a character which would have prevented jurisdiction from attaching in the first instance, and the Court retains The respondent Judge set the hearing for the application for the issuance of a TRO on April 8, 2009. After which he issued a TRO restraining
jurisdiction until it finally disposes of the case (Aruego Jr. v. Court of Appeals, 254 SCRA 711). Sheriff Gaje from garnishing P2,726,189.90 from MSUs LBP-Marawi City Branch account.

When a court has already obtained and is exercising jurisdiction over a controversy, its jurisdiction to proceed to final On April 17, 2009, the respondent Judge conducted a hearing on the application for the issuance of a writ of preliminary injunction. Thereafter,
determination of the case is not affected by a new legislation transferring jurisdiction over such proceedings to another tribunal. he required MSU to file a memorandum in support of its application for the issuance of a writ of preliminary injunction. On April 21, 2009, Sheriff
(Alindao v. Joson, 264 SCRA 211). Once jurisdiction is vested, the same is retained up to the end of the litigation ( Bernate v. Gaje moved to dismiss the case on the ground of lack of jurisdiction. The respondent Judge thereafter granted the motion and dismissed the
Court of Appeals, 263 SCRA 323). case.

RTC On May 8, 2009, complainant Atty. Tomas Ong Cabili, counsel of the private plaintiffs in Civil Case No. 06-2954, filed the instant complaint
charging the respondent Judge with Gross Ignorance of the Law, Grave Abuse of Authority, Abuse of Discretion, and/or Grave Misconduct
Decision: Granted the Motion to dismiss. Prejudicial to the Interest of the Judicial Service for interfering with the order of a co-equal court, Branch 6 of the Iligan City RTC, by issuing the
TRO to enjoin Sheriff Gaje from garnishing P2,726,189.90 from MSUs LBP-Marawi City Branch account.
Basis: Pronouncement of the Supreme Court in Ong v. C.A. 253 SCRA 105 and in the case of Hernandez v. Rural Bank of
Lucena, Inc., G.R. No. L-29791 dated January 10, 1978, wherein it was held that the liquidation court shall have jurisdiction to The respondent Judge denied that he interfered with the order of Branch 6 of the Iligan City RTC. He explained that he merely gave the parties
adjudicate all claims against the bank whether they be against assets of the insolvent bank, for Specific Performance, Breach the opportunity to be heard and eventually dismissed the petition for lack of jurisdiction.
of Contract, Damages or whatever.

CA Office of the Court Administrator (OCA)


Decision: Found the respondent Judge guilty of gross ignorance of the law for violating the elementary rule of non-interference
Lucia appealed the RTC ruling to the CA on the ground that the RTC-Iriga erred in dismissing the case because it had with the proceedings of a court of co-equal jurisdiction. It recommended a fine of P40,000.00, noting that this is the respondent
jurisdiction over Civil Case No. IR-3128 under the rule on adherence of jurisdiction. Judges second offense.

CA ordered the consolidation of the Civil Case No. IR-3128 and the liquidation case pending before RTC-Makat to prevent
confusion, to avoid multiplicity of suits and to save unnecessary cost and expense. ISSUE: WON respondent judge violates the rule on non-interference with the proceedings of a court of co-equal jurisdiction

Lucia filed a motion for reconsideration but it was denied by the CA in its Resolution dated December 14, 2006. RULING:

ISSUE: WON RTC IS VESTED WITH JURISDICTION TO CONTINUE TRYING AND ULTIMATELY DECIDE CIVIL CASE NO. IR-3128 YES.

The doctrine of judicial stability or non-interference in the regular orders or judgments of a co-equal court is an elementary principle in the
RULING: administration of justice: no court can interfere by injunction with the judgments or orders of another court of concurrent jurisdiction having the
power to grant the relief sought by the injunction. The rationale for the rule is founded on the concept of jurisdiction: a court that acquires
NO. jurisdiction over the case and renders judgment therein has jurisdiction over its judgment, to the exclusion of all other coordinate courts, for its
execution and over all its incidents, and to control, in furtherance of justice, the conduct of ministerial officers acting in connection with this
The Court recognizes the doctrine on adherence of jurisdiction. However, such principle is not without exceptions. It is well to quote the ruling of judgment.
the CA on this matter, thus:
Thus, we have repeatedly held that a case where an execution order has been issued is considered as still pending, so that all the proceedings
This Court is not unmindful nor unaware of the doctrine on the adherence of jurisdiction. However, the rule on adherence of on the execution are still proceedings in the suit. A court which issued a writ of execution has the inherent power, for the advancement of
jurisdiction is not absolute and has exceptions. One of the exceptions is that when the change in jurisdiction is curative in justice, to correct errors of its ministerial officers and to control its own processes. To hold otherwise would be to divide the jurisdiction of the
character. appropriate forum in the resolution of incidents arising in execution proceedings. Splitting of jurisdiction is obnoxious to the orderly
administration of justice.
For sure, Section 30, R.A. 7653 is curative in character when it declared that the liquidation court shall have jurisdiction in the
same proceedings to assist in the adjudication of the disputed claims against the Bank. The interpretation of this Section Jurisprudence shows that a violation of this rule warrants the imposition of administrative sanctions.
(formerly Section 29, R.A. 265) becomes more obvious in the light of its intent. In Manalo v. Court of Appeals (366 SCRA 752,
[2001]), the Supreme Court says: In the present case, the respondent Judge clearly ignored the principle of judicial stability by issuing a TRO to temporarily restrain Sheriff Gaje
from enforcing the writ of execution issued by a co-equal court, Branch 6 of the Iligan City RTC, and from pursuing the garnishment of the
xxx The requirement that all claims against the bank be pursued in the liquidation proceedings filed by the Central Bank is amount of P2,726,189.90 from MSUs account with the LBP, Marawi City Branch. The respondent Judge was aware that he was acting on
intended to prevent multiplicity of actions against the insolvent bank and designed to establish due process and orderliness in matters pertaining to the execution phase of a final decision of a co-equal and coordinate court since he even quoted MSUs allegations in his
the liquidation of the bank, to obviate the proliferation of litigations and to avoid injustice and arbitrariness. The lawmaking body April 8, 2009 Order
contemplated that for convenience, only one court, if possible, should pass upon the claims against the insolvent bank and that
the liquidation court should assist the Superintendents of Banks and regulate his operations. The respondent Judge should have refrained from acting on the petition because Branch 6 of the Iligan City RTC retains jurisdiction to rule on
any question on the enforcement of the writ of execution. Section 16, Rule 39 of the Rules of Court ( terceria), cited in the course of the Courts
deliberations, finds no application to this case since this provision applies to claims made by a third person, other than the judgment obligor or
It is clear, therefore, that the liquidation court has jurisdiction over all claims, including that of Lucia against the insolvent bank. As declared in his agent; a third-party claimant of a property under execution may file a claim with another court which, in the exercise of its own jurisdiction,
Miranda v. Philippine Deposit Insurance Corporation, regular courts do not have jurisdiction over actions filed by claimants against an insolvent may issue a temporary restraining order. In this case, the petition for injunction before the respondent Judge was filed by MSU itself, the
bank, unless there is a clear showing that the action taken by the BSP, through the Monetary Board, in the closure of financial institutions was judgment obligor. If Sheriff Gaje committed any irregularity or exceeded his authority in the enforcement of the writ, the proper recourse for
in excess of jurisdiction, or with grave abuse of discretion. The same is not obtaining in this present case. MSU was to file a motion with, or an application for relief from, the same court which issued the decision, not from any other court, or to elevate
the matter to the CA on a petition for certiorari. In this case, MSU filed the proper motion with the Iligan City RTC (the issuing court), but, upon
denial, proceeded to seek recourse through another co-equal court presided over by the respondent Judge.
TOPIC: DOCTRINE OF NON-INTERFERENCE OR JUDICIAL STABILITY
It is not a viable legal position to claim that a TRO against a writ of execution is issued against an erring sheriff, not against the issuing Judge. A
TRO enjoining the enforceability of a writ addresses the writ itself, not merely the executing sheriff. The duty of a sheriff in enforcing writs is
ATTY. TOMAS ONG CABILI VS. JUDGE RASAD G. BALINDONG ministerial and not discretionary.

CITATION: A.M. NO. RTJ-10-2225 SEPTEMBER 6, 2011 As already mentioned above, the appropriate action is to assail the implementation of the writ before the issuing court in whose behalf the
sheriff acts, and, upon failure, to seek redress through a higher judicial body. Significantly, MSU did file its opposition before the issuing court
CASE: Administrative complaint against respondent Acting Presiding Judge Rasad G. Balindong of the Regional Trial Court of Marawi City, Iligan City RTC which denied this opposition.
Branch 8, for Gross Ignorance of the Law, Grave Abuse of Authority, Abuse of Discretion, and/or Grave Misconduct Prejudicial to the Interest of
the Judicial Service. That the respondent Judge subsequently rectified his error by eventually dismissing the petition before him for lack of jurisdiction is not a
defense that the respondent Judge can use. His lack of familiarity with the rules in interfering with the acts of a co-equal court undermines
FACTS: Civil Case No. 06-2954 is an action for damages in Branch 6 of the Iligan City RTC against the Mindanao State University, et al., public confidence in the judiciary through his demonstrated incompetence. In this case, he impressed upon the Iligan public that the kind of
arising from a vehicular accident that caused the death of Jesus Ledesma and physical injuries to several others. interference he exhibited can be done, even if only temporarily, i.e., that an official act of the Iligan City RTC can be thwarted by going to the
Marawi City RTC although they are co-equal courts. That the complaining lawyer, Atty. Tomas Ong Cabili, subsequently reversed course and
manifested that the respondent Judge is basically a good Judge, and should only be reprimanded, cannot affect the respondent Judges liability.
This liability and the commensurate penalty do not depend on the complainant’s personal opinion but on the facts he alleged and proved, and ISSUE: WON Judges Aleonar and Salas may take cognizance of the actions for damagesagainst Judge Villamor for allegedly having rendered
on the applicable law and jurisprudence. When the law is sufficiently basic, a judge owes it to his office to know and to simply apply it. Anything an unjust order of direct contempt against Carlos and Attorney Guerrero which this Court subsequently annulled.
less would be constitutive of gross ignorance of the law.
RULING:
TOPIC: DOCTRINE OF NON-INTERFERENCE OR JUDICIAL STABILITY NO.

No Regional Trial Court can pass upon and scrutinize, and much less declare as unjust a judgment of another Regional Trial Court and
HON. JUDGE ADRIANO R. VILLAMOR VS. HON. JUDGE BERNARDO LL. SALAS sentence the judge thereof liable for damages without running afoul with the principle that only the higher appellate courts, namely, the Court of
Appeals and the Supreme Court, are vested with authority to review and correct errors of the trial courts.
CITATION: G.R. NO. 101041 NOVEMBER 13, 1991
To allow respondent Judges Aleonar and Salas to proceed with the trial of the actions for damages against the petitioner, a co-equal judge of a
FACTS: In 1977, Civil Case No. B-398 (Gloria Naval vs. George Carlos) for recovery of ownership of a parcel of coconut land was filed and co-equal court, would in effect permit a court to review and interfere with the judgment of a co-equal court over which it has no appellate
subsequently raffled to the sala of the petitioner, Judge Adriano Villamor. While the civil case was pending there, respondent Carlos filed jurisdiction or power of review. The various branches of a Court of First Instance (now the Regional Trial Court) being co-equal, may not
Criminal Cases Nos. N-989, N-990, N-991, N-992 and N-993 for qualified theft against Gloria Naval and her helpers. The criminal cases were interfere with each other's cases, judgments and orders.
also assigned to the sala of Judge Villamor.
It is only after the Appellate Court, in a final judgment, has found that a trial judge's errors were committed deliberately and in bad faith may a
Due to the pendency of Civil Case No. B-398, the criminal cases were temporarily archived. After trial in Civil Case No. B-398, a decision was charge of knowingly rendering an unjust decision be levelled against the latter.
rendered in favor of Naval who was declared the lawful owner and possessor of the disputed land. Carlos was ordered to vacate the land.

Thereafter, respondent Carlos, through counsel, moved to activate the archived criminal cases. Having declared Naval the lawful owner and TOPIC: LACK OF JURISDICTION – RAISED AT EARLIEST POSSIBLE OPPORTUNITY
possessor of the contested land in Civil Case No. B-398, Judge Villamor dismissed the criminal cases against her and her co-accused.

Judge Villamor likewise granted execution pending appeal of his decision in Civil Case No. B-398. This order was challenged by Carlos in the HEIRS OF CESAR MARASIGAN VS. APOLONIO MARASIGAN, ET AL.
Court of Appeals and in this Court, both without success.
CITATION: G.R. NO. 156078 MARCH 14, 2008
Administrative case
FACTS: Alicia was survived by her siblings: Cesar, Apolonio, Lilia, and Benito; Marissa, a sister-in-law; and the children of her brothers who
Carlos filed an administrative case, A.M. No. RTJ-87-105, against Judge Villamor, charging him with having issued illegal orders and an unjust predeceased her: Francisco, Horacio, and Octavio. She died intestate and without issue on 21 January 1995.
decision in Civil Case No. B-398.
On December 17, 1997, a Complaint for Judicial Partition of the Estate of Alicia Marasigan was filed before the RTC by several of her heirs and
Decision: SC, in an En Banc resolution, summarily dismissed the administrative case on November 21, 1988. private respondents herein, namely, Apolonio, Lilia, Octavio, Jr., Horacio, Benito, Jr., and Marissa, against Cesar, docketed as Special Civil
Action No. P-77-97.
Civil Action for Damages
According to private respondents, Alicia owned in common with her siblings 13 parcels of land called Hacienda Sta. Rita in Pili and Minalabac,
Civil Case No. CEB-6478 Camarines Sur, with an aggregate area of 4,960,963 square meters or 496 hectares.

Cause of action: For knowingly rendering an unjust judgment when he dismissed the five (5) criminal cases against Naval, et Alicia left behind her 2/21 shares in the 13 parcels of land.
al.
In answer to the private respondents Complaint, Cesar enumerated Alicia’s several other properties and assets which he also wanted included
The summons in Civil Case No. CEB-6478 was served upon Judge Villamor on December 10,1987. in the action for partition.

The next day, instead of answering the complaint, Judge Villamor issued in Criminal Cases Nos. N-0989 to Cesar’s request for inclusion was contested by private respondents on the ground that the properties he enumerated had already been
0993 an order of direct contempt against Carlos and his lawyer. Attorney Antonio T. Guerrero, "for degrading previously partitioned and distributed to the appropriate parties.
the respect and dignity of the court through the use of derogatory and contemptous language before the court,"
and sentenced each of them to suffer the penalty of imprisonment for five (5) days and to pay a fine of P500. RTC
Decided in favor of private respondents and issued an Order of Partition of the Estate of Alicia Marasigan
Carlos immediately filed in SC a petition for certiorari with a prayer for the issuance of a writ of preliminary
injunction against the Judge. As the parties could not agree on how they shall physically partition among themselves Alicias estate, private respondents filed a Motion to
Appoint Commissioners following the procedure outlined in Sections 4, 5, 6 and 7 of Rule 69 of the Rules of Court.
Decision: SC promptly restrained Judge Villamor from enforcing his Order of Contempt against Carlos and
Attorney Guerrero. The RTC granted the Motion and appointed Myrna V. Badiong, Assistant Provincial Assessor of Camarines Sur, as Chairman of the Board of
Commissioners.

Judge Villamor filed a motion to dismiss the complaint for lack of jurisdiction. The trial court granted the motion. The order of Private respondents nominated Sandie B. Dacara as the second commissioner. Cesar failed to nominate a third commissioner despite due
dismissal was affirmed by the Court of Appeals. Carlos appealed to this Court which also denied the petition. notice. Upon lapse of the period given, only two commissioners were appointed

Unfazed by these setbacks, Carlos and his counsel, Attorney Antonio Guerrero, filed separate complaints for damages against Judge Villamor Cesar opposed the foregoing findings and prayed for the disapproval of the Commissioners Report.
for knowingly rendering an unjust order of contempt.
Petitioner’s argument:
The estate is not indivisible just because of the different locations and conditions of the parcels of land constituting the same.
Civil Case No. CEB-8802 Section 5, Rule 69 of the Rules of Court can only be availed of if the partition or division of the real properties involved would
be prejudicial to the interest of any of the parties. He asserted that despite the segregation of his share, the remaining parcels
Atty Guerrero filed a complaint for damages was raffled to Branch 21, Regional Trial Court, Cebu City, presided over by Judge of land would still be serviceable for the planting of rice, corn, and sugarcane, thus evidencing that no prejudice would be
Peary G. Aleonar. Carlos' complaint for damages was docketed as Civil Case No. CEB-8823 and raffled to Branch 8, Regional caused to the interests of his co-heirs.
Trial Court of Cebu City presided over by Judge Bernardo LL. Salas.
Respondents’ answer:
Judge Villamor filed a motion to dismiss but it was denied. The physical division is impossible because Alicia’s estate is equivalent to 2/21 shares in Hacienda Sta. Rita, which is
composed of 13 parcels under different titles and tax declarations, situated in different barangays and municipalities, and
1st SC Case: Petition for certiorari and prohibition with restraining order (G.R. No. 101296) covers an area of 496 hectares.

Decision: SC issued a TRO against Judge Aleonar to stop him from proceeding in Civil Case No. CEB8802
RTC Decision: Issued an Order dated June 22, 2001 approving in toto the recommendations embodied in the Commissioners Report,
On May 20, 1991, a Manifestation was filed by Judge Villamor praying Judge Salas to dismiss Civil Case No. CEB- 8823 but particularly, the recommendation that the property be assigned to one of the heirs at P700,000.00 per hectare or a total amount of
the motion was denied by respondent Judge on July 2, 1991. P4,724,726.00, after finding the same to be in accordance with the Rules of Court and the New Civil Code.

2nd SC Case: Petition for certiorari and prohibition with restraining order (G.R. No. 101041) Cesar filed a Motion for Reconsideration which was denied by RTC for lack of merit.

Court Resolution: CA

1. Temporarily restraining Judge Salas from further proceeding in Civil Case No. CEB-8823; (Cesar died on 25 October 2001. He was substituted by his heirs and herein petitioners, namely, Luz Regina, Cesar, Jr., Benito, Santiago,
Renato, Jose, Geraldo, Orlando, Peter, Paul, Mauricio, Rommel, Michael, Gabriel, and Maria Luz, all surnamed Marasigan.)
2. Granting the petitioner's prayer that this case be consolidated with G.R. No. 101296 (pp. 37-39, Rollo of G.R.
Action: Petition for Certiorari and Prohibition under Rule 65 of the Rules of Court
No. 101041).
CA decision: Dismissed petition and ruled that the RTC acted within its authority in issuing the Order of June 22, 2001. The Court of Appeals
found that petitioners failed to discharge the burden of proving that the proceedings before the Board of Commissioners were unfair and SERAFIN TIJAM VS. MAGDALENO SIBONGHANOY
prejudicial. It likewise found that the petitioners were not denied due process considering that they were afforded the opportunity to be heard
during the hearing for approval of the Commissioners Report on January 18, 2001. According to the appellate court, whether or not the physical CITATION: G.R. NO. L-21450 APRIL 15, 1968
division of the estate will cause prejudice to the interests of the parties is an issue addressed to the discretion of the Commissioners.
FACTS: On July 19, 1948, spouses Serafin Tijam and Felicitas Tagalog commenced Civil Case No. R-660 in the Court of First Instance of
A motion for Reconsideration was filed by the petitioners but the same was denied by CA. Cebu against the spouses Magdaleno Sibonghanoy and Lucia Baguio to recover from them the sum of P1,908.00, with legal interest thereon
from the date of the filing of the complaint until the whole obligation is paid, plus costs.
Petitioners filed a petition for review in SC under Rule 45 of the Revised Rules of Court. Pending resolution of the instant Petition by this Court,
the RTC granted private respondents Urgent Motion for Execution on December 26, 2002. The RTC ordered the sale of petitioners 1/7 pro- As prayed for in the complaint, a writ of attachment was issued by the court against defendants' properties, but the same was soon dissolved
indiviso share in Alicia’s estate upon the urgent motion of private respondents dated September 27, 2002 for the partial execution of the upon the filing of a counter-bond by defendants and the Manila Surety and Fidelity Co., Inc. hereinafter referred to as the Surety, on the 31st of
judgment of the Court approving the Commissioners report pending certiorari. the same month.

Petitioners share in Alicia’s estate was sold in a public auction on February 26, 2003. Based on the Commissioners Report on the Auction Sale, After being duly served with summons the defendants filed their answer in which, after making some admissions and denials of the material
there were two bidders, Apolonio Marasigan and Amado Lazaro. Apolonio, with a bid of P701,000.00 per hectare, won over Amado Lazaro, averments of the complaint, they interposed a counterclaim. This counterclaim was answered by the plaintiffs.
whose bid was P700,000.00 per hectare. Petitioners 1/7 share as Cesars heirs in Alicias estate was sold in the public auction for
P3,777,689.00. This amount is lower than the P4,724,726.30 price of the 1/7 share in Alicia’s estate as earlier determined by the RTC
Commissioners due allegedly to the acquisition by the Department of Agrarian Reform (DAR) of a portion of Hacienda Sta. Rita located in
Minilabac, Camarines Sur which was placed under Republic Act No. 6657, or the Comprehensive Agrarian Reform Law, with 100.00 hectares Decision: Rendered judgment in favor of the petitioner and issued a writ of execution against the defendants.
thereof compulsorily acquired.
The writ having been returned unsatisfied, the plaintiffs moved for the issuance of a writ of execution against the Surety's bond,
Petitioners filed with the RTC a Motion to Declare Failure of Bidding and to Annul Public Auction Sale on March 24, 2003.
Surety’s Action: filed a written opposition upon two grounds, namely:
On May 5, 2003, however, the RTC released an Omnibus Order ruling, among other things, that the objection of petitioners as to the difference
of the value of their 1/7 share as determined by the Commissioners vis--vis the winning bid was no longer an issue since Apolonio Marasigan 1. Failure to prosecute
indicated his willingness to pay for the deficiency. 2. Absence of a demand upon the Surety for the payment of the amount due under the judgment.

Following the public auction and sale of their 1/7 share in the property, petitioners filed a Notice of Appeal with the RTC indicating that they
were appealing the May 5, 2003 Omnibus Order of the RTC to the Court of Appeals. Thereafter, petitioners filed a Record on Appeal pursuant Surety’s Prayer: Prayed the Court not only to deny the motion for execution against its counter-bond but also the following affirmative
to Section 3, Rule 41 of the Rules of Court, praying that it be approved and transmitted to the Court of Appeals. relief : "to relieve the herein bonding company of its liability, if any, under the bond in question"

The RTC issued an Order denying due course to petitioners Notice of Appeal on the ground that the proper remedy is not appeal, but certiorari. Court’s Decision: The Court denied this motion on the ground solely that no previous demand had been made on the Surety for the
Petitioners then filed another Petition before the Court of Appeals for Certiorari and Mandamus, praying that the RTC be directed to approve satisfaction of the judgment.
their Notice of Appeal and Record on Appeal, and to forward the same to the appellate court.
Thereafter the necessary demand was made, and upon failure of the Surety to satisfy the judgment, the plaintiffs filed a second motion for
In a Resolution dated October 10, 2003, the Court of Appeals dismissed CA-G.R. SP No. 78912 outright on the ground that the verification and execution against the counterbond. On the date set for the hearing thereon, the Court, upon motion of the Surety's counsel, granted the latter a
certificate of non-forum shopping of the petition was signed by only Cesar Marasigan, Jr., without any accompanying document to prove his period of five days within which to answer the motion. Upon its failure to file such answer, the Court granted the motion for execution and the
authority to sign on behalf of the other petitioners. Petitioners filed a Motion for Reconsideration but it was denied by the Court of Appeals in a corresponding writ was issued.
Resolution dated July 12, 2004.
Surety’s Action: moved to quash the writ on the ground that the same was issued without the required summary hearing
provided for in Section 17 of Rule 59 of the Rules of Court.
ISSUE: WON the Court of Appeals erred in affirming in toto the RTC Order adopting the Commissioners’ recommendation on the manner of
partition of the estate of Alicia Marasigan Court’s Decision: Denied the Motion

CA

RULING: Surety appealed to the CA from such order of denial and from the one denying its motion for reconsideration

YES. CA Decision: Affirmed the orders appealed from.

First, it bears to point out that Cesar, petitioner’s predecessor, did not file any motion to dismiss, and his answer before the RTC did not bear Surety filed a Motion to Dismiss alleging substantially that appellees action was filed in the Court of First Instance of Cebu on July 19, 1948 for
the defenses/objections of lack of jurisdiction or cause of action on these grounds; consequently, these must be considered waived. The the recovery of the sum of P1,908.00 only; that a month before that date Republic Act No. 296, otherwise known as the Judiciary Act of 1948,
exception that the court may still dismiss a case for lack of jurisdiction over the subject matter, although the same is not pleaded, but is had already become effective, Section 88 of which placed within the original exclusive jurisdiction of inferior courts all civil actions where the
apparent in the pleadings or evidence on record, does not find application to the present petition. Second, petitioner’s arguments on the lack of value of the subject-matter or the amount of the demand does not exceed P2,000.00, exclusive of interest and costs. The Court of First
jurisdiction of the RTC over the case more appropriately pertain to venue, rather than jurisdiction over the subject matter, and are, moreover, Instance therefore had no jurisdiction to try and decide the case.
not apparent from the pleadings and evidence on record. Third, the property subject of partition is only the 47.2 hectare pro-indiviso area
representing the estate of Alicia. It does not include the entire 496 hectares of land comprising Hacienda Sta. Rita. Surety’s prayer: Prayed the Court of Appeals to set aside its decision and to dismiss the case.

Even petitioners argument that non-payment of appropriate docket fees by private respondents deprived the RTC of jurisdiction to partition the CA’s resolution: CA required the appellees to answer the motion to dismiss, but they failed to do so.
entire Hacienda Sta. Rita deserves scant consideration. In National Steel Corporation v. Court of Appeals, the Court ruled:
Decision:
x x x while the lack of jurisdiction of a court may be raised at any stage of an action, nevertheless, the party raising such
question may be estopped if he has actively taken part in the very proceedings which he questions and he only objects to the It would indeed appear from the record that the action at bar, which is a suit for collection of money in the sum of exactly
court’s jurisdiction because the judgment or the order subsequently rendered is adverse to him. P1,908.00 exclusive of interest, was originally instituted in the Court of First Instance of Cebu on July 19, 1948. But about a
month prior to the filing of the complaint, more specifically on June 17, 1948, the Judiciary Act of 1948 took effect, depriving the
Court of First Instance of original jurisdiction over cases in which the demand, exclusive of interest, is not more than P2,000.00.
Irrefragably, petitioners raised the issues of jurisdiction for lack of payment of appropriate docket fees and lack of cause of action belatedly in
their Memorandum before this Court. Cesar and petitioners’ were noticeably mum about these in the proceedings before. In fact, Cesar actively Xxxx
participated in the proceedings conducted before the RTC by seeking affirmative reliefs therefrom, such as the inclusion of more properties in
the partition. Hence, petitioners are already estopped from assailing the jurisdiction of the RTC on this ground. Considering, however, that the Supreme Court has the "exclusive" appellate jurisdiction over "all cases in which the jurisdiction
of any inferior court is in issue" (See. 1, Par. 3[3], Judiciary Act of 1948, as amended), we have no choice but to certify, as we
It is conceded that this Court adheres to the policy that where the court itself clearly has no jurisdiction over the subject matter or the nature of hereby do certify, this case to the Supreme Court
the action, the invocation of this defense may be done at any time. While it is the general rule that neither waiver nor estoppel shall apply to
confer jurisdiction upon a court, the Court may rule otherwise under meritorious and exceptional circumstances. One such exception is Tijam v. ACCORDINGLY, pursuant to Section 31 of the Judiciary Act of 1948 as amended, let the record of this case be forwarded to
Sibonghanoy, which finds application in this case. This Court held in Tijam that after voluntarily submitting a cause and encountering an the Supreme Court.
adverse decision on the merits, it is too late for the loser to question the jurisdiction or power of the court."

After an exhaustive study of the merits of the case and the pleadings submitted by the parties, this Court is convinced that the Court of Appeals
did not err in affirming the Order of the RTC which approved the Commissioners recommendations as to the manner of implementing the Order ISSUE: WON the motion to dismiss on the ground of lack of jurisdiction of the Court of First Instance during the pendency of the appeal will
of Partition of Alicia’s estate. There is no reason to reverse the Court of Appeals dismissal of petitioners Petition for Certiorari and Prohibition prosper.
and ruling that the RTC acted well-within its jurisdiction in issuing the assailed Order. Nowhere is it shown that the RTC committed such patent,
gross and prejudicial errors of law or fact, or a capricious disregard of settled law and jurisprudence, as to amount to a grave abuse of RULING:
discretion or lack of jurisdiction on its part, in adopting and confirming the recommendations submitted by the Commissioners, and which would
have warranted the issuance of a writ of certiorari. NO.

It is an undisputed fact that the action commenced by appellees in the Court of First Instance of Cebu against the Sibonghanoy spouses was
TOPIC: LACK OF JURISDICTION – RAISED AT EARLIEST POSSIBLE OPPORTUNITY
for the recovery of the sum of P1,908.00 only — an amount within the original exclusive jurisdiction of inferior courts in accordance with the
provisions of the Judiciary Act of 1948 which had taken effect about a month prior to the date when the action was commenced. True also is the The ruling of the Court of Appeals that "a party may be estopped from raising such [jurisdictional] question if he has actively taken part in the
rule that jurisdiction over the subject matter is conferred upon the courts exclusively by law, and as the lack of it affects the very authority of the very proceeding which he questions, belatedly objecting to the court’s jurisdiction in the event that the judgment or order subsequently
court to take cognizance of the case, the objection may be raised at any stage of the proceedings. However, considering the facts and rendered is adverse to him" is based on the doctrine of estoppel by laches. We are aware of that doctrine first enunciated by this Court in Tijam
circumstances of the present case — which shall forthwith be set forth — We are of the opinion that the Surety is now barred by laches from v. Sibonghanoy. In Tijam, the party-litigant actively participated in the proceedings before the lower court and filed pleadings therein. Only 15
invoking this plea at this late hour for the purpose of annulling everything done heretofore in the case with its active participation. years thereafter, and after receiving an adverse Decision on the merits from the appellate court, did the party-litigant question the lower court’s
jurisdiction. Considering the unique facts in that case, we held that estoppel by laches had already precluded the party-litigant from raising the
As already stated, the action was commenced in the Court of First Instance of Cebu on July 19, 1948, that is, almost fifteen years before the question of lack of jurisdiction on appeal. In Figueroa v. People, we cautioned that Tijam must be construed as an exception to the general rule
Surety filed its motion to dismiss on January 12, 1963 raising the question of lack of jurisdiction for the first time. and applied only in the most exceptional cases whose factual milieu is similar to that in the latter case.

A party may be estopped or barred from raising a question in different ways and for different reasons. Thus we speak of estoppel in pais, or The facts are starkly different in this case, making the exceptional rule in Tijam inapplicable. Here, petitioner Republic filed its Opposition to the
estoppel by deed or by record, and of estoppel by laches. application for registration when the records were still with the RTC. At that point, petitioner could not have questioned the delegated jurisdiction
of the MTC, simply because the case was not yet with that court. When the records were transferred to the MTC, petitioner neither filed
Laches, in a general sense is failure or neglect, for an unreasonable and unexplained length of time, to do that which, by exercising due pleadings nor requested affirmative relief from that court. On appeal, petitioner immediately raised the jurisdictional question in its Brief. Clearly,
diligence, could or should have been done earlier; it is negligence or omission to assert a right within a reasonable time, warranting a the exceptional doctrine of estoppel by laches is inapplicable to the instant appeal.
presumption that the party entitled to assert it either has abandoned it or declined to assert it.
Laches has been defined as the "failure or neglect, for an unreasonable and unexplained length of time, to do that which, by exercising due
The doctrine of laches or of "stale demands" is based upon grounds of public policy which requires, for the peace of society, the diligence, could or should have been done earlier; it is negligence or omission to assert a right within a reasonable time, warranting the
discouragement of stale claims and, unlike the statute of limitations, is not a mere question of time but is principally a question of the inequity or presumption that the party entitled to assert it either has abandoned or declined to assert it." In this case, petitioner Republic has not displayed
unfairness of permitting a right or claim to be enforced or asserted. such unreasonable failure or neglect that would lead us to conclude that it has abandoned or declined to assert its right to question the lower
court's jurisdiction.
It has been held that a party cannot invoke the jurisdiction of a court to sure affirmative relief against his opponent and, after obtaining or failing
to obtain such relief, repudiate or question that same jurisdiction. In the case just cited, by way of explaining the rule, it was further said that the We uphold the jurisdiction of the MTC, but remand the case to the court a quo for further proceedings in order to determine if the property in
question whether the court had jurisdiction either of the subject-matter of the action or of the parties was not important in such cases because question forms part of the alienable and disposable land of the public domain.
the party is barred from such conduct not because the judgment or order of the court is valid and conclusive as an adjudication, but for the
reason that such a practice cannot be tolerated — obviously for reasons of public policy.
SERAFIN TIJAM, ET AL. vs. MAGDALENO SIBONGHANOY alias GAVINO SIBONGHANOY and LUCIA BAGUIO, MANILA SURETY AND
Furthermore, it has also been held that after voluntarily submitting a cause and encountering an adverse decision on the merits, it is too late for FIDELITY CO., INC. (CEBU BRANCH)
the loser to question the jurisdiction or power of the court (Pease vs. Rathbun-Jones etc). And in Littletonvs. Burgess, the Court said that it is G.R. No. L-21450 April 15, 1968
not right for a party who has affirmed and invoked the jurisdiction of a court in a particular matter to secure an affirmative relief, to afterwards
deny that same jurisdiction to escape a penalty. Topic: Lack of jurisdiction over subject matter may be raised at any stage

Upon this same principle is what We said in the three cases mentioned in the resolution of the Court of Appeals of May 20, 1963 ( supra) — to Facts:
the effect that we frown upon the "undesirable practice" of a party submitting his case for decision and then accepting the judgment, only if Barely one month after the effectivity of the Judiciary Act of 1948, spouses Tijam commenced Civil Case No. R-660 in the Court of First
favorable, and attacking it for lack of jurisdiction, when adverse. Instance of Cebu against the spouses Sibonghanoy to recover from them the sum of P1,908.00, with legal interest thereon from the date of the
filing of the complaint until the whole obligation is paid, plus costs. A writ of attachment was issued by the court against defendants' properties,
From the time the Surety became a quasi-party on July 31, 1948, it could have raised the question of the lack of jurisdiction of the Court of First but the same was soon dissolved upon the filing of a counter-bond by defendants and the Manila Surety and Fidelity Co., Inc. After being duly
Instance of Cebu to take cognizance of the present action by reason of the sum of money involved which, according to the law then in force, served with summons the defendants filed their answer in which, after making some admissions and denials of the material averments of the
was within the original exclusive jurisdiction of inferior courts. It failed to do so. Instead, at several stages of the proceedings in the court a quo complaint, they interposed a counterclaim. This counterclaim was answered by the plaintiffs.
as well as in the Court of Appeals, it invoked the jurisdiction of said courts to obtain affirmative relief and submitted its case for a final
adjudication on the merits. It was only after an adverse decision was rendered by the Court of Appeals that it finally woke up to raise the The Court rendered judgment in favor of the plaintiffs and, after the same had become final and executory, upon motion of the latter, the Court
question of jurisdiction. Were we to sanction such conduct on its part, We would in effect be declaring as useless all the proceedings had in the issued a writ of execution against the defendants. The writ having been returned unsatisfied, the plaintiffs moved for the issuance of a writ of
present case since it was commenced on July 19, 1948 and compel the judgment creditors to go up their Calvary once more. The inequity and execution against the Surety's bond, against which the Surety filed a written opposition upon two grounds, namely, (1) Failure to prosecute and
unfairness of this is not only patent but revolting. (2) Absence of a demand upon the Surety for the payment of the amount due under the judgment.

The Court denied this motion on the ground solely that no previous demand had been made on the Surety for the satisfaction of the judgment.
Thereafter the necessary demand was made, and upon failure of the Surety to satisfy the judgment, the plaintiffs filed a second motion for
REPUBLIC OF THE PHILIPPINES vs. BANTIGUE POINT DEVELOPMENT CORPORATION execution against the counterbond. On the date set for the hearing thereon, the Court, upon motion of the Surety's counsel, granted the latter a
G. R. No. 162322 March 14, 2012 period of five days within which to answer the motion. Upon its failure to file such answer, the Court granted the motion for execution and the
corresponding writ was issued.
Topic: Lack of jurisdiction over subject matter may be raised at any stage
Subsequently, the Surety moved to quash the writ on the ground that the same was issued without the required summary hearing provided for
Facts: in Section 17 of Rule 59 of the Rules of Court. As the Court denied the motion, the Surety appealed to the Court of Appeals from such order of
On July 17, 1997, respondent Bantigue Point Development Corporation filed with the Regional Trial Court (RTC) of Rosario, Batangas an denial and from the one denying its motion for reconsideration. Although the appellees failed to file their brief, the Court of Appeals, on
application for original registration of title over Lot 8060 of Cad 453-D, San Juan Cadastre, with an area of more or less 10,732 square meters December 11, 1962, decided the case affirming the orders appealed from.
and with a total assessed value of ₱14,920.
On January 8, 1963, the Surety filed a motion asking for extension of time within which to file a motion for reconsideration. The Court of
On July 18, 1997, the RTC issued an Order setting the case for initial hearing on October 22, 1997. On August 7, 1997, it issued a second Appeals granted the motion in its resolution of January 10 of the same year. Thereafter, the Surety filed a pleading entitled MOTION TO
Order setting the initial hearing on November 4, 1997. DISMISS, alleging, among others, that the Court of First Instance had no jurisdiction to try and decide the case and prayed the Court of
Appeals to set aside its decision and to dismiss the case. The Court of Appeals required the appellees to answer the motion to dismiss, but
Petitioner Republic filed its Opposition to the application for registration on January 8, 1998 while the records were still with the RTC. On March failed to do so. On May 20 of the same year, the Court resolved to set aside its decision and to certify the case to Us. Hence, this petition.
31, 1998, the RTC Clerk of Court transmitted motu proprio the records of the case to the MTC of San Juan, because the assessed value of the
property was allegedly less than ₱100,000. Issue:
Whether or not the surety bond is estopped from questioning the jurisdiction of the CFI Cebu for the first time upon appeal.
Thereafter, the MTC entered an Order of General Default and commenced with the reception of evidence. Among the documents presented by
respondent in support of its application are Tax Declarations, a Deed of Absolute Sale in its favor, and a Certification from the Department of Ruling:
Environment and Natural Resources (DENR) Community Environment and Natural Resources Office (CENRO) of Batangas City that the lot in It is an undisputed fact that the action commenced by appellees in the Court of First Instance of Cebu against the Sibonghanoy spouses was
question is within the alienable and disposable zone. Thereafter, it awarded the land to respondent Corporation. for the recovery of the sum of P1,908.00 only — an amount within the original exclusive jurisdiction of inferior courts in accordance with the
provisions of the Judiciary Act of 1948 which had taken effect about a month prior to the date when the action was commenced. True also is the
Acting on an appeal filed by the Republic, the CA ruled that since the former had actively participated in the proceedings before the lower court, rule that jurisdiction over the subject matter is conferred upon the courts exclusively by law, and as the lack of it affects the very authority of the
but failed to raise the jurisdictional challenge therein, petitioner is thereby estopped from questioning the jurisdiction of the lower court on court to take cognizance of the case, the objection may be raised at any stage of the proceedings. However, considering the facts and
appeal. The CA further found that respondent Corporation had sufficiently established the latter’s registrable title over the subject property after circumstances of the present case — which shall forthwith be set forth — We are of the opinion that the Surety is now barred by laches
having proven open, continuous, exclusive and notorious possession and occupation of the subject land by itself and its predecessors-in- from invoking this plea at this late hour for the purpose of annuling everything done heretofore in the case with its active
interest even before the outbreak of World War II. participation.

Dissatisfied with the CA’s ruling, petitioner Republic filed this instant Rule 45 Petition. As already stated, the action was commenced in the Court of First Instance of Cebu on July 19, 1948, that is, almost fifteen years
before the Surety filed its motion to dismiss on January 12, 1963 raising the question of lack of jurisdiction for the first time .
Issue:
Whether or not petitioner Republic is not estopped from questioning the jurisdiction of the Municipal Trial Court over the application for original It must be remembered that although the action, originally, was exclusively against the Sibonghanoy spouses the Surety became a quasi-party
registration of land title even for the first time on appeal. therein since July 31, 1948 when it filed a counter-bond for the dissolution of the writ of attachment issued by the court of origin (Record on
Appeal, pp. 15-19). Since then, it acquired certain rights and assumed specific obligations in connection with the pending case, in accordance
Ruling: with sections 12 and 17, Rule 57, Rules of Court.
At the outset, we rule that petitioner Republic is not estopped from questioning the jurisdiction of the lower court, even if the former raised the
jurisdictional question only on appeal. The rule is settled that lack of jurisdiction over the subject matter may be raised at any stage of Upon the filing of the first motion for execution against the counter-bond the Surety not only filed a written opposition thereto praying for its
the proceedings. Jurisdiction over the subject matter is conferred only by the Constitution or the law. It cannot be acquired through a denial but also asked for an additional affirmative relief — that it be relieved of its liability under the counter-bond upon the grounds relied upon
waiver or enlarged by the omission of the parties or conferred by the acquiescence of the court. Consequently, questions of in support of its opposition — lack of jurisdiction of the court a quo not being one of them.
jurisdiction may be cognizable even if raised for the first time on appeal.
Then, at the hearing on the second motion for execution against the counter-bond, the Surety appeared, through counsel, to ask for time within
which to file an answer or opposition thereto. This motion was granted, but instead of such answer or opposition, the Surety filed the motion to Respondents' cause of action before the COSLAP pertains to their claim of ownership over the subject property, which is an action
dismiss mentioned heretofore. involving title to or possession of real property, or any interest therein, the jurisdiction of which is vested with the Regional Trial
Courts or the Municipal Trial Courts depending on the assessed value of the subject property.
A party may be estopped or barred from raising a question in different ways and for different reasons. Thus we speak of estoppel in pais, or
estoppel by deed or by record, and of estoppel by laches. Since the COSLAP has no jurisdiction over the action, all the proceedings therein, including the decision rendered, are null and void .
A judgment issued by a quasi-judicial body without jurisdiction is void. It cannot be the source of any right or create any obligation. All acts
Laches, in a general sense is failure or neglect, for an unreasonable and unexplained length of time, to do that which, by exercising due performed pursuant to it and all claims emanating from it have no legal effect. Having no legal effect, the situation is the same as it would be as
diligence, could or should have been done earlier; it is negligence or omission to assert a right within a reasonable time, warranting a if there was no judgment at all. It leaves the parties in the position they were before the proceedings.
presumption that the party entitled to assert it either has abandoned it or declined to assert it.
Respondents’ allegation that petitioner is estopped from questioning the jurisdiction of the COSLAP by reason of laches does not
The doctrine of laches or of "stale demands" is based upon grounds of public policy which requires, for the peace of society, the hold water. Petitioner is not estopped from raising the jurisdictional issue, because it may be raised at any stage of the proceedings,
discouragement of stale claims and, unlike the statute of limitations, is not a mere question of time but is principally a question of the inequity or even on appeal, and is not lost by waiver or by estoppel. The fact that a person attempts to invoke unauthorized jurisdiction of a court does
unfairness of permitting a right or claim to be enforced or asserted. not estop him from thereafter challenging its jurisdiction over the subject matter, since such jurisdiction must arise by law and not by mere
consent of the parties.
It has been held that a party can not invoke the jurisdiction of a court to sure affirmative relief against his opponent and, after obtaining or failing
to obtain such relief, repudiate or question that same jurisdiction. In the case just cited, by way of explaining the rule, it was further said that the In Regalado v. Go, the Court held that laches should be clearly present for the Sibonghanoy doctrine to apply, thus:
question whether the court had jurisdiction either of the subject-matter of the action or of the parties was not important in such cases because Laches is defined as the "failure or neglect for an unreasonable and unexplained length of time, to do that which, by exercising due
the party is barred from such conduct not because the judgment or order of the court is valid and conclusive as an adjudication, but for the diligence, could or should have been done earlier, it is negligence or omission to assert a right within a reasonable length of time,
reason that such a practice can not be tolerated — obviously for reasons of public policy. warranting a presumption that the party entitled to assert it either has abandoned it or declined to assert it."

Furthermore, it has also been held that after voluntarily submitting a cause and encountering an adverse decision on the merits, it is too late for In Sibonghanoy, the defense of lack of jurisdiction was raised for the first time in a motion to dismiss filed by the Surety almost 15 years after
the loser to question the jurisdiction or power of the court. And in Littleton vs. Burgess, 16 Wyo. 58, the Court said that it is not right for a party the questioned ruling had been rendered. At several stages of the proceedings, in the court a quo as well as in the Court of Appeals, the Surety
who has affirmed and invoked the jurisdiction of a court in a particular matter to secure an affirmative relief, to afterwards deny that same invoked the jurisdiction of the said courts to obtain affirmative relief and submitted its case for final adjudication on the merits. It was only when
jurisdiction to escape a penalty. the adverse decision was rendered by the Court of Appeals that it finally woke up to raise the question of jurisdiction.

The facts of this case show that from the time the Surety became a quasi-party on July 31, 1948, it could have raised the question of the lack of The factual settings attendant in Sibonghanoy are not present in the case at bar that would justify the application of estoppel by laches against
jurisdiction of the Court of First Instance of Cebu to take cognizance of the present action by reason of the sum of money involved which, the petitioner. Here, petitioner assailed the jurisdiction of the COSLAP when she appealed the case to the CA and at that time, no
according to the law then in force, was within the original exclusive jurisdiction of inferior courts. It failed to do so. Instead, at several stages of considerable period had yet elapsed for laches to attach. Therefore, petitioner is not estopped from assailing the jurisdiction of the
the proceedings in the court a quo as well as in the Court of Appeals, it invoked the jurisdiction of said courts to obtain affirmative relief and COSLAP. Additionally, no laches will even attach because the judgment is null and void for want of jurisdiction.
submitted its case for a final adjudication on the merits. It was only after an adverse decision was rendered by the Court of Appeals that it finally
woke up to raise the question of jurisdiction. Were we to sanction such conduct on its part, We would in effect be declaring as useless all the
MEGAN SUGAR CORPORATION vs. REGIONAL TRIAL COURT of ILOILO, Branch 68, Dumangas, Iloilo; New Frontier Sugar Corporation and
proceedings had in the present case since it was commenced on July 19, 1948 and compel the judgment creditors to go up their Calvary once
more. The inequity and unfairness of this is not only patent but revolting. EQUITABLE PCI BANK
G.R. No. 170352 June 1, 2011
Coming now to the merits of the appeal: after going over the entire record, We have become persuaded that We can do nothing better than to
quote in toto, with approval, the decision rendered by the Court of Appeals on December 11, 1962 x x x granting plaintiffs' motion for execution Topic: Objection to jurisdiction over a party; holding out theory or doctrine of ostensible agency
against the surety x x x.
Facts:
On July 23, 1993, respondent New Frontier Sugar Corporation (NFSC) obtained a loan from respondent Equitable PCI Bank (EPCIB) secured
CELIA S. VDA. DE HERRERA vs. EMELITA BERNARDO, EVELYN BERNARDO as Guardian of Erlyn, Crislyn and Crisanto Bernardo by a real estate mortgage over NFSC’s land and a chattel mortgage over NFSC’s sugar mill. Due to liquidity problems, NFSC entered into a
G.R. No. 170251 June 1, 2011 Memorandum of Agreement (MOA) with Central Iloilo Milling Corporation (CIMICO), whereby the latter agreed to take-over the operation and
management of the NFSC raw sugar factory and facilities for the period covering crop years 2000 to 2003.
Topic: Lack of jurisdiction over subject matter may be raised at any stage
On April 19, 2002, NFSC filed a compliant for specific performance and collection against CIMICO for the latter's failure to pay its obligations
Facts: under the MOA. CIMICO countered by filing with the Regional Trial Court (RTC) of Dumangas, Iloilo, Branch 68, a case against NFSC for sum
Respondent heirs of Crisanto S. Bernardo, represented by Emelita Bernardo, filed a complaint before the Commission on the Settlement of of money and/or breach of contract.
Land Problems (COSLAP) against Alfredo Herrera (Alfredo) for interference, disturbance, unlawful claim, harassment and trespassing over a
portion of a parcel of land situated at Barangay Dalig, Cardona, Rizal, with an area of 7,993 square meters. The complaint was docketed as On May 10, 2002, EPCIB instituted extra-judicial foreclosure proceedings due to NFSC's failure to pay. During public auction, EPCIB was the
COSLAP Case No. 99-221. sole bidder and was thus able to buy the entire property and consolidate the titles in its name. EPCIB then hired Philippine Industrial Security
Agency (PISA) to secure the land and the sugar mill.
Respondents claimed that said land was originally owned by their predecessor-in-interest, Crisanto Bernardo, and was later on acquired by
Crisanto S. Bernardo. This was later on covered by Tax Declaration No. CD-006-0828 under the name of the respondents. On September 16, 2002, CIMICO filed with the RTC an Amended Complaint where it impleaded PISA and EPCIB. As a result, upon the motion
of CIMICO, the RTC issued a restraining order, directing EPCIB and PISA to desist from taking possession over the property in dispute. Hence,
Petitioner, on the other hand, alleged that the portion of the subject property consisting of about 700 square meters was bought by Diosdado CIMICO was able to continue its possession over the property.
Herrera, Alfredo's father, from a certain Domingo Villaran. Upon the death of Diosdado Herrera, Alfredo inherited the 700-square-meter lot.
On October 3, 2002, CIMICO and petitioner Megan Sugar Corporation (MEGAN) entered into a MOA whereby MEGAN assumed CIMICO's
The COSLAP, in a Resolution dated December 6, 1999, ruled that respondents have a rightful claim over the subject property. Consequently, a rights, interests and obligations over the property. Thus, MEGAN started operating the sugar mill on November 18, 2002.
motion for reconsideration and/or reopening of the proceedings was filed by Alfredo. The COSLAP, in an Order dated August 21, 2002, denied
the motion and reiterated its Order dated December 6, 1999. Aggrieved, petitioner Celia S. Vda. de Herrera, as the surviving spouse of Alfredo, Thereafter, Passi Iloilo Sugar Central, Inc. (Passi Sugar) filed with the RTC a Motion for Intervention claiming to be the vendee of EPCIB.
filed a petition for certiorari with the CA. The CA, Twelfth Division, in its April 28, 2005 Decision, dismissed the petition and affirmed the During the hearing on the said motion, Atty. Reuben Mikhail Sabig (Atty. Sabig) appeared before the RTC as counsel for MEGAN. Several
resolution of the COSLAP. The CA ruled that the COSLAP has exclusive jurisdiction over the present case and, even assuming that the counsels objected to Atty. Sabig's appearance since MEGAN was not a party to the proceedings; however, Atty. Sabig manifested that his
COSLAP has no jurisdiction over the land dispute of the parties herein, petitioner is already estopped from raising the issue of jurisdiction statements would bind MEGAN.
because Alfredo failed to raise the issue of lack of jurisdiction before the COSLAP and he actively participated in the proceedings before the
said body. Petitioner filed a motion for reconsideration, which was denied by the CA in a Resolution dated October 17, 2005. Hence, petitioner Several motions were filed by EPCIB to hold in escrow the sugar quedans or the proceeds therefrom which was granted by the RTC. Atty.
elevated the case to this Court via Petition for Review on Certiorari under Rule 45 of the Rules of Court. Sabig filed an Omnibus Motion for Reconsideration and Clarification which was denied. Aggrieved by the orders issued by the RTC, MEGAN
filed before the CA a petition for certiorari arguing mainly on two points; first, that the RTC erred when it determined that MEGAN was
Petitioner averred that the COSLAP has no adjudicatory powers to settle and decide the question of ownership over the subject land. Further, subrogated to the obligations of CIMICO and; second, that the RTC had no jurisdiction over MEGAN. CA denied such motion, ruling that since
the present case cannot be classified as explosive in nature as the parties never resorted to violence in resolving the controversy. Petitioner Atty. Sabig had actively participated before the RTC, MEGAN was already estopped from assailing the RTCs jurisdiction. Aggrieved, MEGAN
submits that it is the Regional Trial Court which has jurisdiction over controversies relative to ownership of the subject property. then filed a Motion for Reconsideration, which was, however, denied by the CA. Hence, this petition.

Respondents, on the other hand, alleged that the COSLAP has jurisdiction over the present case. Further, respondents argued that petitioner is Issue:
estopped from questioning the jurisdiction of the COSLAP by reason of laches due to Alfredo's active participation in the actual proceedings Whether or not MEGAN is already estopped from assailing the jurisdiction of the RTC.
before the COSLAP. Respondents said that Alfredo's filing of the Motion for Reconsideration and/or Reopening of the proceedings before the Ruling:
COSLAP is indicative of his conformity with the questioned resolution of the COSLAP. After a judicial examination of the records pertinent to the case at bar, this Court agrees with the finding of the CA that MEGAN is already
estopped from assailing the jurisdiction of the RTC.
Issue:
Whether or not petitioner is estopped from questioning the jurisdiction of the COSLAP by reason of laches due to Alfredo's active participation The doctrine of estoppel is based upon the grounds of public policy, fair dealing, good faith and justice, and its purpose is to forbid one to speak
in the actual proceedings before the COSLAP. against his own act, representations, or commitments to the injury of one to whom they were directed and who reasonably relied thereon. The
Ruling: doctrine of estoppel springs from equitable principles and the equities in the case. It is designed to aid the law in the administration of justice
In the instant case, the COSLAP has no jurisdiction over the subject matter of respondents' complaint . The present case does not fall where without its aid injustice might result. It has been applied by this Court wherever and whenever special circumstances of a case so
under any of the cases enumerated under Section 3, paragraph 2 (a) to (e) of E.O. No. 561. The dispute between the parties is not critical and demand.
explosive in nature, nor does it involve a large number of parties, nor is there a presence or emergence of social tension or unrest.
Based on the events and circumstances surrounding the issuance of the assailed orders, this Court rules that MEGAN is estopped from
It is axiomatic that the jurisdiction of a tribunal, including a quasi-judicial officer or government agency, over the nature and subject matter of a assailing both the authority of Atty. Sabig and the jurisdiction of the RTC. While it is true, as claimed by MEGAN, that Atty. Sabig said in court
petition or complaint is determined by the material allegations therein and the character of the relief prayed for, irrespective of whether the that he was only appearing for the hearing of Passi Sugars motion for intervention and not for the case itself, his subsequent acts, coupled with
petitioner or complainant is entitled to any or all such reliefs. MEGANs inaction and negligence to repudiate his authority, effectively bars MEGAN from assailing the validity of the RTC proceedings under
the principle of estoppel.
plaintiff with leave of court to include the government of the After this Court issued an order on October 15, 1985 ordering the
MEGAN can no longer deny the authority of Atty. Sabig as they have already clothed him with apparent authority to act in their Republic as defendant and reducing the amount of damages, and re-assessment of the docket fee in the present case and other
behalf. It must be remembered that when Atty. Sabig entered his appearance, he was accompanied by Concha, MEGAN’s director and general attorney's fees prayed for to P100,000.00. Said amended cases that were investigated, on November 12, 1985 the trial court
manager. Concha himself attended several court hearings, and on December 17, 2002, even sent a letter to the RTC asking for the status of complaint was also admitted. directed plaintiffs to rectify the amended complaint by stating the
the case. A corporation may be held in estoppel from denying as against innocent third persons the authority of its officers or agents who have amounts which they are asking for. It was only then that plaintiffs
been clothed by it with ostensible or apparent authority. Atty. Sabig may not have been armed with a board resolution, but the appearance specified the amount of damages in the body of the complaint in
of Concha made the parties assume that MEGAN had knowledge of Atty. Sabig’s actions and, thus, clothed Atty. Sabig with apparent the reduced amount of P10,000,000.00. Still no amount of
authority such that the parties were made to believe that the proper person and entity to address was Atty. Sabig. Apparent authority, damages were specified in the prayer. Said amended complaint
or what is sometimes referred to as the "holding out" theory, or doctrine of ostensible agency, imposes liability, not as the result of was admitted.
the reality of a contractual relationship, but rather because of the actions of a principal or an employer in somehow misleading the
public into believing that the relationship or the authority exists. The action was considered not only one for recovery of ownership No such honest difference of opinion was possible as the
but also for damages, so that the filing fee for the damages allegations of the complaint, the designation and the prayer show
One of the instances of estoppel is when the principal has clothed the agent with indicia of authority as to lead a reasonably prudent person to should be the basis of assessment. Although the payment of the clearly that it is an action for damages and specific performance.
believe that the agent actually has such authority. With the case of MEGAN, it had all the opportunity to repudiate the authority of Atty. Sabig docketing fee of P60.00 was found to be insufficient, The docketing fee should be assessed by considering the amount
since all motions, pleadings and court orders were sent to MEGAN office. However, MEGAN never questioned the acts of Atty. Sabig and even nevertheless, it was held that since the payment was the result of of damages as alleged in the original complaint.
took time and effort to forward all the court documents to him. an "honest difference of opinion as to the correct amount to be
paid as docket fee" the court "had acquired jurisdiction over the
To this Court mind, MEGAN cannot feign knowledge of the acts of Atty. Sabig, as MEGAN was aware from the very beginning that CIMICO was case and the proceedings thereafter had were proper and
involved in an on-going litigation. regular." Hence, as the amended complaint superseded the
original complaint, the allegations of damages in the amended
The rule is that the active participation of the party against whom the action was brought, coupled with his failure to object to the complaint should be the basis of the computation of the filing fee.
jurisdiction of the court or administrative body where the action is pending, is tantamount to an invocation of that jurisdiction and a
willingness to abide by the resolution of the case and will bar said party from later on impugning the court or body’s jurisdiction .
Based on the preceding discussion, this Court holds that MEGAN's challenge to Atty. Sabig’s authority and the RTCs jurisdiction was a CA Ruling:
mere afterthought after having received an unfavorable decision from the RTC . Certainly, it would be unjust and inequitable to the other The basis of assessment of the docket fee should be the amount of damages sought in the original complaint and not in the amended
parties if this Court were to grant such a belated jurisdictional challenge. complaint.

Manchester Development v. Court of Appeals ISSUE:


G.R. No. 75919, May 7, 1987 Whether or not the court acquired jurisdiction over the case when the correct and proper docket fees has not been paid.
TOPIC: Payment of filing/docket fees – jurisdictional
SC Ruling:
NATURE OF THE CASE: Motion for Reconsideration A case is deemed filed only upon payment of the docket fee regardless of the actual date of filing in court." Thus, in the present case the trial
court did not acquire jurisdiction over the case by the payment of only P410.00 as docket fee. Neither can the amendment of the complaint
FACTS: thereby vest jurisdiction upon the Court. For all legal purposes there is no such original complaint that was duly filed which could be amended.
This was originally a case of an action for torts and damages and specific performance with a prayer for temporary restraining order. The Consequently, the order admitting the amended complaint and all subsequent proceedings and actions taken by the trial court are null and
damages were not specifically stated in the prayer but the body of the complaint assessed a P78.75M damages suffered by the petitioner. The void.
amount of docket fees paid was only P410.00. The petitioner then amended the complaint and reduced the damages to P10M only.
The Court cannot close this case without making the observation that it frowns at the practice of counsel who filed the original complaint in this
Petitioners in support of their contention that the �ling fee must be assessed on the basis of the amended complaint cite the case of Magaspi case of omitting any specification of the amount of damages in the prayer although the amount of over P78 million is alleged in the body of the
vs. Ramolete. They contend that the Court of Appeals erred in ruling that the filing fee should be levied by considering the amount of damages complaint. This is clearly intended for no other purpose than to evade the payment of the correct filing fees if not to mislead the docket clerk in
sought in the original complaint. the assessment of the filing fee. This fraudulent practice was compounded when, even as this Court had taken cognizance of the anomaly and
ordered an investigation, petitioner through another counsel filed an amended complaint, deleting all mention of the amount of damages being
asked for in the body of the complaint. It was only when in obedience to the order of this Court of October 18, 1985, the trial court directed that
Differences: the amount of damages be specified in the amended complaint, that petitioners' counsel wrote the damages sought in the much reduced
amount of P10,000,000.00 in the body of the complaint but not in the prayer thereof. The design to avoid payment of the required docket fee is
obvious.
MAGASPI CASE MANCHESTER CASE (present case)
To put a stop to this irregularity, henceforth all complaints, petitions, answers and other similar pleadings should specify the amount of damages
It was an action for recovery of ownership and possession of a An action for torts and damages and specific performance with being prayed for not only in the body of the pleading but also in the prayer, and said damages shall be considered in the assessment of the
parcel of land with damages prayer for temporary restraining order, etc. filing fees in any case. Any pleading that fails to comply with this requirement shall not be accepted nor admitted, or shall otherwise be
expunged from the record.
The prayer in the complaint seeks not only the annulment of title The prayer is for the issuance of a writ of preliminary prohibitory
of the defendant to the property, the declaration of ownership and injunction during the pendency of the action against the defendants' The Court acquires jurisdiction over any case only upon the payment of the prescribed docket fee. An amendment of the complaint or similar
delivery of possession thereof to plaintiffs but also asks for the announced forfeiture of the sum of P3 Million paid by the plaintiffs pleading will not thereby vest jurisdiction in the Court, much less the payment of the docket fee based on the amounts sought in the amended
payment of actual, moral, exemplary damages and attorney's for the property in question, to attach such property of defendants pleading. The ruling in the Magaspi case in so far as it is inconsistent with this pronouncement is overturned and reversed.
fees arising therefrom in the amounts specified therein that may be sufficient to satisfy any judgment that may be
rendered, and after hearing, to order defendants to execute a WHEREFORE, the motion for reconsideration is denied for lack of merit.
contract of purchase and sale of the subject property and annul
defendants' illegal forfeiture of the money of plaintiff, ordering
Nestle Phils. v. FY Sons, G.R. No. 150780, May 5, 2006
defendants jointly and severally to pay plaintiff actual,
TOPIC: Payment of filing/docket fees – jurisdictional
compensatory and exemplary damages as well as 25% of said
amounts as may be proved during the trial as attorney's fees and
NATURE OF THE CASE: petition for review on certiorari under Rule 45 of the Rules of Court assailing the decision of the CA which in turn
declaring the tender of payment of the purchase price of plaintiff
affirmed with modification the decision of the RTC, as well as the CA's resolution which denied petitioner's motion for reconsideration.
valid and producing the effect of payment and to make the
injunction permanent. The amount of damages sought is not
FACTS:
specified in the prayer although the body of the complaint alleges
Petitioner is a corporation engaged in the manufacture and distribution of all Nestle products nationwide. Respondent is a corporation engaged
the total amount of over P78 Million as damages suffered by
in trading, marketing, selling and distributing food items to restaurants and food service outlets. On December 23, 1998, Nestlé and FY Sons
plaintiff.
entered into a distributorship agreement (agreement) whereby petitioner would supply its products for respondent to distribute to its food
service outlets. FY Sons executed a deed of assignment in favor of Nestlé, assigning a time deposit of Calixto Laureano in the amount of
Upon the �ling of the complaint there was an honest difference of There can be no such honest difference of opinion. As may be P500k to secure FY Sons’ credit purchases from Nestlé. Laureano also executed a special power of attorney, authorizing FY Sons to use the
opinion as to the nature of the action in the Magaspi case. The gleaned from the allegations of the complaint as well as the time deposit as collateral.
complaint was considered as primarily an action for recovery of designation thereof, it is both an action
ownership and possession of a parcel of land. The damages for damages and specific performance. The docket fee paid upon The areas covered by the agreement were Baguio, Dagupan, Angeles, Bulacan, Pampanga, Urdaneta, La Union, Tarlac and Olongapo. At the
stated filing of complaint in the amount only of P410.00 by considering the end of 1989, the agreement expired and the parties executed a renewal agreement on January 22, 1990. A supplemental agreement was
were treated as merely ancillary to the main cause of action. action to be merely one for specific performance where the amount executed on June 27, 1990, to take effect on July 1, 1990.
Thus, the docket fee of only P60.00 and P10.00 for the sheriff's involved is not capable of pecuniary estimation is obviously
fee were paid. erroneous. Although the total amount of damages sought is not On July 2, 1990, Nestlé fined FY Sons P20,000 for allegedly selling 50 Krem-Top liquid coffee creamer cases to Lu Hing Market (Tarlac), as this
stated in the prayer of the complaint yet it is spelled out in the body was purportedly proscribed by the agreement. FY Sons paid the fine. When the same product was sold to Augustus Bakery and Grocery,
of the complaint totaling in the amount of P78,750,000.00 which another fine (P40,000) was imposed for the same reason.
should be the basis of assessment of the filing fee.
On October 19, 1990, FY Sons (through counsel) wrote Nestlé to demand damages and to complain about the latter’s breaches of their
The trial court ordered the plaintiffs to pay the amount of Plaintiff through another counsel with leave of court filed an agreement and the various acts of bad faith. On 5 Nov, Nestlé sent FY Sons a demand letter and notice of termination, alleging that FY Sons
P3,104.00 as filing fee covering the damages alleged in the amended complaint on September 12, 1985 for the inclusion of had outstanding accounts of P995k~. When the alleged accounts were not settled, Nestlé applied the P500k time deposit as partial payment.
original complaint as it did not consider the damages to be merely Philips Wire and Cable Corporation as co-plaintiff and by
ancillary or incidental to the action for recovery of ownership and eliminating any mention of the amount of damages in the body of FY Sons filed a complaint for damages against Nestlé, alleging bad faith.
possession of real property. An amended complaint was filed by the complaint. The prayer in the original complaint was maintained.
 Nestlé made representations of rendering support, so FY Sons was lured into executing an agreement.  the court acquires jurisdiction over the claim only upon payment of the prescribed docket fee.
 FY Sons invested money, time, and effort, only for Nestlé to breach the agreement by committing various acts of bad faith.
 By the agreement’s termination, Nestlé would obtain market gains made by FY Sons at the latter’s efforts and expenses.
Indeed, a court acquires jurisdiction over the claim of damages upon payment of the correct docket fees. In this case, it is not disputed that
respondent paid docket fees based on the amounts prayed for in its complaint. Respondent adduced evidence to prove its losses. It was proper
Respondent’s Prayer: for the CA and the RTC to consider this evidence and award the sum of P1,000,000. Had the courts below awarded a sum more than
Respondent sought actual damages of P1,000,000, moral damages of P200,000, exemplary damages of P100,000, attorney's fees of P1,000,000, which was the amount prayed for, an additional filing fee would have been assessed and imposed as a lien on the judgment.
P100,000, plus the return of the P500,000 time deposit and costs of suit. However, the courts limited their award to the amount prayed for.

Petitioner’s Answer: Given that petitioner was not able to prove that respondent had unpaid accounts in the amount of P995,319.81, the seizure of the P500,000
Petitioner interposed a counterclaim for P495,319.81 representing the balance of respondent's overdue accounts, with interest of 2% per month time deposit was improper. As a result, the refund of this amount with interest is also called for.
from the date of default until fully paid, moral damages of P100,000, exemplary damages of P200,000, attorney's fees of P120,000 and costs of
suit. Petitioner's counterclaims are necessarily without merit. It failed to prove the alleged outstanding accounts of respondent. Accordingly, it is not
entitled to the supposed unpaid balance of P495,319.81 with interest. Petitioner, being at fault and in bad faith, and there being no proof that
RTC: respondent was guilty of any wrongdoing, cannot claim moral and exemplary damages and attorney's fees from respondent.
RTC ruled in favor of the respondent and ordered the petitioner to pay P1,000,000 as actual damages, P100,000 as exemplary damages,
P100,000 as attorney’s fees and P53,214.26, the amount entitled from the petitioner. WHEREFORE, the petition is hereby DENIED for lack of merit. The decision of the Court of Appeals dated January 11, 2001 and resolution
dated November 14, 2001 in CA-G.R. CV No. 57299 are hereby AFFIRMED.

 Nestlé failed to provide support; it unjustifiably refused to deliver stocks; the imposition of the P20k fine was void for having no Sun Insurance v. Asuncion
basis; Nestlé terminated the agreement without sufficient basis and in bad faith. G.R. Nos. 79937-38, February 13, 1989
 Nestlé failed to prove FY Sons’ alleged outstanding obligation. TOPIC: Payment of filing/docket fees – jurisdictional
o Nestle’s Statement of Account showing the alleged unpaid balance is undated, and it does not show receipt
thereof by FY Sons, and when, if such indeed was received. FACTS:
o There are no supporting documents to sustain such unpaid accounts. On February 28, 1984, petitioner Sun Insurance filed a complaint with the RTC Makati for the consignation of a premium refund on a fire
insurance policy with a prayer for the judicial declaration of its nullity against private respondent Manuel Uy Po Tiong. Private respondent as
declared in default for failure to file the required answer within the reglementary period.
CA:
Rendered a decision affirming the RTC's decision with modification: (1) the actual damages is INCREASED from P1,000,000.00 to On the other hand, on March 28, 1984, private respondent filed a complaint in the RTC QC for the refund of premiums and the issuance of a
P1,500,000.00; and (2) the amount of P53,214.26 payable by the petitioner to the is DELETED. writ of preliminary attachment, initially against petitioner Sun Insurance, and thereafter including E.B. Philipps and D.J. Warby as additional
defendants. The complaint docketed as Civil Case Q-41177 sought, among others, the payment of damages. Although the prayer in the
ISSUES: complaint did not quantify the amount of damages sought said amount may be inferred from the body of the complaint to be about P50 Million.

(1) Whether or not petitioner’s witness (Rayos) is competent to testify. Only the amount of P210.00 was paid by private respondent as docket fee which prompted petitioners' counsel to raise his objection. Said
(2) Whether or not the CA erred in awarding damages and refund of the P500,000 time deposit. objection was disregarded by respondent Judge Jose P. Castro who was then presiding over said case. Upon the order of this Court, the
records of said case together with 22 other cases assigned to different branches of the RTC QC which were under investigation for under-
assessment of docket fees were transmitted to the SC. The SC ordered that the cases be re-raffled, the judges in said cases to reassess the
EVIDENCE FOR FY SONS EVIDENCE FOR NESTLÉ docket fees and that in case of deficiency, to order its payment. The Resolution also requires all clerks of court to issue certificates of re-
assessment of docket fees. All litigants were likewise required to specify in their pleadings the amount sought to be recovered in their
complaints.
Testimony of Cristina Rayos
Testimony of Florentino Yue, Jr.
Thus, Judge Solano, to whose sala Civil Case Q-41177 was temporarily assigned, instructed the Clerk of Court to issue a certificate of
 Prepared statement of account on the basis of the assessment of the docket fee paid by private respondent and, in case of deficiency, to include the same in said certificate. On January 7, 1984,
 Director and officer invoices and delivery orders corresponding to the to forestall a default, a cautionary answer was filed by petitioners. On August 30,1984, an amended complaint was filed by private respondent
alleged overdue accounts of FY Sons including the two additional defendants aforestated.

Respondent Judge Asuncion, to whom Civil Case No. Q41177 was thereafter assigned, after his assumption into office on January 16, 1986,
issued a Supplemental Order requiring the parties in the case to comment on the Clerk of Court's letter-report signifying her difficulty in
Re: Cristina Rayos complying with the Resolution of the SC since the pleadings filed by private respondent did not indicate the exact amount sought to be
recovered. Private respondent filed a "Compliance" and a "Re-Amended Complaint" stating therein a claim of "not less than P10 Million as
 She admitted that the invoices corresponding to the alleged overdue accounts are not signed, because that there were delivery actual compensatory damages" in the prayer. In the body of the said second amended complaint however, private respondent alleges actual
orders covering the transactions. However, she did not identify the signatures on the delivery orders as the persons who and compensatory damages and attorney's fees in the total amount of about P44,601,623.70.
received the goods for Nestlé.
 She could not have identified the same, for she was not involved in the delivery, as she is only in charge of the records and On January 24, 1986, Judge Asuncion issued another Order admitting the second amended complaint and stating therein that the same
documents on all accounts receivables as part of her duties as Credit and Collection Manager. constituted proper compliance with the SC Resolution and that a copy thereof should be furnished the Clerk of Court for the reassessment of
the docket fees. The reassessment by the Clerk of Court based on private respondent's claim of "not less than P10 M as actual and
compensatory damages" amounted to P39,786.00 as docket fee. This was subsequently paid by private respondent.
Re: FY Sons’ witness Yue, Jr.
Petitioners then filed a petition for certiorari with the CA questioning the said order of Judge Asuncion.
 Nestlé: Yue admitted in open court that FY Sons had an unpaid obligation to Nestlé of around P900k. On April 24, 1986, private respondent filed a supplemental complaint alleging an additional claim of P20 M as damages so the total claim
amounts to about P65 Million. Seven months after filing the supplemental complaint, the private respondent paid the additional docket fee of
 FY Sons: This statement was merely in response to the judge’s question on what ground Nestlé supposedly terminated the
P80,396.00.
agreement. Yue was not being asked, nor was he addressing, the truth of such ground.
 SC: Nestlé is WRONG, as it took Yue’s statement out of context; cannot be considered a judicial admission. On August 13, 1987, the CA denied the petition insofar as it seeks annulment of the order, and petitioner’s motion to dismiss the amended
complaint. Hence, the instant petition.
SC RULING: During the pendency of this petition and in conformity with the said judgment of respondent court, private respondent paid the additional docket
Rayos testified on a statement of account she prepared on the basis of invoices and delivery orders which she, however, knew nothing about. fee of P62,432.90 on April 28, 1988.
She had no personal knowledge of the facts on which the accounts were based since, admittedly, she was not involved in the delivery of goods
and was merely in charge of the records and documents of all accounts receivable as part of her duties as credit and collection manager. She ISSUE:
thus knew nothing of the truth or falsity of the facts stated in the invoices and delivery orders, i.e., whether such deliveries were in fact made in Whether or not the RTC acquire jurisdiction over the case even if there was nonpayment of the correct and proper docket fee.
the amounts and on the dates stated, or whether they were actually received by respondent. She was not even the credit and collection
manager during the period the agreement was in effect. This can only mean that she merely obtained these documents from another without Petitioners’ contention:
any personal knowledge of their contents. Thus, she was incompetent to testify on whether or not the invoices and delivery orders turned over Considering that the total amount sought to be recovered in the amended and supplemental complaint is P64,601,623.70 the docket fee that
to her correctly reflected the details of the deliveries made. should be paid by private respondent is P257,810.49, more or less. Not having paid the same, petitioners contend that the complaint should be
dismissed and all incidents arising therefrom should be annulled. As basis, petitioners cite Manchester Development Corporation vs. CA:
The CA declared that petitioner was not able to prove that respondent had unpaid accounts, thus debunking the claim of a valid termination. The Court acquires jurisdiction over any case only upon the payment of the prescribed docket fee. An amendment of the
The CA also held petitioner guilty of various acts which violated the provisions of the agreement. Consequently, for petitioner's breach of the complaint or similar pleading will not thereby vest jurisdiction in the Court, much less the payment of the docket fee based on
agreement, the CA awarded actual damages to the amounts sought in the amended pleading. The ruling in the Magaspi Case in so far as it is inconsistent with this
respondent in the amount of P1,000,000. Petitioner, other than claiming that it validly terminated the agreement, did not challenge the findings pronouncement is overturned and reversed.
of the CA that it committed various violations of the agreement. Hence, there was legal basis for the grant of actual damages.
Respondent’s contention:
Petitioner’s contention: Manchester cannot apply retroactively for at the time said civil case was filed in court there was no such ruling as yet. Magaspi v. Ramolete
applies wherein it was held that the trial court acquired jurisdiction over the case even if the docket fee paid was insufficient.
 documentary evidence presented by respondent to prove actual damages in the amount of P4,246,015.60 should not have
been considered because respondent's complaint only prayed for an award of P1,000,000. RULING:
YES, the court acquired jurisdiction over the case. compensating for the value of the lost goods, it suffered damages and was constrained to engage the services of counsel to enforce and
protect its right to recover compensation under said policies, for which services it obligated itself to pay the sum equivalent to twenty-five (25%)
Nevertheless, the contention that Manchester cannot retroactively apply is untenable. Statutes regulating the procedure of the courts will be of any amount recovered as and for attorney’s fees and legal expenses. Pyramid was assessed P610 docket fee, apparently on the basis of
construed as applicable to actions pending and undetermined at the time of their passage. Procedural laws are retrospective in that sense and the amount of P50,000 specified in the prayer representing attorney’s fees, which it duly paid.
to that extent.
Petitioners filed a Motion to Dismiss on the ground of, inter alia, lack of jurisdiction, Pyramid not having paid the docket fees in full arguing that
In Lazaro vs. Endencia and Andres, this Court held that the payment of the full amount of the docket fee is an indispensable step for the in the prayer in the Complaint, plaintiff deliberately omitted to specify what these damages are in order to evade the payment of the docket
perfection of an appeal. Plaintiff-appellant deposited the deficiency in the docket fee outside the 15-day reglementary period for appeal. Thus, fees. To the Motion to Dismiss Pyramid filed its Opposition, alleging that if there was a mistake in the assessment of the docket fees, the trial
the CFI (as appellate court) did not acquire jurisdiction as the appeal was not perfected. court was not precluded from acquiring jurisdiction over the complaint as “it has the authority to direct the mistaken party to complete the
docket fees in the course of the proceedings.”
In Lee vs. Republic, the petitioner filed a verified declaration of intention to become a Filipino citizen by sending it through registered mail to the
Office of the Solicitor General in 1953 but the required filing fee was paid only in 1956. Citing Lazaro, this Court concluded that the filing of RTC Makati:
petitioner's declaration of intention on October 23, 1953 produced no legal effect until the required filing fee was paid on May 23, 1956. Dismissed, saying that the case being for specific performance, it is not dismissible on that ground but unless proper docket fees are paid, the
RTC can only grant what was prayed for in the Complaint.
In Malimit vs. Degamo, the same principles enunciated in Lazaro and Lee were applied. It was an original petition for quo warranto contesting
the right to office of proclaimed candidates which was mailed, addressed to the clerk of the CFI, within the one-week period after the CA:
proclamation as provided therefor by law. However, the required docket fees were paid only after the expiration of said period. Consequently, Partially granted, ordering Pyramid to pay the correct docket fees on the basis of the losses alleged in the body of the complaint, plus the
this Court held that the date of such payment must be deemed to be the real date of filing of aforesaid petition and not the date when it was attorney’s fees mentioned in the prayer, within a reasonable time which should not go beyond the applicable prescriptive or reglementary
mailed. period.

Again, in Garica vs, Vasquez, this Court reiterated the rule that the docket fee must be paid before a court will act on a petition or complaint. Petitioners' Argument:
However, we also held that said rule is not applicable when petitioner seeks the probate of several wills of the same decedent as he is not They invoke the doctrine in Manchester Development Corporation v. Court of Appeals that a pleading which does not specify in the prayer the
required to file a separate action for each will but instead he may have other wills probated in the same special proceeding then pending before amount sought shall not be admitted or shall otherwise be expunged, and that the court acquires jurisdiction only upon the payment of the
the same court. prescribed docket fee.

Then in Magaspi, this Court reiterated the ruling in Malimit and Lee that a case is deemed filed only upon payment of the correct docket fee Respondent's Argument:
regardless of the actual date of its filing in court. In the said case, there was an honest difference of opinion as to the correct amount to be paid They invoke the application of Sun Insurance Office, Ltd. (SIOL) v. Asuncion and subsequent rulings relaxing the Manchester ruling by allowing
as docket fee because the action appears to be one for the recovery of property the docket fee of P60.00 was correct; and that as the action is payment of the docket fee within a reasonable time, in no case beyond the applicable prescriptive or reglementary period, where the filing of
also one, for damages, the SC upheld the assessment of the additional docket fee based on the damages alleged in the amended complaint as the initiatory pleading is not accompanied by the payment of the prescribed docket fee.
against the assessment of the trial court which was based on the damages alleged in the original complaint.
ISSUES:
However, SC overturned Magaspi in Manchester. Manchester involves an action for torts and damages and specific performance with a prayer
for the issuance of a temporary restraining order, etc.. The amount of damages sought is not specified in the prayer although the body of the 1. Whether respondent, Pyramid Logistics and Trucking Corporation (Pyramid), which filed a complaint, denominated as one for
complaint alleges the total amount of over P78 Millon allegedly suffered by plaintiff. Applying the principle in Magaspi that "the case is deemed specific performance and damages, against petitioners Philippine First Insurance Company, Inc. (Philippine First) and
filed only upon payment of the docket fee regardless of the actual date of filing in court," this Court held that the trial court did not acquire Paramount General Insurance Corporation (Paramount) before the Regional Trial Court (RTC) of Makati, paid the correct
jurisdiction over the case by payment of only P410.00 for the docket fee. Neither can the amendment of the complaint thereby vest jurisdiction docket fee
upon the Court. For all legal purposes there was no such original complaint duly filed which could be amended. Consequently, the order 2. If in the negative, whether the complaint should be dismissed or Pyramid can still be ordered to pay the fee.
admitting the amended complaint and all subsequent proceedings and actions taken by the trial court were declared null and void.

The facts and circumstances of the present case are similar to Manchester. The principle in Manchester could very well be applied in the SC RULING:
present case. The pattern and the intent to defraud the government of the docket fee due is obvious not only in the filing of the original Yes, Pyramid filed the correct docket fee.
complaint but also in the filing of the second amended complaint.
In the case of Tacay vs. RTC of Tagum, Davao del Norte, the SC clarified the effect of the Sun Insurance ruling on the Manchester ruling as
However, in Manchester, petitioner did not pay any additional docket fee until the case was decided by the SC on May 7, 1987. Thus, in follows:
Manchester, due to the fraud committed on the government, this Court held that the court a quo did not acquire jurisdiction over the case and
that the amended complaint could not have been admitted inasmuch as the original complaint was null and void. The requirement in Circular No. 7 that complaints, petitions, answers, and similar pleadings should specify the amount of damages being
prayed for not only in the body of the pleading but also in the prayer, has not been altered. What has been revised is the rule that subsequent
In the present case, a more liberal interpretation of the rules is called for considering that, unlike Manchester, private respondent demonstrated amendment of the complaint or similar pleading will not thereby vest jurisdiction in the Court, much less the payment of the docket fee based
his willingness to abide by the rules by paying the additional docket fees as required. on the amount sought in the amended pleading, the trial court now being authorized to allow payment of the fee within a reasonable time but in
no case beyond the applicable prescriptive period or reglementary period. Moreover, a new rule has been added, governing the awards of
Nevertheless, petitioners contend that the docket fee that was paid is still insufficient considering the total amount of the claim. This is a matter claims not specified in the pleading – i.e., damages arising after the filing of the complaint or similar pleading – as to which the additional filing
which the clerk of court of the lower court and/or his duly authorized docket clerk or clerk in-charge should determine and, thereafter, if any fee therefore shall constitute a lien on the judgment.
amount is found due, he must require the private respondent to pay the same.
In the case at bar, Pyramid failed to specify in its prayer the amount of claims/damages it was seeking both in the original and amended
Thus, the Court rules as follows: complaint. It reasoned out that it was not aware of the extent of the liability of the insurance companies under their respective policies. It left the
1. It is not simply the filing of the complaint or appropriate initiatory pleading, but the payment of the prescribed docket fee, that vests a trial matter of liability to the trial court’s determination.
court with jurisdiction over the subject matter or nature of the action. Where the filing of the initiatory pleading is not accompanied by payment
of the docket fee, the court may allow payment of the fee within a reasonable time but in no case beyond the applicable prescriptive or Even assuming that the amounts are yet to be determined, the rule in Manchester, as modified by Sun Insurance, still applies. In the case of
reglementary period. Ayala Corporation vs. Madayag, the SC pronounced the following: While it is true that the determination of certain damages x x x is left to the
sound discretion of the court, it is the duty of the parties claiming such damages to specify the amount sought on the basis of which the court
2. The same rule applies to permissive counterclaims, third party claims and similar pleadings, which shall not be considered filed until and may make a proper determination, and for the proper assessment of the appropriate docket fees. The exception contemplated as to claims not
unless the filing fee prescribed therefor is paid. The court may also allow payment of said fee within a reasonable time but also in no case specified or to claims although specified are left for determination of the court is limited only to any damages that may arise after the filing of the
beyond its applicable prescriptive or reglementary period. complaint or similar pleading for then it will not be possible for the claimant to specify nor speculate as to the amount thereof.
3. Where the trial court acquires jurisdiction over a claim by the filing of the appropriate pleading and payment of the prescribed filing fee but,
subsequently, the judgment awards a claim not specified in the pleading, or if specified the same has been left for determination by the court, Home Guaranty Corp. v. R-11 Home Guaranty Corp. v. R-11 Builders
the additional filing fee therefor shall constitute a lien on the judgment. It shall be the responsibility of the Clerk of Court or his duly authorized G.R. No. 192649, March 9, 2011
deputy to enforce said lien and assess and collect the additional fee. TOPIC: Payment of filing/docket fees – jurisdictional

WHEREFORE, the petition is DISMISSED for lack of merit. The Clerk of Court of the court a quo is hereby instructed to reassess and FACTS:
determine the additional filing fee that should be paid by private respondent considering the total amount of the claim sought in the original On 19 March 1993, a Joint Venture Agreement (JVA) was entered into between respondents National Housing Authority (NHA) and R-II
complaint and the supplemental complaint as may be gleaned from the allegations and the prayer thereof and to require private respondent to Builders, Inc. (R-II Builders) for the implementation of the Smokey Mountain Development and Reclamation Project (SMDRP).
pay the deficiency, if any, without pronouncement as to costs.
On 26 September 1994, NHA and R-II Builders, alongside petitioner Housing Guaranty Corporation (HGC) as guarantor and the Philippine
National Bank (PNB) as trustee, entered into an Asset Pool Formation Trust Agreement which provided the mechanics for the implementation
of the project. To back the project, an Asset Pool was created.
Phil. First Insurance v. Paramount Gen. Insurance
G.R. No. 165147, July 9, 2008 On the same date, the parties likewise executed a Contract of Guaranty whereby HGC, upon the call made by PNB and conditions therein
TOPIC: Payment of filing/docket fees – jurisdictional specified, undertook to redeem the regular Smokey Mountain Project Participation Certificate (SMPPCs) upon maturity and to pay the simple
interest thereon to the extent of 8.5% per annum.
FACTS:
On November 8, 2000, the delivery van of Pyramid bearing license plate number PHL-545 which was loaded with goods belonging to California Subsequent to R-II Builders' infusion of P300 Million into the project, the issuance of the SMPPCs and the termination of PNB’s services on 29
Manufacturing Corporation (CMC) valued at P907,149.07 left the CMC Bicutan Warehouse but the van, together with the goods, failed to reach January 2001, NHA, R-II Builders and HGC agreed on the institution of Planters Development Bank (PDB) as trustee on 29 January 2001. By
its destination and its driver and helper were nowhere to be found, to its damage and prejudice; that it filed a criminal complaint against the 24 October 2002, however, all the Regular SMPPCs issued had reached maturity and, unredeemed, already amounted to an aggregate face
driver and the helper for qualified theft, and a claim with herein petitioners as co-insurers of the lost goods but, in violation of petitioners’ value of P2.513 Billion. The lack of liquid assets with which to effect redemption of the regular SMPPCs prompted PDB to make a call on
undertaking under the insurance policies, they refused without just and valid reasons to compensate it for the loss; and that as a direct HGC’s guaranty and to execute in the latter’s favor a Deed of Assignment and Conveyance (DAC) of the entire Asset Pool.
consequence of petitioners’ failure, despite repeated demands, to comply with their respective undertakings under the Insurance Policies by
where the court has no jurisdiction over the original complaint and the purpose of the amendment is to confer jurisdiction upon the court.
On 1 September 2005, R-II Builders filed the complaint against HGC and NHA before Branch 24 of the Manila Regional Trial Court, a Special Hence, with jurisdiction over the case yet to properly attach, HGC correctly fault the CA for upholding respondent RTC's admission of R-II
Commercial Court (SCC). Contending that HGC’s failure to redeem the outstanding regular SMPPCs despite obtaining possession of the Asset Builders' Second Amended Complaint despite non-payment of the docket fees for its original complaint and Amended and Supplemental
Pool ballooned the stipulated interests and materially prejudiced its stake on the residual values of the Asset Pool, R-II Builders alleged, among Complaint as well as the clear intent to evade payment thereof.
other matters, that the DAC should be rescinded since PDB exceeded its authority in executing the same prior to HGC’s redemption and
payment of the guaranteed SMPPCs.
Unicapital v. Consing
G.R. Nos. 175277 & 175285, September 11, 2013
Having filed its answer to the complaint, in the meantime, HGC went on to move for the conduct of a preliminary hearing on its affirmative
TOPIC: Payment of filing/docket fees – jurisdictional
defenses which included such grounds as lack of jurisdiction. On 2 August 2007, R-II Builders, in turn, filed a motion to admit its Amended and
Supplemental Complaint which deleted the prayer for resolution of the DAC initially prayed for in its original complaint. In lieu thereof, said
FACTS:
pleading introduced causes of action for conveyance of title to and/or possession of the entire Asset Pool, for NHA to pay the Asset Pool the
Petitioner Consing obtained for himself and his mother an 18M loan from Unicapital. It was secured by Promissory notes and a Real estate
sum of P1,803,729,757.88 representing the cost of the changes and additional works on the project and for an increased indemnity for
mortgage constituted on a parcel of land in Cavite registered under the name of the mother of petItioner, Cecilia dela Cruz. In pursuance with
attorney’s fees in the sum of P2,000,000.00.
the option to purchase the mortgaged property, Unicapital agreed to purchase one half of the property. The payment was set-off by the loan
while the other half was purchased by Plus Builders Inc. (PBI), a joint venture partner of Unicapital. Before both companies could develop the
Consistent with its joint order dated 2 January 2008 which held that R-II Builders’ complaint was an ordinary civil action and not an intra-
land, they discovered that the property was in the names of Po Willie Yu and Juanito Tan Teng and that dela Cruz' title was a mere forgery.
corporate controversy, Branch 24 of the Manila RTC issued a clarificatory order dated 1 February 2008 to the effect, among other matters, that
Thus, PBI and Unicapital send separate demand letters to dela Cruz and Consing, seeking for the return of the purchase price they had paid
it did not have the authority to hear the case. As a consequence, the case was re-raffled to respondent Branch 22 of the Manila RTC. R-II
for the subject property.
Builders filed a motion to admit it Second Amended Complaint, on the ground that its previous Amended and Supplemental Complaint had not
yet been admitted in view of the non-payment of the correct docket fees therefor. Said Second Amended Complaint notably resurrected R-II
On May 3, 1999, Consing filed a complaint, denominated as a Complex Action for Declaratory Relief and later amended to Complex Action for
Builders’ cause of action for resolution of the DAC, deleted its causes of action for accounting and conveyance of title to and/or possession of
Injunctive Relief before the RTC-Pasig City docketed as SCA No. 1759. In his complaint, he claimed that the incessant demands/recovery
the entire Asset Pool, reduced the claim for attorney’s fees to P500,000.00, sought its appointment as Receiver pursuant to Rule 59 of the
efforts made upon him constituted harassment and oppression which severely affected his personal and professional life. He also averred that
Rules of Court and, after an inventory in said capacity, prayed for approval of the liquidation and distribution of the Asset Pool in accordance
he was coerced to commit a violation of Batas Pambansa Blg. 22 as Unicapital and PBI, over threats of filing a case against him, kept on
with the parties’ agreements.
forcing him to issue a post-dated check in the amount sought to be recovered, notwithstanding their knowledge that he had no funds for the
same. He further alleged that Unicapital and URI required him to sign blank deeds of sale and transfers without cancelling the old ones in
On 2 September 2008, HGC filed its opposition to the admission of R-II Builders’ Second Amended Complaint on
violation of the laws on land registration and real estate development. Likewise, he added that Unicapital and PBI’s representatives were
the ground that respondent RTC had no jurisdiction to act on the case until payment of the correct docket fees.
speaking of him in a manner that was inappropriate and libelous. For their part, Unicapital, et al. filed separate Motions to Dismiss Consing's
complaint on the ground of failure to state a cause of action. They posited that the RTC-Pasig City did not acquire jurisdiction over the case
HGC’s contention:
given that Consing failed to pay the proper amount of docket fees and that it had no jurisdiction over their supposed violations of the
Corporation Code and Revised Securities Act. The RTC-Pasig City issued a Resolution denying the above motions to dismiss, holding that
(a) the case is real action and the docket fees paid by R-II Builders were grossly insufficient because the estimated value of
Consing, Jr.’s complaint sufficiently stated a cause of action for tort and damages pursuant to Article 19 of the Civil Code. Unicapital et al.
properties in the Asset Pool exceeds P5,000,000,000.00.
moved for reconsideration which was, however, denied. Aggrieved, they elevated the denial of their motions to dismiss before the CA via a
(b) a complaint cannot be amended to confer jurisdiction when the court had none;
petition for certiorari and prohibition. The CA rendered a Decision holding that no grave abuse of discretion was committed by the RTC-Pasig
(c) the RTC should have simply denied the Urgent Ex-Parte Motion for Annotation of Lis Pendens instead of rendering an advisory
City in refusing to dismiss Consing's complaint.
opinion thereon. In addition, HGC faulted R-II Builders with forum shopping, in view of its 10 filing of the complaint before
Branch 91 of the Quezon City RTC, involving a claim for receivables from the NHA. In turn, R-II Builders opposed the foregoing On the other hand, on August 4, 1999, Unicapital filed a complaint for sum of money with damages against Consing and Dela Cruz before the
motion and, on the theory that the Asset Pool was still in danger of dissipation, filed an urgent motion to resolve its application RTC-Makati City, docketed as Civil Case No. 99-1418. PBI also filed a complaint for damages and attachment before the RTC of Manila,
for the appointment of a receiver and submitted its nominees for said position. Branch 12, docketed as Civil Case No. 99-95381, also predicated on the same set of facts. For his part, Consing filed a Motion to Dismiss
which was, however, denied. Thereafter, he filed a Motion for Consolidation of Civil Case No. 99-1418 with his own initiated SCA No. 1759
pending before the RTC-Pasig City. The RTC-Makati City dismissed his motion for consolidation. His motion for reconsideration therefrom was
RTC: also denied. Hence, he filed a petition for certiorari before the CA ascribing grave abuse of discretion on the part of the RTC-Makati City in
(a) denied HGC's motion for reconsideration; refusing his motion for consolidation. The CA rendered a Decision sustaining the orders of the RTC-Makati City which denied the motion for
(b) granted R-II Builders' application for appointment of receiver consolidation. Undaunted, he filed a motion for reconsideration therefrom but was denied.

Imputing grave abuse of discretion against the RTC for not dismissing the case and for granting R-II Builders' application for receivership, HGC ISSUE:
filed the Rule 65 petition for certiorari and prohibition before the CA which rendered decision, upon the following findings and conclusions: Whether or not the CA erred in upholding the RTC- Pasig City’s denial of Unicapital, et al.’s motion to dismiss.

(a) Irrespective of whether it is real or one incapable of pecuniary estimation, the action commenced by R-II Builders indubitably SC RULING:
falls squarely within the jurisdiction of respondent RTC; A cause of action is defined as the act or omission by which a party violates a right of another. It is well-settled that the existence of a cause of
(b) From the allegations of R-II Builders' original complaint and amended complaint the character of the relief primarily sought, i.e., action is determined by the allegations in the complaint. In this relation, a complaint is said to sufficiently assert a cause of action if, admitting
the declaration of nullity of the DAC, the action before respondent RTC is one where the subject matter is incapable of what appears solely on its face to be correct, the plaintiff would be entitled to the relief prayed for. Thus, if the allegations furnish adequate
pecuniary estimation; basis by which the complaint can be maintained, then the same should not be dismissed, regardless of the defenses that may be averred by
(c) R-II Builders need not pay any deficiency in the docket fees considering its withdrawal of its Amended and Supplemental the defendants.
Complaint;
(d) A receiver may be appointed without formal hearing, particularly when it is within the interest of both parties and does not result In this case, the Court finds that Consing, Jr.’s complaint in SCA No.1759 properly states a cause of action since the allegations there
in the delay of any government infrastructure projects or economic development efforts; insufficiently bear out a case for damages under Articles 19 and 26 of the Civil Code.
(e) Respondent RTC's act of calling the attention of the Manila Registrar of Deeds to R-II Builders' Urgent Ex-Parte Motion for
Annotation of Lis Pendens is well-within its residual power to act on matters before it; and Records disclose that Consing, Jr.’s complaint contains allegations which aim to demonstrate the abusive manner in which Unicapital and PBI,
(f) The withdrawal of R-II Builders' Amended and Supplemental Complaint discounted the forum shopping imputed against it by et al. enforced their demands against him. Accordingly, these specific allegations, if hypothetically admitted, may result into the recovery of
damages pursuant to Article 19 of the Civil Code.
HGC.
Neither should Consing, Jr.’s failureto pay the required docket fees lead to the dismissal of his complaint. It has long been settled that while the
court acquires jurisdiction over any case only upon the payment of the prescribed docket fees, its non-payment at the time of the filing of the
ISSUES: complaint does not automatically cause the dismissal of the complaint provided that the fees are paid within a reasonable period.
Whether or not the CA erred in upholding respondent RTC's admission of R-II Builders' Second Amended Complaint despite non-payment of Consequently, Unicapital, et al.’s insistence that the stringent rule on non-payment of docket fees enunciated in the case of Manchester
the docket fees for its original complaint and Amended and Supplemental Complaint. Development Corporation v. CA should be applied in this case cannot be sustained in the absence of proof that Consing, Jr. intended to
defraud the government by his failure to pay the correct amount of filing fees. As pronounced in the case of Heirs of Bertuldo Hinog v. Hon.
SC RULING: Melicor:
Having admitted that its original complaint partook the nature of a real action and having been directed to pay the correct docket fees for its
Amended and Supplemental Complaint, R-II Builders is, furthermore, clearly chargeable with knowledge of the insuf ficiency of the docket fees Plainly, while the payment of the prescribed docket fee is a jurisdictional requirement, even its non-payment at the time of filing
it paid. Unmistakably manifesting its intent to evade payment of the correct docket fees, moreover, R-II Builders withdrew its Amended and does not automatically cause the dismissal of the case, as long as the fee is paid within the applicable prescriptive or
Supplemental Complaint after its admission and, in lieu thereof, filed its' Second Amended Complaint on the ground that said earlier pleading reglementary period, more so when the party involved demonstrates a willingness to abide by the rules prescribing such
cannot be considered admitted in view of its non-payment of the docket and other fees it was directed to pay. In so doing, however, R-II payment. Thus, when insufficient filing fees were initially paid by the plaintiffs and there was no intention to defraud the
Builders conveniently overlooked the fact that the very same argument could very well apply to its original complaint for which — given its government, the Manchester rule does not apply.
admitted nature as a real action — the correct docket fees have also yet to be paid.
In fine, the Court finds no reversible error on the part of the CA in sustaining the RTC-Pasig City's denial of Unicapital, et al.'s motion to dismiss.
For nonpayment of the correct docket fees which, for real actions, should be computed on the basis of the assessed value of the property, or if As such, the petitions in G.R. Nos. 175277 and 175285 must be denied.
there is none, the estimated value thereof as alleged by the claimant, respondent RTC should have denied admission of R-II Builders' Second
Amended Complaint and ordered the dismissal of the case. Although a catena of decisions rendered by this Court eschewed the application of
the doctrine laid down in the Manchester case, said decisions had been consistently premised on the willingness of the party to pay the correct DURISOL Philippines, Inc. vs Court of Appeals
docket fees and/or absence of intention to evade payment of the correct docket fees. This cannot be said of R-II Builders which not only failed G.R. No. 121106; February 20, 2002
to pay the correct docket fees for its original complaint and Amended and Supplemental Complaint but also clearly evaded payment of the TOPIC: Kinds of Jurisdiction-General Jurisdiction-RTC
same by filing its Second Amended Complaint.
CASE NATURE:
By itself, the propriety of admitting R-II Builders' Second Amended Complaint is also cast in dubious light when viewed through the prism of the This is a petition for review of the decision of the Court of Appeals in dismissing petitioner Durisol Philippines, Inc.’s petition for
general prohibition against amendments intended to confer jurisdiction where none has been acquired yet. Although the policy in this annulment of judgment.
jurisdiction is to the effect that amendments to pleadings are favored and liberally allowed in the interest of justice, amendment is not allowed
FACTS: (2) the late Alejandro Montañer, Sr. is a Muslim;
On January 17, 1962 and December 5, 1969, petitioner Durisol obtained industrial loans from respondent Development Bank (3) petitioners are the first family of the decedent;
o3f the Philippines (DBP) amounting to P1,213,000.00 and P2,698,800.00, respectively. As security therefor, petitioner executed a mortgage on (4) Liling Disangcopan is the widow of the decedent;
two parcels of registered land located in Polo (now Valenzuela), Bulacan, covered by Transfer Certificates of Title Nos. 29906 and 29909. (5) Almahleen Liling S. Montañer is the daughter of the decedent; and
(6) the estimated value of and a list of the properties comprising the estate of the decedent. .
PLAINTIFF’S PRAYER:
Private respondents prayed for the Shari’a District Court to order, among others, the following:
CAUSE OF THE DISPUTE:
(1) the partition of the estate of the decedent; and
After petitioner defaulted in the payment of the loans, DBP instituted a petition for the extrajudicial foreclosure of mortgage. On
(2) the appointment of an administrator for the estate of the decedent
March 6, 1972, petitioner’s president, Rene Knecht, borrowed from DBP the two TCT’s purportedly to obtain new titles in accordance with the
DEFENDANT’S ANSWER:
approved subdivision plan of the properties. DBP agreed provided that the bank’s existing encumbrances, including the mortgage, shall be
Petitioners filed an Answer with a Motion to Dismiss mainly on the following grounds:
annotated on all the new certificates of title. Contrary to its promise, however, petitioner never returned the titles to the properties to DBP. Thus,
(1) the Shari’a District Court has no jurisdiction over the estate of the late Alejandro Montañer, Sr., because he was a Roman
despite having purchased the properties at the foreclosure sale, DBP was unable to register the property in its name.
Catholic;
The foreclosure sale was already held, wherein DBP emerged as the highest bidder. Petitioner, however, filed a complaint for
(2) private respondents failed to pay the correct amount of docket fees; and
annulment of the extrajudicial foreclosure before the then Court of First Instance (CFI) of Valenzuela, Bulacan. The CFI rendered judgment
(3) private respondents’ complaint is barred by prescription, as it seeks to establish filiation between Almahleen Liling S.
upholding the validity of the foreclosure which then affirmed by the Court of Appeals.
Montañer and the decedent, pursuant to Article 175 of the Family Code.
CASE ORIGINALLY FILED:
On February 25, 1977, DBP instituted before the Court of First Instance of Valenzuela, Bulacan, Branch VIII, a petition for
On November 22, 2005, the Shari’a District Court dismissed the private respondents’ complaint. The district court held that
surrender of the owner’s duplicate titles covering the foreclosed properties.
Alejandro Montañer, Sr. was not a Muslim, and its jurisdiction extends only to the settlement and distribution of the estate of deceased Muslims.
PLAINTIFF’S AVERMENTS:
ISSUE:
That petitioner failed to exercise its right of redemption of the properties which were sold at public auction after foreclosure of
WHETHER OR NOT THE SHARI’A DISTRICT COURT HAS JURISDICTION OVER THE SETTLEMENT OF THE ESTATE OF DECEASED
the mortgage thereof.
MUSLIMS;
DEFENDANT’S ANSWER:
WHETHER OR NOT SHARI’A DISTRICT COURT – MARAWI CITY LACKS JURISDICTION OVER PETITIONERS WHO ARE ROMAN
Raising the defenses that the petition fails to state a cause of action; that it had already paid its loans to DBP; that it had a valid
CATHOLICS AND NON-MUSLIMS.
adverse claim on the properties covered by the seven new titles; and that DBP’s action was barred by laches and estoppel.
RTC:
RULING: YES
The trial court rendered summary judgment, ordering petitioner to surrender to the court within five days the seven certificates
Article 143(b) of Presidential Decree No. 1083, otherwise known as the Code of Muslim Personal Laws of the
of title.
Philippines, provides that the Shari’a District Courts have exclusive original jurisdiction over the settlement of the estate of
Petitioner filed a motion for reconsideration, which contained an alternative prayer to record in the titles its adverse claim
deceased Muslims:
representing the amount of improvements it introduced on the property.
ARTICLE 143. Original jurisdiction. — (1) The Shari'a District Court shall have exclusive original jurisdiction
The lower court denied petitioner’s motion for reconsideration
over:
CA:
xxxx
Petitioner thus appealed to the Intermediate Appellate Court, the IAC rendered a decision ordering that the case be remanded
(b) All cases involving disposition, distribution and settlement of the estate of
to the lower court for further proceedings. The IAC held that it was improper for the trial court to render summary judgment because there were
deceased Muslims, probate of wills, issuance of letters of administration or appointment of
genuine issues involved. This decision became final and executory.
administrators or executors regardless of the nature or the aggregate value of the property.
More than four years later, or on September 2, 1994, petitioner instituted before the Court of Appeals a petition to annul the trial
The determination of the nature of an action or proceeding is controlled by the averments and character of the relief sought in
court’s decision and Resolution, alleging for the first time that the trial court had no jurisdiction over the case.
the complaint or petition. The designation given by parties to their own pleadings does not necessarily bind the courts to treat it according to the
The Court of Appeals rendered the now assailed decision dismissing the petition for annulment of judgment. Petitioner
said designation. Rather than rely on "a falsa descriptio or defective caption," courts are "guided by the substantive averments of the
Durisol’s subsequent motion for reconsideration was likewise denied for lack of merit.
pleadings."
ISSUE:
The court did not agree with the contention of the petitioners that the district court does not have jurisdiction over the case
(1) Whether or not the trial court had jurisdiction over the petition for issuance of new duplicate owner’s certificate of title; and
because of an allegation in their answer with a motion to dismiss that Montañer, Sr. is not a Muslim. Jurisdiction of a court over the nature of
(2) Whether or not petitioner was estopped from challenging the court’s lack of jurisdiction.
the action and its subject matter does not depend upon the defenses set forth in an answer or a motion to dismiss. Otherwise, jurisdiction
would depend almost entirely on the defendant or result in having "a case either thrown out of court or its proceedings unduly delayed by
RULING: YES
simple stratagem. Indeed, the "defense of lack of jurisdiction which is dependent on a question of fact does not render the court to lose or be
The first paragraph of Rule 47, Section 2, of the 1997 Rules of Civil Procedure provides:
deprived of its jurisdiction."
Grounds for annulment. — The annulment may be based only on the ground of extrinsic
The Shari’a District Court has the authority to hear and receive evidence to determine whether it has jurisdiction, which
fraud and lack of jurisdiction.
requires an a priori determination that the deceased is a Muslim. If after hearing, the Shari’a District Court determines that the deceased was
At the outset, it should be stressed that in a petition for annulment of judgment based on lack of jurisdiction, petitioner must
not in fact a Muslim, the district court should dismiss the case for lack of jurisdiction.
show not merely an abuse of jurisdictional discretion but an absolute lack of jurisdiction. Lack of jurisdiction means absence of or no
jurisdiction, that is, the court should not have taken cognizance of the petition because the law does not vest it with jurisdiction over the subject
matter. Jurisdiction over the nature of the action or subject matter is conferred by law. Misael Vera v. Hon. Judge Francisco Arca
The regional trial court, formerly the court of first instance, is a court of general jurisdiction. All cases, the jurisdiction over which G.R. No. L-25721, May 26, 1969
is not specifically provided for by law to be within the jurisdiction of any other court, fall under the jurisdiction of the regional trial court. But the TOPIC: Cases on the constitutionality of treaty, international or executive agreement, law, presidential decree, proclamation, order, instruction
regional trial court is also a court of limited jurisdiction over, among others, cadastral and land registration cases. All proceedings involving title
to real property, or specifically land registration cases, including its incidents such as the issuance of owner’s duplicate certificate of title, are FACTS:
matters cognizable by the regional trial courts. It has been ruled that the regional trial courts have jurisdiction over all actions involving It was alleged that the Tax Census Act has been enforced and implemented since 1962 and that the parties who filed the suit,
possession of land, except forcible entry and illegal detainer now respondents before us, "are required to make and file Sworn Statements of Assets, Income and Liabilities" in accordance with the Act. It
Even assuming arguendo that the regional trial court had no jurisdiction over the surrender of duplicate title, petitioner can no was likewise asserted that the then Senator Camilo Osias filed a bill to repeal such legislation and that the Secretary of Finance had admitted
longer raise this ground after having actively participated in the prosecution of the case. A judgment rendered by a trial court for alleged lack of that it had not been able to produce the result expected from it, the information yielded not justifying the trouble caused not only to the public
jurisdiction cannot be considered void where the party who has the right to challenge it failed to do so at the first instance. but also to the government.
Indeed, it was only two decades after the institution of the case at bar, when the issue of lack of jurisdiction was first raised. The main portion of the petition before respondent Judge dealt with the alleged infirmity of the Tax Census Act as being
However, it is already too late since the judgment had already attained finality, considering that more than four years have elapsed without any violative of the constitutional right to liberty, to the guarantee against self-incrimination and the protection against unreasonable searches and
action from petitioner. seizures with a citation from both Philippine and American cases in support of such a plea. A writ of preliminary injunction was therein likewise
Rule 47, Section 3 expressly provides that a petition for annulment of judgment based on lack of jurisdiction must be filed sought, the argument being advanced that the other respondents before us in the petition before respondent Judge "would suffer great and
before it is barred by laches or estoppel. Hence, it has been held that while jurisdiction over the subject matter of a case may be raised at any irreparable damage arising from the non-filing of their Sworn Statement of Assets, Income and Liabilities, as they are subject to criminal
time of the proceedings, this rule presupposes that laches or estoppel has not supervened. prosecution under Section 5 of the Tax Census Law which is in clear violation of herein petitioners' aforesaid constitutional and legal rights and
which would render the judgment in favor of herein petitioners ineffectual"
Montañer v. Shari’ah District Court
G.R. No. 174975, January 20, 2009
TOPIC: Exclusive Original Jurisdiction- Shari’ah District Court-Settlement of Estate of Deceased Muslims CAUSE OF THE DISPUTE:

CASE NATURE: Then came the order of respondent Judge, which is the basis of the present petition for certiorari and prohibition, noting that a
This Petition for Certiorari and Prohibition seeks to set aside the Orders of the Shari’a District Court, Fourth Shari’a Judicial hearing on the plea for the issuance of the writ for preliminary injunction took place on February 19, 1966 and ordering the issuance thereof
District, Marawi City. upon the posting of a bond of P1,000.00, thus restraining petitioners from requiring the other respondents and other similarly situated to file
their sworn statements of assets, income and liabilities under Republic Act No. 2070.
FACTS:

PLAINTIFF’S AVERMENTS:
On August 17, 1956, petitioner Luisa Kho Montañer, a Roman Catholic, married Alejandro Montañer, Sr. at the Immaculate Petitioners predicate their plea on the allegation that respondent Judge gravely abused his discretion in issuing the writ of
Conception Parish in Cubao, Quezon City. Petitioners Alejandro Montañer, Jr., Lillibeth Montañer-Barrios, and Rhodora Eleanor Montañer- preliminary injunction as the Tax Census Act is valid and constitutional, there being neither any self-incrimination feature nor unreasonable
Dalupan are their children. On May 26, 1995, Alejandro Montañer, Sr. died. search and seizure taint, there being moreover a presumption of its conformity with the fundamental law and no grave and irreparable injury
CASE ORIGINALLY FILED: being suffered by the other respondents, petitioners before respondent Judge.
On August 19, 2005, private respondents Liling Disangcopan and her daughter, Almahleen Liling S. Montañer, both Muslims, Petitioners likewise justify their contention that there was a grave abuse of discretion on the part of respondent Judge in the
filed a "Complaint" for the judicial partition of properties before the Shari’a District Court. issuance of such writ of preliminary injunction due to his failure to consider the serious injury it would cause the paramount public interest, to
PLAINTIFF’S AVERMENTS: realize that the enforcement of penal laws cannot thus be restrained and to take note that the other respondents as petitioners before him are
(1) in May 1995, Alejandro Montañer, Sr. died; guilty of laches.
PLAINTIFF’S PRAYER: ISSUE:
Petitioners in this special civil action seek the setting aside of the writ of preliminary injunction issued by respondent Judge and WHETHER OR NOT THE SUPREME COURT MAY ENTERTAIN THE CASE.
would restrain him perpetually from further hearing the suit for prohibition and injunction pending before him. RULING: NO
DEFENDANT’S ANSWER:
Respondents reiterate their argument against the validity of the Act for the asserted transgression on the constitutional
protection against self-incrimination and against unreasonable searches and seizures. The instant petition is a direct recourse from the assailed orders of the RTC. Notably, petitioners did not cite the rule under the
They did likewise question its validity as being in excess of the State's taxing power. Rules of Court by which the petition was filed. If the petition is to be treated as a petition filed under Rule 65 of the Rules of Court, the petition
ISSUE: must be dismissed outright for having been filed prematurely.
WHETHER OR NOT RESPONDENT JUDGE OUGHT TO HAVE ISSUED THE WRIT OF PRELIMINARY INJUNCTION TO RESTRAIN THE Primarily, although the Supreme Court, the Court of Appeals and the Regional Trial Courts have concurrent jurisdiction to issue
ENFORCEMENT OF THE TAX CENSUS ACT. writs of certiorari, prohibition, mandamus, quo warranto, habeas corpus and injunction, such concurrence does not give the petitioner
RULING: NO unrestricted freedom of choice of court forum.
Whatever may be said of the original petition for prohibition and injunction filed by the other respondents before the respondent Similarly, there are no special and important reasons that petitioners cite to justify their direct recourse to this Court under Rule
Judge, it cannot be plausibly asserted the facts have been alleged which would make manifest the violation of any of their constitutional rights. 65. On the other hand, direct recourse to the Court has been allowed for petitions filed under Rule 45 when only questions of law are raised, as
Instead of relying on facts, they contended themselves with the general allegation that for them the Tax Census Act was null. It was assailed for in this case.
presumably violating the right to liberty, the protection against unreasonable searches and seizures and the prohibition against self- Section 1 of Rule 45 clearly states that the following may be appealed to the Supreme Court through a petition
incrimination. What was thus being sought in effect was a declaration of invalidity based on the belief that its constitutional infirmity is apparent for review by certiorari: 1) judgments; 2) final orders; or 3) resolutions of the Court of Appeals, the Sandiganbayan, the
on its face. Regional Trial Court or similar courts, whenever authorized by law. The appeal must involve only questions of law, not of fact.
Respondent Judge took into consideration purely legal arguments, no evidence being introduced, both for and against the A direct recourse to the Supreme Court can be taken for a review of the decisions, final orders or resolutions of the RTC, but
validity of the challenged statute. Moreover, his attention was invited to the presumption of validity that every legislative act has in its favor as only on questions of law. Under Section 5 of Article VIII of the Constitution, the Supreme Court has the power to
well as the doctrine that the task of suspending the operation of the law "is a matter of extreme delicacy because that is an interference with the (2) Review, revise, reverse, modify, or affirm on appeal or certiorari as the law or the Rules of Court may
official acts not only with the duly elected representatives of the people in Congress but also of the highest magistrate of the land." Respondent provide, final judgments and orders of lower courts in:
Judge was deaf to the force of such cogent and persuasive constitutional law doctrines. He issued the preliminary injunction nonetheless. (e) All cases in which only an error or question of law is involved.
It is manifest that respondent Judge did overstep the bounds of discretion that set limits to the authority he is entitled to This kind of direct appeal to this Court of RTC judgments, final orders or resolutions is provided for in Section 2(c) of Rule 41,
exercise in the issuance of the preliminary injunction to restrain the enforcement of a statute. There can be no dissent from the proposition that which reads:
where the action required of a lower court would be tantamount, even if only for a temporary period, to disregarding the clearly expressed will of SEC. 2. Modes of appeal.
the two branches of the government, the need for caution is greatest. Here, respondent Judge was apparently oblivious of such a need. (c) Appeal by certiorari. In all cases where only questions of law are raised or involved, the appeal shall be to
Respondent Judge did possess discretion to issue or not to issue a preliminary injunction. That discretion, however, according the Supreme Court by petition for review on certiorari in accordance with Rule 45.
to the circumstances disclosed, was abused, and abused gravely.
Procedurally then, petitioners could have appealed the RTC Decision affirming the MTC (1) to the Court on questions of law
only; or (2) if there are factual questions involved, to the CA -- as they in fact did. Unfortunately for petitioners, the CA properly dismissed their
Constancio Mendoza v. Mayor Enrilo Villas petition for review because of serious procedural defects. This action foreclosed their only available avenue for the review of the factual
findings of the RTC.
G.R. No. 187256, February 23, 2011 Thus, the Court shall exercise liberality and consider the instant petition as one filed under Rule 45. The Court noted that it has
TOPIC: Petitions for Certiorari, Prohibition, Mandamus, Quo Warranto, Habeas Corpus, writs of Amparo, Habeas Data, Kalikasan, injunctions the discretion to determine whether a petition was filed under Rule 45 or 65 of the Rules of Court.
CASE NATURE: Nevertheless, even providing that the petition was not filed prematurely, it must still be dismissed for having become moot and
This is a Petition filed by Constancio F. Mendoza and Sangguniang Barangay of Balatasan, Bulalacao, Oriental Mindoro. In the academic.
Petition, it is prayed that the Court: (1) set aside the Order dated February 2, 2009 of the Regional Trial Court (RTC), Branch 43 in Roxas, With the conduct of the 2010 barangay elections, a supervening event has transpired that has rendered this case moot and
Oriental Mindoro denying petitioners’ motion for reconsideration of the Order dated February 2, 2009; and (2) direct the RTC to continue with academic and subject to dismissal. This is because Mendoza’s term of office has expired with the conduct of last year’s local elections. As
the proceedings in Special Civil Action No. 08-10 entitled Constancio Mendoza v. Mayor Enrilo Villas. such, Special Civil Action No. 08-10, where the assailed Orders were issued, can no longer prosper. Mendoza no longer has any legal standing
FACTS: to further pursue the case, rendering the instant petition moot and academic.
In the 2007 barangay elections, Mendoza obtained the highest votes for the position of Punong Barangay of Barangay
Balatasan, Bulalacao, Oriental Mindoro, while respondent Liwanag Herato obtained the highest number of votes for the position of Barangay
Kagawad. Notably, Mayor Enrilo Villas was the incumbent Mayor of Bulalacao, Oriental Mindoro at the time of the barangay elections.
CREBA v. DAR Secretary

CAUSE OF THE DISPUTE: G.R. 183409, June 18, 2010


TOPIC: Petitions for Certiorari, Prohibition

After the elections, the Commission on Elections (COMELEC) proclaimed Mendoza as the duly-elected Punong Barangay of CASE NATURE:
Balatasan. Thus, the losing candidate, Thomas Pajanel, filed a petition for quo warranto with the Municipal Trial Court (MTC) of Mansalay- This is Petition for Certiorari and Prohibition (with application for temporary restraining order and/or writ of preliminary
Bulalacao. The MTC issued a Decision dated February 23, 2008, disqualifying Mendoza and declaring that Herato was entitled to succeed him injunction) under Rule 65 of the 1997 Revised Rules of Civil Procedure, filed by herein petitioner Chamber of Real Estate and Builders
as Punong Barangay with Herato garnering the highest number of votes as a Barangay Kagawad. Mendoza appealed the MTC Decision to the Associations, Inc. (CREBA) seeking to nullify and prohibit the enforcement of Department of Agrarian Reform (DAR) Administrative Order (AO)
COMELEC. No. 01-02, as amended by DAR AO No. 05-07,and DAR Memorandum No. 88, for having been issued by the Secretary of Agrarian Reform
On February 28, 2008, Villas administered the Oath of Office to Herato. Then, Villas issued a Memorandum directing all with grave abuse of discretion amounting to lack or excess of jurisdiction as some provisions of the aforesaid administrative issuances are
department heads of the Municipal Government to act only on documents signed or authorized by Herato. illegal and unconstitutional.
Meanwhile, Mendoza sought the advice of the Department of the Interior and Local Government (DILG) as to who should FACTS:
exercise the powers of Punong Barangay of Balatasan given the prevailing controversy. DILG Undersecretary Austere A. Panadero responded
to Mendoza’s inquiry informing Villas that Mendoza should occupy the post of Punong Barangay as there was no Writ of Execution Pending
Appeal of the MTC Decision. Petitioner CREBA, a private non-stock, non-profit corporation duly organized and existing under the laws of the Republic of the
Philippines, is the umbrella organization of some 3,500 private corporations, partnerships, single proprietorships and individuals directly or
indirectly involved in land and housing development, building and infrastructure construction, materials production and supply, and services in
CASE ORIGINALLY FILED: the various related fields of engineering, architecture, community planning and development financing. The Secretary of Agrarian Reform is
Petitioners filed a Petition for Mandamus with Damages and Prayer for the Writ of Preliminary Mandatory Injunction, pending named respondent as he is the duly appointive head of the DAR whose administrative issuances are the subject of this petition.
with the Regional Trial Court, Branch 43 in Roxas, Oriental Mindoro. The Secretary of Agrarian Reform issued, on 29 October 1997, DAR AO No. 07-97, entitled "Omnibus Rules and Procedures
PLAINTIFF’S PRAYER: Governing Conversion of Agricultural Lands to Non-Agricultural Uses," which consolidated all existing implementing guidelines related to land
Petitioners prayed that the LBP be directed to release the funds of Barangay Balatasan to them in order to render necessary, use conversion. The aforesaid rules embraced all private agricultural lands regardless of tenurial arrangement and commodity produced, and
basic public services to the inhabitants of the barangay. all untitled agricultural lands and agricultural lands reclassified by Local Government Units (LGUs) into non-agricultural uses after 15 June
DEFENDANT’S ANSWER: 1988.
Subsequently, on 30 March 1999, the Secretary of Agrarian Reform issued DAR AO No. 01-99, entitled "Revised Rules and
Regulations on the Conversion of Agricultural Lands to Non-agricultural Uses," amending and updating the previous rules on land use
conversion.
Villas and Herato interpose the following affirmative defenses: Its coverage includes the following agricultural lands, to wit: (1) those to be converted to residential, commercial, industrial,
(1) that the petition for mandamus was defective, being directed against two or more different entities and requiring to perform institutional and other non-agricultural purposes; (2) those to be devoted to another type of agricultural activity such as livestock, poultry, and
different acts; and fishpond ─ the effect of which is to exempt the land from the Comprehensive Agrarian Reform Program (CARP) coverage; (3) those to be
(2) that Mendoza does not have any clear and legal right for the writ of mandamus. converted to non-agricultural use other than that previously authorized; and (4) those reclassified to residential, commercial, industrial, or other
non-agricultural uses on or after the effectivity of Republic Act No. 6657 and are to be converted to such uses.
On 28 February 2002, the Secretary of Agrarian Reform issued another Administrative Order, i.e., DAR AO No. 01-02, entitled
RTC: "2002 Comprehensive Rules on Land Use Conversion," which further amended DAR AO No. 07-97 and DAR AO No. 01-99, and repealed all
issuances inconsistent therewith. The aforesaid DAR AO No. 01-02 covers all applications for conversion from agricultural to non-agricultural
uses or to another agricultural use.
CAUSE OF THE DISPUTE:
The RTC issued the assailed order dismissing the petition on the strength of the COMELEC Resolution disqualifying Mendoza Thereafter, the Secretary of Agrarian Reform amended certain provisions of DAR AO No. 01-02 by formulating DAR AO No.
from running in the 2007 elections. As stated, petitioners’ motion for reconsideration of the Order was denied. 05-07, particularly addressing land conversion in time of exigencies and calamities.
From such orders the petitioners went directly to the Supreme Court. To address the unabated conversion of prime agricultural lands for real estate development, the Secretary of Agrarian Reform
further issued Memorandum No. 88, which temporarily suspended the processing and approval of all land use conversion applications.
By reason thereof, petitioner claims that there is an actual slowdown of housing projects, which, in turn, aggravated the Refused to petioner’s demand
housing shortage, unemployment and illegal squatting problems to the substantial prejudice not only of the petitioner and its members but more
so of the whole nation. RTC:
PLAINTIFF’S AVERMENTS: Dismissed the case
Petitioner contends that DAR AO No. 01-02, as amended, was made in violation of Section 65 of Republic Act No. 6657 Basis/reason
because it covers all applications for conversion from agricultural to non-agricultural uses or to other agricultural uses, such as the conversion The trial court held that petitioner failed to prove that respondent Clark Liberty engaged in unfair competition as there is no
of agricultural lands or areas that have been reclassified by the LGUs or by way of Presidential Proclamations, to residential, commercial, showing that it employed deceit or otherwise committed acts constituting bad faith; that the bottles of Fundador brandy
industrial or other non-agricultural uses. imported by respondent are the ones imported by plaintiff and that these bottles are not genuine, defective, or of poor quality.
Petitioner further asseverates that Section 2.19, Article I of DAR AO No. 01-02, as amended, making reclassification of Motion for reconsideration RTC:
agricultural lands subject to the requirements and procedure for land use conversion, violates Section 20 of Republic Act No. 7160, because it Denied
was not provided therein that reclassification by LGUs shall be subject to conversion procedures or requirements, or that the DAR’s approval or
clearance must be secured to effect reclassification. CA:
Petitioner similarly avers that the promulgation and enforcement of DAR AO No. 01-02, as amended, constitute deprivation of Dismissed the case
liberty and property without due process of law. There is deprivation of liberty and property without due process of law because under DAR AO Basis/reason
No. 01-02, as amended, lands that are not within DAR’s jurisdiction are unjustly, arbitrarily and oppressively prohibited or restricted from Lack of jurisdiction as quoted
legitimate use on pain of administrative and criminal penalties. It is an admitted fact respondent Clark Liberty is one of the duly licensed and authorized duty free shops at the Clark Special
PLAINTIFF’S PRAYER: Economic Zone since 1998 which sells imported grocery items including liquors, appliances, household wares, etc. and is
Petitioner avows that DAR Memorandum No. 88 is not a valid exercise of police power for it is the prerogative of the legislature exclusively regulated by the Clark Development Corporation, created by Republic Act No. 7227, known as the Bases
and that it is unconstitutional because it suspended the land use conversion without any basis. Conversion and Development Act of 1992. It is therefore a juridical creation of Republic Act No. 7227 in relation to Executive
ISSUE: Order No. 62 and Presidential Proclamation No. 163, creating the Clark Special Economic Zone, under the exclusive
WHETHER OR NOT THE DAR SECRETARY HAS JURISDICTION OVER LANDS THAT HAVE BEEN RECLASSIFIED AS RESIDENTIAL, jurisdiction, authority and regulation of the Clark Development Corporation. As such juridical creation, this Court has no
COMMERCIAL, INDUSTRIAL, OR FOR OTHER NON-AGRICULTURAL USES. jurisdiction to determine whether or not petitioner is entitled to the issuance of an injunctive relief since such authority and
RULING: YES jurisdiction belong the Honorable Supreme Court in accordance with Section 21 of Republic Act No. 7227
The authority of the Secretary of Agrarian Reform to include "lands not reclassified as residential, commercial, industrial or
other non-agricultural uses before 15 June 1988" in the definition of agricultural lands finds basis in jurisprudence. ISSUE:
In Ros v. Department of Agrarian Reform, this Court has enunciated that after the passage of Republic Act No. 6657, Wether or not the CA has jurisdiction of the case
agricultural lands, though reclassified, have to go through the process of conversion, jurisdiction over which is vested in the DAR. However,
agricultural lands, which are already reclassified before the effectivity of Republic Act No. 6657 which is 15 June 1988, are exempted from RULING:
conversion. Jurisdiction is the authority to hear and determine a cause. Jurisdiction over the subject matter is the power to hear and
It bears stressing that the said date of effectivity of Republic Act No. 6657 served as the cut-off period for automatic determine the general class to which the proceedings in question belong. Jurisdiction over the subject matter is conferred by
reclassifications or rezoning of agricultural lands that no longer require any DAR conversion clearance or authority. It necessarily follows that law and not by the consent or acquiescence of any or all of the parties or by erroneous belief of the court that it exists. Basic is
any reclassification made thereafter can be the subject of DAR’s conversion authority. Having recognized the DAR’s conversion authority over the rule that jurisdiction over the subject matter is determined by the cause or causes of action as alleged in the complaint. But
lands reclassified after 15 June 1988, it can no longer be argued that the Secretary of Agrarian Reform was wrongfully given the authority and where the actual issues are evident from the records of the case, then jurisdiction over the subject matter need not depend
power to include "lands not reclassified as residential, commercial, industrial or other non-agricultural uses before 15 June 1988" in the upon the literal averments in the complaint, but on the law as applied to established facts.
definition of agricultural lands. Here, in order to determine whether the court a quo has jurisdiction over petitioners complaint for injunction, we have to
Primarily, although this Court, the Court of Appeals and the Regional Trial Courts have concurrent jurisdiction to issue writs interpret the law as applied to the established facts. There is no question that respondent Clark Liberty is a registered
of certiorari, prohibition, mandamus, quo warranto, habeas corpus and injunction, such concurrence does not give the petitioner unrestricted enterprise of the Clark Special Economic Zone and is primarily regulated by R.A. No. 7227, otherwise known as the Bases
freedom of choice of court forum. Conversion and Development Act of 1992.
The rationale for this rule is two-fold: (a) it would be an imposition upon the precious time of this Court; and (b) it would cause The underlying purpose of the Legislature in enacting R.A. No. 7227 is provided by Section 2, thus:
an inevitable and resultant delay, intended or otherwise, in the adjudication of cases, which in some instances had to be remanded or referred SEC. 2. Declaration of Policies. It is hereby declared the policy of the Government to accelerate the sound and balanced
to the lower court as the proper forum under the rules of procedure, or as better equipped to resolve the issues because this Court is not a trier conversion into alternative productive uses of the Clark and Subic military reservations and their extensions (John Hay Station,
of facts. Wallace Air Station, ODonnell Transmitter Station, San Miguel Naval Communications Station and Capas Relay station), to
The Court thus reaffirms the judicial policy that it will not entertain direct resort to it unless the redress desired cannot be raise funds by the sale of portions of Metro Manila military camps and to apply said funds for the development and conversion
obtained in the appropriate courts, and exceptional and compelling circumstances, such as cases of national interest and of serious to productive civilian use of the lands covered under the 1947 Military Bases Agreement between the Philippines and the
implications, justify the availment of the extraordinary remedy of writ of certiorari, calling for the exercise of its primary jurisdiction. United States of America, as amended.
In the case at bench, petitioner failed to specifically and sufficiently set forth special and important reasons to justify direct It is likewise the declared policy of the Government to enhance the benefits to be derived from said properties in order to
recourse to this Court and why this Court should give due course to this petition in the first instance. The present petition should have been promote the economic and social development of Central Luzon in particular and the country in general.
initially filed in the Court of Appeals in strict observance of the doctrine on the hierarchy of courts. Failure to do so is sufficient cause for the Republic Act No. 7227 goes on further to provide that:
dismissal of this petition. SEC. 4. Purposes of the Conversion Authority. The Conversion authority shall have the following purposes:
xxx
(c) To encourage the active participation of the private sector in transforming the Clark and Subic military reservations and their
Allied Domecq Phil. v. Hon. Villon extensions into other productive uses;
G.R. No. 156264, September 30, 2004 The establishment, registration, and operation of respondent Clark Liberty and the other enterprises within the Clark Special
TOPIC: Manner of Acquiring Jurisdiction over subject matter Economic Zone are projects (involving the private sector) which convert Clark Air Base, a military reservation, into productive
CASE NATURE: uses. In this connection, Section 21 of R.A. No. 7227 provides:
Petition for review on certiorari assailing the Decision of the Court of Appeals dated May 27, 2002 and Resolution dated November 29, 2002 in SEC. 21. Injunction and Restraining Order. The implementation of the projects for the conversion into alternative productive
CA-G.R. SP No. 63802 which dismissed the special civil action for certiorari filed by Allied Domecq, Philippines Inc. (ADPI), herein uses of the military reservations are urgent and necessary and shall not be restrained or enjoined except by an order issued by
petitioner, for want of jurisdiction. the Supreme Court of the Philippines.
FACTS: Verily, the Court of Appeals did not err when it dismissed CA-G.R. SP No. 63802 for want of jurisdiction.
On May 8, 1996, petitioner ADPI entered into an exclusive distributorship agreement with Pedro Domecq, S.A., a corporation organized and
existing under the laws of Spain, engaged in the manufacture of wine and brandy. Under the said agreement, Pedro Domecq, S.A. granted
petitioner the sole and exclusive right to import and distribute in the Philippines various Pedro Domecq, S.A. products including Fundador
brandy until May 17, 2000. Upon its expiration, the agreement is deemed automatically extended for an indefinite period of time Padlan v. Dinglasan
Petitioner then applied for a Certificate of Registration with the Bureau of Food and Drugs (BFAD), pursuant to Department of Health G.R. No. 180321, March 20, 2013
Administrative Order No. 17, series of 1979, requiring all imported food products to be registered with the BFAD prior to their distribution in the
local markets. TOPIC: Manner of Acquiring Jurisdiction over subject matter
On June 2, 1998, BFAD wrote then Director Quintin L. Kintanar of the Bureau of Customs, requesting that entry of imported shipments of CASE NATURE:
Fundador brandy should not be allowed in the Philippines, unless the importer presents a valid Certificate of Registration issued by the Petition for review on certiorari assailing the Decision dated June 29, 2007 of the Court of Appeals (CA) in CA-G.R. CV No. 86983, and the
BFAD. The Bureau of Customs granted petitioners request and on July 13, 1998, issued Customs Memorandum Circular No. 228-098. Resolution dated October 23, 2007 denying petitioner's Motion for Reconsideration.
Cause of dispute: FACTS:
On April 12, 1999, Clark Liberty Warehouse, Inc. (Clark Liberty), herein private respondent, a duly licensed duty-free shop operating in the Elenita Dinglasan (Elenita) was the registered owner of a parcel of land designated as Lot No. 625 of the Limay Cadastre which is covered by
Clark Special Economic Zone, imported 800 cases or a total of 9,420 bottles of Fundador brandy. Transfer Certificate of Title (TCT) No. T-105602, with an aggregate area of 82,972 square meters. While on board a jeepney, Elenita’s mother,
Since the importation by respondent Clark Liberty was not covered by the BFAD Certificate of Product Registration, the Bureau of Customs Lilia Baluyot (Lilia), had a conversation with one Maura Passion (Maura) regarding the sale of the said property. Believing that Maura was a real
seized and impounded the shipment pursuant to Customs Memorandum Circular No. 228-98, in relation to Sections 101 (K) and 2530 of the estate agent, Lilia borrowed the owner’s copy of the TCT from Elenita and gave it to Maura. Maura then subdivided the property into several
Tariff and Customs Code.The imported brandy then became the subject of seizure proceedings before the District Collector of Customs of lots from Lot No. 625-A to Lot No. 625-O, under the name of Elenita and her husband Felicisimo Dinglasan (Felicisimo).
the Port of Manila, docketed as S.I. No. 99-140
Cause of dispute:
Plaintiff’s argument: Through a falsified deed of sale bearing the forged signature of Elenita and her husband Felicisimo, Maura was able to sell the lots to different
Petitioner then filed a motion to intervene in S.I. No. 99-140 alleging, among others, that it sustained damages caused by respondent Clark buyers. On April 26, 1990, Maura sold Lot No. 625-K to one Lorna Ong (Lorna), who later caused the issuance of TCT No. 134932 for the
Libertys illegal importation. However, the Bureau of Customs District Collector failed to resolve the motion. subject property under her name. A few months later, or sometime in August 1990, Lorna sold the lot to petitioner Editha Padlan for ₱4,000.00.
Thus, TCT No. 134932 was cancelled and TCT No. 137466 was issued in the name of petitioner.
Plaintiff’s prayer:
Issuance of a temporary restraining order (TRO) and a writ of preliminary injunction and demanding that the Clark Liberty cease and desist Plaintiff’s argument:
from importing, distributing, selling, or marketing Fundador brandy in the Philippines and refrain from claiming the seized shipment and Lacks of jurisdiction over her person
participating in the seizure proceedings. Petitioner insists that summons was not validly served upon her, considering that at the time summons was served, she was residing in Japan.
Petitioner contends that pursuant to Section 15, Rule 14 of the Rules of Civil Procedure, when the defendant does not reside in the Philippines
Defendant’s answer: and the subject of the action is property within the Philippines of the defendant, service may be effected out of the Philippines by personal
service or by publication in a newspaper of general circulation. In this case, summons was served only by substituted service to her mother. From the Complaint, the case filed by respondent is not simply a case for the cancellation of a particular certificate of title and the revival of
Hence, the court did not acquire jurisdiction over her person. another. The determination of such issue merely follows after a court of competent jurisdiction shall have first resolved the matter of who
between the conflicting parties is the lawful owner of the subject property and ultimately entitled to its possession and enjoyment. The action is,
Lacks of jurisdiction of the subject matter therefore, about ascertaining which of these parties is the lawful owner of the subject lot, jurisdiction over which is determined by the assessed
Also, petitioner posits that the court lacks jurisdiction of the subject matter, considering that from the complaint, it can be inferred that the value value of such lot.
of the property was only ₱4,000.00, which was the amount alleged by respondents that the property was sold to petitioner by Lorna In no uncertain terms, the Court has already held that a complaint must allege the assessed value of the real property subject of the complaint
Plaintiff’s prayer: or the interest thereon to determine which court has jurisdiction over the action. In the case at bar, the only basis of valuation of the subject
Finally, petitioner stresses that she was a buyer in good faith. It was Maura who defrauded the respondents by selling the property to Lorna property is the value alleged in the complaint that the lot was sold by Lorna to petitioner in the amount of ₱4,000.00. No tax declaration was
without their authority. even presented that would show the valuation of the subject property. In fact, in one of the hearings, respondents’ counsel informed the court
Defendant’s answer: that they will present the tax declaration of the property in the next hearing since they have not yet obtained a copy from the Provincial
CA was correct in ruling in their favor. Assessor’s Office. However, they did not present such copy.
RTC: To reiterate, where the ultimate objective of the plaintiffs is to obtain title to real property, it should be filed in the proper court having jurisdiction
Padlan is buyer in good faith over the assessed value of the property subject thereof. Since the amount alleged in the Complaint by respondents for the disputed lot is only
₱4,000.00, the MTC and not the RTC has jurisdiction over the action. Therefore, all proceedings in the RTC are null and void.
CA:
Reverse the decision of RTC and ordered the cancellation of TCT
Guy v. Court of Appeals
Basis/reason
G.R. No. 165849, December 10, 2007
The CA found that petitioner purchased the property in bad faith from Lorna. The CA opined that although a purchaser is not
expected to go beyond the title, based on the circumstances surrounding the sale, petitioner should have conducted further
TOPIC: Manner of Acquiring Jurisdiction over subject matter
inquiry before buying the disputed property. The fact that Lorna bought a 5,000-square-meter property for only ₱4,000.00 and
CASE NATURE:
selling it after four months for the same amount should have put petitioner on guard. With the submission of the Judgment in
Five (5) consolidated cases which stemmed from Civil Case No. 04-109444 filed with the Regional Trial Court (RTC), Branch 24, Manila,
Criminal Case No. 4326 rendered by the RTC, Branch 2, Balanga, Bataan, entitled People of the Philippines v. Maura
subsequently re-raffled to Branch 46 and eventually to Branch 25
Passion10 and the testimonies of respondents, the CA concluded that respondents sufficiently established that TCT No.
FACTS:
134932 issued in the name of Lorna and TCT No. 137466 issued in the name of petitioner were fraudulently issued and,
The instant controversies arose from a family dispute. Gilbert Guy is the son of Francisco and Simny Guy. Geraldine, Gladys and Grace are his
therefore, null and void.
sisters. The family feud involves the ownership and control of 20,160 shares of stock of Northern Islands Co., Inc. (Northern Islands) engaged
Motion for recon CA:
in the manufacture, distribution, and sales of various home appliances bearing the 3-D trademark.
Denied the motion
Simny and her daughters Geraldine, Gladys and Grace, as well as Northern Islands and Emilia Tabugadir, have been impleaded as
respondents in the above-entitled cases. Northern Islands is a family-owned corporation organized in 1957 by spouses Francisco and
ISSUE:
respondent Simny Guy. In November 1986, they incorporated Lincoln Continental Development Corporation, Inc. (Lincoln Continental) as a
Whether or not the Honorable Court has jurisdiction over the person of the petitioner
holding company of the 50% shares of stock of Northern Islands in trust for their three (3) daughters, respondents Geraldine, Gladys and
Whether or not the Honorable Court has jurisdiction over the subject matter of the case
Grace. Sometime in December 1986, upon instruction of spouses Guy, Atty. Andres Gatmaitan, president of Lincoln Continental, indorsed in
blank Stock Certificate No. 132 (covering 8,400 shares) and Stock Certificate No. 133 (covering 11,760 shares) and delivered them to Simny.
RULING:
Cause of dispute:
The petition is meritorious.
In 1984, spouses Guy found that their son Gilbert has been disposing of the assets of their corporations without authority. In order to protect the
Respondents filed the complaint in 1999, at the time Batas Pambansa Blg. (BP) 129, the Judiciary Reorganization Act of 1980, was already
assets of Northern Islands, Simny surrendered Stock Certificate Nos. 132 and 133 to Emilia Tabugadir, an officer of Northern Islands. The
amended by Republic Act (RA) No. 7691, An Act Expanding the Jurisdiction of the Metropolitan Trial Courts, Municipal Trial Courts, and
20,160 shares covered by the two Stock Certificates were then registered in the names of respondent sisters, thus enabling them to assume an
Municipal Circuit Trial Courts, amending for the purpose BP Blg. 129.
active role in the management of Northern Islands.
Section 1 of RA 7691, amending BP Blg. 129, provides that the RTC shall exercise exclusive original jurisdiction on the following actions:
On January 27, 2004, during a special meeting of the stockholders of Northern Islands, Simny was elected President; Grace as Vice-President
Section 1. Section 19 of Batas Pambansa Blg. 129, otherwise known as the "Judiciary Reorganization Act of 1980," is hereby amended to read
for Finance; Geraldine as Corporate Treasurer; and Gladys as Corporate Secretary. Gilbert retained his position as Executive Vice
as follows:
President. This development started the warfare between Gilbert and his sisters.
Sec. 19. Jurisdiction in civil cases. – Regional Trial Courts shall exercise exclusive original jurisdiction:
(1) In all civil actions in which the subject of the litigation is incapable of pecuniary estimation;
(2) In all civil actions which involve the title to, or possession of, real property, or any interest therein, where the assessed value of the property
involved exceeds Twenty Thousand Pesos (₱20,000.00) or for civil actions in Metro Manila, where such value exceeds Fifty Thousand Pesos On March 18, 2004, Lincoln Continental filed with the RTC, Branch 24, Manila a Complaint for Annulment of the Transfer of Shares of Stock
(₱50,000.00), except actions for forcible entry into and unlawful detainer of lands or buildings, original jurisdiction over which is conferred upon
the Metropolitan Trial Courts, Municipal Trial Courts, and Municipal Circuit Trial Courts; x x x
Section 3 of RA 7691 expanded the exclusive original jurisdiction of the first level courts, thus: against respondents, docketed as Civil Case No. 04-109444. The complaint basically alleges that Lincoln Continental owns 20,160 shares of
Section 3. Section 33 of the same law BP Blg. 129 is hereby amended to read as follows:
Sec. 33. Jurisdiction of Metropolitan Trial Courts, Municipal Trial Courts and Municipal Circuit Trial Courts in Civil Cases. – Metropolitan Trial
Courts, Municipal Trial Courts, and Municipal Circuit Trial Courts shall exercise: stock of Northern Islands; and that respondents, in order to oust Gilbert from the management of Northern Islands, falsely transferred the said
xxxx
(3) Exclusive original jurisdiction in all civil actions which involve title to, or possession of, real property, or any interest therein where the
assessed value of the property or interest therein does not exceed Twenty Thousand Pesos (₱20,000.00) or, in civil actions in Metro Manila, shares of stock in respondent sisters names. Lincoln Continental then prayed for an award of damages and that the management of Northern
where such assessed value does not exceed Fifty Thousand Pesos (₱50,000.00) exclusive of interest, damages of whatever kind, attorney's
fees, litigation expenses and costs: Provided, That in cases of land not declared for taxation purposes, the value of such property shall be
determined by the assessed value of the adjacent lots. Islands be restored to Gilbert. Lincoln also prayed for the issuance of a temporary restraining order (TRO) and a writ of preliminary mandatory
Respondents filed their Complaint with the RTC; hence, before proceeding any further with any other issues raised by the petitioner, it is
essential to ascertain whether the RTC has jurisdiction over the subject matter of this case based on the above-quoted provisions.
However, in order to determine which court has jurisdiction over the action, an examination of the complaint is essential. Basic as a hornbook injunction to prohibit respondents from exercising any right of ownership over the shares.
principle is that jurisdiction over the subject matter of a case is conferred by law and determined by the allegations in the complaint which Plaintiff’s argument:
comprise a concise statement of the ultimate facts constituting the plaintiff's cause of action. The nature of an action, as well as which court or
body has jurisdiction over it, is determined based on the allegations contained in the complaint of the plaintiff, irrespective of whether or not the
plaintiff is entitled to recover upon all or some of the claims asserted therein. The averments in the complaint and the character of the relief
On the question of forum shopping, petitioners Gilbert and Lincoln Continental contend that the acts of respondents in filing a petition
sought are the ones to be consulted. Once vested by the allegations in the complaint, jurisdiction also remains vested irrespective of whether or
not the plaintiff is entitled to recover upon all or some of the claims asserted therein. What determines the jurisdiction of the court is the nature
of the action pleaded as appearing from the allegations in the complaint. The averments therein and the character of the relief sought are the
ones to be consulted. for certiorari and mandamus in CA-G.R. SP No. 85069 and withdrawing the same and their subsequent filing of a petition for certiorari in CA-
Respondents’ Complaint narrates that they are the duly registered owners of Lot No. 625 of the Limay Cadastre which was covered by TCT No.
T-105602. Without their knowledge and consent, the land was divided into several lots under their names through the fraudulent manipulations
of Maura. One of the lots was Lot 625-K, which was covered by TCT No. 134785. On April 26, 1990, Maura sold the subject lot to Lorna. By G.R. SP No. 87104 constitute forum shopping; that respondents withdrew their petition in CA-G.R. SP No. 85069 after the Tenth Division
virtue of the fictitious sale, TCT No. 134785 was cancelled and TCT No. 134932 was issued in the name of Lorna. Sometime in August 1990,
Lorna sold the lot to petitioner for a consideration in the amount of ₱4,000.00. TCT No. 134932 was later cancelled and TCT No. 137466 was
issued in the name of petitioner. Despite demands from the respondents, petitioner refused to surrender possession of the subject property. issued a Resolution dated October 20, 2004 denying their application for a writ of preliminary injunction; that they then filed an identical petition
Respondents were thus constrained to engage the services of a lawyer and incur expenses for litigation. Respondents prayed for the RTC (a)
to declare TCT No. 137466 null and to revive TCT No. T-105602 which was originally issued and registered in the name of the respondents;
and (b) to order petitioner to pay attorney’s fees in the sum of ₱50,000.00 and litigation expenses of ₱20,000.00, plus cost of suit. in CA-G.R. SP No. 87104 seeking the same relief alleged in their petition in CA-G.R. SP No. 85069; and that by taking cognizance of the
An action "involving title to real property" means that the plaintiff's cause of action is based on a claim that he owns such property or that he
has the legal rights to have exclusive control, possession, enjoyment, or disposition of the same. Title is the "legal link between (1) a person
who owns property and (2) the property itself." "Title" is different from a "certificate of title" which is the document of ownership under the petition in CA-G.R. SP No. 87104, instead of dismissing it outright on the ground of forum shopping, the Court of Appeals committed grave
Torrens system of registration issued by the government through the Register of Deeds. While title is the claim, right or interest in real property,
a certificate of title is the evidence of such claim.
In the present controversy, before the relief prayed for by the respondents in their complaint can be granted, the issue of who between the two abuse of discretion tantamount to lack or excess of jurisdiction.
contending parties has the valid title to the subject lot must first be determined before a determination of who between them is legally entitled to Plaintiff’s prayer:
the certificate of title covering the property in question.
On January 23, 2001, almost two months following the conclusion of the 3rd Quinquennial Session, the Board of Trustees appointed the
petitioner President of AUP. During his tenure, or from November 11 to November 13, 2002, a group from the NPUM conducted an external
. The petitioners in CA-G.R. SP No. 85069 prayed that the following Orders be set aside: performance audit. The audit revealed the petitioners autocratic management style, like making major decisions without the approval or
recommendation of the proper committees, including the Finance Committee; and that he had himself done the canvassing and purchasing of
materials and made withdrawals and reimbursements for expenses without valid supporting receipts and without the approval of the Finance
(1) the Order of inhibition dated June 22, 2004 issued by the presiding judge of the RTC of Manila, Committee. The audit concluded that he had committed serious violations of fundamental rules and procedure in the disbursement and use of
funds.
The NPUM Executive Committee and the Board of Trustees decided to immediately request the services of the General Conference Auditing
Branch 24; and Service (GCAS) to determine the veracity of the audit findings. Accordingly, GCAS auditors worked in the campus from December 4 to
December 20, 2002 to review the petitioners transactions during the period from April 2002 to October 2002. On December 20, 2002, CGAS
auditors reported the results of their review, and submitted their observations and recommendations to the Board of Trustees.
Upon receipt of the CGAS report that confirmed the initial findings of the auditors on January 8, 2003, the NPUM informed the petitioner of the
(2) the Order dated July 12, 2004 issued by Branch 46 setting Gilberts application for preliminary
findings and required him to explain.
On January 15, 2003, Chairman Dayson and the NPUM Treasurer likewise informed the petitioner inside the NPUM office on the findings of the
auditors in the presence of the AUP Vice-President for Financial Affairs, and reminded him of the possible consequences should he fail to
injunction for hearing. satisfactorily explain the irregularities cited in the report. He replied that he had already prepared his written explanation.
The Board of Trustees set a special meeting at 2 p.m. on January 22, 2003. Being the Secretary, the petitioner himself prepared the agenda
and included an item on his case. In that meeting, he provided copies of the auditors report and his answers to the members of the Board of
Trustees.After hearing his explanations and oral answers to the questions raised on issues arising from the report, the members of the Board of
RTC: Trustees requested him to leave to allow them to analyze and evaluate the report and his answers. Despite a long and careful deliberation,
Complaint and the Complaint-in-Intervention are hereby DISMISSED however, the members of the Board of Trustees decided to adjourn that night and to set another meeting in the following week considering that
Basis/reason the meeting had not been specifically called for the purpose of deciding his case. The adjournment would also allow the Board of Trustees
The trial court held that Civil Case No. 04-109444 is a baseless and an unwarranted suit among family members; that based more time to ponder on the commensurate disciplinary measure to be meted on him.
on the evidence, Gilbert was only entrusted to hold the disputed shares of stock in his name for the benefit of the other family On January 23, 2003, Chairman Dayson notified the petitioner in writing that the Board of Trustees would hold in abeyance its deliberation on
members; and that it was only when Gilbert started to dispose of the assets of the familys corporations without their knowledge his answer to the auditors report and would meet again at 10:00 a.m. on January 27, 2003. Chairman Dayson indicated that some sectors in
that respondent sisters caused the registration of the shares in their respective names the campus had not been properly represented in the January 22, 2003 special meeting, and requested the petitioner as Secretary to ensure
that all sectors are duly represented in the next meeting of the Board of Trustees.
CA: In the January 27, 2003 special meeting, the petitioner sent a letter to the Board of Trustees. The members, by secret ballot, voted to remove
Affirmed the decision of RTC him as President because of his serious violations of fundamental rules and procedures in the disbursement and use of funds as revealed by
ISSUE: the special audit; to appoint an interim committee consisting of three members to assume the powers and functions of the President; and to
Whether respondents are guilty of forum shopping and that the Court of Appeals committed grave abuse of discretion tantamount to lack or recommend him to the NPUM for consideration as Associate Director for Secondary Education.
excess of jurisdiction. On January 28, 2003, the petitioner was handed inside the NPUM office a letter, together with a copy of the minutes of the special meeting held
RULING: the previous day. In turn, he handed to Chairman Dayson a letter requesting two weeks within which to seek a reconsideration, stating that he
No grave abuse of discretion amounting to lack or excess of jurisdiction needed time to obtain supporting documents because he was then attending to his dying mother.
A party is guilty of forum shopping when he repetitively avails of several judicial remedies in different courts, simultaneously or successively, all In the evening of January 28, 2003, the Board of Trustees, most of whose members had not yet left Cavite, reconvened to consider and decide
substantially founded on the same transactions and the same essential facts and circumstances, and all raising substantially the same issues the petitioners request for reconsideration. During the meeting, he made an emotional appeal to allow him to continue as President, promising
either pending in, or already resolved adversely by some other court. [6] It is prohibited by Section 5, Rule 7 of the 1997 Rules of Civil to immediately vacate his office should he again commit any of the irregularities cited in the auditors report. He added that should the Board of
Procedure, as amended, which provides: Trustees not favor his appeal, he would settle for a retirement package for him and his wife and would leave the church.
SECTION 5. Certification against forum shopping. The plaintiff or principal party shall certify under oath in the complaint or other initiatory The Board of Trustees denied the petitioners request for reconsideration because his reasons were not meritorious. Board Member Elizabeth
pleading asserting a claim for relief, or in a sworn certification annexed thereto and simultaneously filed therewith: (a) that he has not Role served the notice of the denial on him the next day, but he refused to receive the notice, simply saying Alam ko na yan.
theretofore commenced any action or filed any other claim involving the same issues in any court, tribunal, or quasi-judicial agency and, to the The petitioner later obtained a copy of the inter-school memorandum dated January 31, 2003 informing AUP students, staff, and faculty
best of his knowledge, no such other action or claim is pending therein; (b) if there is such other pending action or claim, a complete statement members about his relief as President and the appointment of an interim committee to assume the powers and duties of the President.
of the present status thereof; and (c) if he should thereafter learn that the same or similar action has been filed or is pending, he shall report On February 4, 2003, the petitioner brought his suit for injunction and damages in the RTC, with prayer for the issuance of a temporary
that fact within five (5) days therefrom to the court wherein his aforesaid complaint or initiatory pleading has been filed. restraining order (TRO), impleading AUP and its Board of Trustees, represented by Chairman Dayson, and the interim committee. His
Failure to comply with the foregoing requirements shall not be curable by mere amendment of the complaint or other initiatory pleading but complaint alleged that the Board of Trustees had relieved him as President without valid grounds despite his five-year term; that the Board of
shall be cause for the dismissal of the case without prejudice, unless otherwise provided, upon motion and hearing. The submission of a false Trustees had thereby acted in bad faith; and that his being denied ample and reasonable time to present his evidence deprived him of his right
certification or non-compliance with any of the undertakings therein shall constitute indirect contempt of court, without prejudice to the to due process.
corresponding administrative and criminal actions. If the acts of the party or his counsel clearly constitute willful and deliberate forum shopping, The suit being intra-corporate and summary in nature, the application for TRO was heard by means of affidavits. In the hearing of February 7,
the same shall be ground for summary dismissal with prejudice and shall constitute direct contempt, as well as a cause for administrative 2003, the parties agreed not to harass each other. The RTC used the mutual agreement as its basis to issue a status quo order on February
sanctions. 11, 2003.
Forum shopping is condemned because it unnecessarily burdens our courts with heavy caseloads, unduly taxes the manpower and financial In their answer with counterclaim, the respondents denied the allegations of the petitioner, and averred that he had been validly removed for
resources of the judiciary and trifles with and mocks judicial processes, thereby affecting the efficient administration of justice. [7] The primary cause; and that he had been granted ample opportunity to be heard in his defense.
evil sought to be proscribed by the prohibition against forum shopping is, however, the possibility of conflicting decisions being rendered by the Cause of dispute:
different courts and/or administrative agencies upon the same issues.[8] The dispute centers on whether the removal of the petitioner as President of respondent Adventist University of the Philippines (AUP) was
Forum shopping may only exist where the elements of litis pendentia are present or where a final judgment in one case will amount to res valid, and whether his term in that office was five years, as he insists, or only two years, as AUP insists.
judicata in the other.[9] Litis pendentia as a ground for dismissing a civil action is that situation wherein another action is pending between the Plaintiff’s argument:
same parties for the same cause of action, such that the second action is unnecessary and vexatious. The elements of litis pendentia are as THE COURT OF APPPEALS HAS DECIDED CONTRARY TO LAW AND JURISPRUDENCE WHEN IT RULED THAT THE EXTRAORDINARY
follows: (a) identity of parties, or at least such as representing the same interest in both actions; (b) identity of rights asserted and the relief WRIT OF CERTIORARI APPLIED IN THE CASE AT BAR.
prayed for, the relief being founded on the same facts; and (c) the identity of the two cases such that judgment in one, regardless of which party THE COURT OF APPEALS DECIDED A QUESTION OF SUBSTANCE IN A WAY NOT IN ACCORD WITH THE ESTABLISHED LAW AND
is successful, would amount to res judicata in the other.[10] From the foregoing, it is clear that sans litis pendentia or res judicata, there can be JURISPRUDENCE THAT ADMISSIONS, VERBAL OR WRITTEN, MADE BY A PARTY IN THE COURSE OF THE PROCEEDINGS IN THE
no forum shopping. SAME CASE, DOES NOT REQUIRE PROOF, BY REQUIRING PETITIONER BARAYUGA TO PRESENT EVIDENCE THAT HIS TERM AS
While the first element of litis pendentia identity of parties is present in both CA-G.R. SP No. 85069 and CA-G.R. SP No. 87104, however, the PRESIDENT OF AUP IS FOR FIVE (5) YEARS.
second element, does not exist. The petitioners in CA-G.R. SP No. 85069 prayed that the following Orders be set aside: THE COURT OF APPEALS DECIDED A QUESTION OF SUBSTANCE IN A WAY NOT IN ACCORD WITH LAW AND ESTABLISHED FACTS
(1) the Order of inhibition dated June 22, 2004 issued by the presiding judge of the RTC of Manila, Branch 24; and WHEN IT RULED THAT PETITIONER BARAYUGA HAS ONLY A TERM OF TWO (2) YEARS INSTEAD OF FIVE (5) YEARS AS CLEARLY
(2) the Order dated July 12, 2004 issued by Branch 46 setting Gilberts application for preliminary injunction for hearing. ADMITTED BY PRIVATE RESPONDENT AUP IN ITS ANSWER.
THE COURT OF APPEALS DECIDED A QUESTION OF SUBSTANCE IN A WAY NOT IN ACCORD WITH LAW AND JURISPRUDENCE BY
In their petition in CA-G.R. SP No. 87104, respondents prayed for the annulment of the writ of preliminary injunction issued by the RTC, Branch SOLELY RELYING ON THE CASE OF NATIONAL POWER CORPORATION v. COURT OF APPEALS, WHICH INVOLVE FACTS DIFFERENT
46 after the expiration of the TRO issued by the Tenth Division of the Court of Appeals. Evidently, this relief is not identical with the relief sought FROM THE PRESENT CASE.
by respondents in CA-G.R. SP No. 85069. Clearly, the second element of litis pendentia the identity of reliefs sought - is lacking in the two THE COURT OF APPEALS DECIDED A QUESTION OF SUBSTANCE IN A WAY NOT IN ACCORD WITH LAW AND ESTABLISHED FACTS
petitions filed by respondents with the appellate court. Thus, we rule that no grave abuse of discretion amounting to lack or excess of WHEN IT UNJUSTIFIABLY ALLOWED THE WAIVER OF NOTICE FOR THE SPECIAL MEETING OF THE BOARD OF TRUSTEES.
jurisdiction may be attributed to the Court of Appeals (Eighth Division) for giving due course to respondents petition in CA-G.R. SP No. 87104. THE COURT OF APPEALS DECIDED A QUESTION OF SUBSTANCE IN A WAY NOT IN ACCORD WITH LAW AND ESTABLISHED FACTS
WHEN IT ERRONEOUSLY CONCLUDED THAT PETITIONER BARAYUGA WAS MERELY OCCUPYING THE POSITION OF AUP
PRESIDENT IN A HOLD-OVER CAPACITY.

Barayuga v. Adventist University


Defendant’s answer:
G.R. No. 168008, August 17, 2011
PETITIONER IS NOT AN ELECTED TRUSTEE OF THE AUP BOARD, NOR WAS (HE) ELECTED AS PRESIDENT, AND AS SUCH, HE CAN
TOPIC: Manner of Acquiring Jurisdiction over subject matter
CLAIM NO RIGHT TO THE AUP PRESIDENCY, BEING TWICE DISQUALIFIED BY LAW, WHICH RENDERS MOOT AND ACAMEDIC ALL OF
CASE NATURE:
THE ARGUMENTS IN THIS PETITION.
Petition for review on certiorari
EVEN IF WE FALSELY ASSUME EX GRATIA THAT PETITIONER IS AN ELECTED TRUSTEE AND ELECTED PRESIDENT, THE TWO (2)
FACTS:
YEAR TERM PROVIDED IN AUPS BY-LAWS REQUIRED BY THE CORPORATION CODE AND APPROVED BY THE SEC IS WHAT
AUP, a non-stock and non-profit domestic educational institution incorporated under Philippine laws on March 3, 1932, was directly under the
GOVERNS THE INTRA-CORPORATE CONTROVERSY, THE AUPS ADMISSION IN ITS ANSWER THAT HE HAS A FIVE (5) YEAR TERM
North Philippine Union Mission (NPUM) of the Southern Asia Pacific Division of the Seventh Day Adventists. During the 3rd Quinquennial
BASED ON HIS INVOKED SAMPLE CONSTITUTION, BY-LAWS AND POLICY OF THE SEVENTH DAY ADVENTIST NOTWITHSTANDING.
Session of the General Conference of Seventh Day Adventists held from November 27, 2000 to December 1, 2000, the NPUM Executive
PURSUANT TO THE RULES AND SETTLED JURISPRUDENCE, THE ADMISSION IN THE ANSWER IS NOT EVEN PREJUDICIAL AT ALL.
Committee elected the members of the Board of Trustees of AUP, including the Chairman and the Secretary. Respondent Nestor D. Dayson
was elected Chairman while the petitioner was chosen Secretary.
EVEN IF WE FALSELY ASSUME, JUST FOR THE SAKE OF ARGUMENT, THAT THE PETITIONER HAD A FIVE (5) YEAR TERM AS
UNIVERSITY PRESIDENT, HE WAS NONETHELESS VALIDLY TERMINATED FOR LOSS OF CONFIDENCE, GIVEN THE NUMEROUS
ADMITTED ANOMALIES HE COMMITTED. being able to answer the results of the special audit point-by-point belied his allegation of denial of due process; that AUP was
PETITIONER CANNOT COMPLAIN THAT NOTICES OF THE BOARD MEETING WERE NOT SENT TO ALL THE TWENTY FIVE (25)
TRUSTEES OF THE AUP BOARD, SINCE: [1] AS THE AUP SECRETARY, IT WAS HE WHO HAD THE DUTY TO SEND THE NOTICES; [2]
WORSE, HE ATTENDED AND EXHAUSTIVELY DEFENDED HIS WRITTEN ANSWER IN THE AUP BOARD OF TRUSTEES MEETING, the party that stood to be injured by the issuance of the injunctive writ in the form of a demoralized administration, studentry,
THUS, WAIVING ANY NOTICE OBJECTION; [3] WORST OF ALL, HIS AFTERTHOUGHT OBJECTION IS DECEPTIVELY FALSE IN FACT.

faculty and staff, sullied reputation, and dishonest leadership; and that the assailed RTC order sowed confusion and chaos
RTC:
Granting Barayuga application for the writ of preliminary injunction
Basis/reason
because the RTC thereby chose to subordinate the interest of the entire AUP community to that of the petitioner who had been

After further hearing, the RTC issued on April 25, 2003 its controversial order, granting the petitioners application for a writ of
deemed not to have satisfied the highest ideals required of his office.

ISSUE:
preliminary injunction. It thereby resolved three issues, namely: (a) whether the special board meetings were valid; (b) whether Wheter the CA correctly ruled that the petitioner had no legal right to the position of President of AUP that could be protected by the injunctive
writ issued by the RTC.
RULING:
the conflict-of-interest provision in the By-Laws and Working Policy was violated; and (c) whether the petitioner was denied due The removal of the petitioner as President of AUP, being made in accordance with the AUP Amended By-Laws, was valid. With that, our going
into the other issues becomes unnecessary. We conclude that the order of the RTC granting his application for the writ of preliminary injunction
was tainted with manifestly grave abuse of discretion; that the CA correctly nullified and set aside the order; and that his claim for damages,
process. It found for the petitioner upon all the issues. On the first issue, it held that there was neither a written request made being bereft of factual and legal warrant, should be dismissed.

City of Dumaguete v. Philippine Ports Authority


by any two members of the Board of Trustees nor proper notices sent to the members as required by AUPs By-Laws, which
G.R. No. 168973, August 24, 2011
TOPIC: Manner of Acquiring Jurisdiction over subject matter
CASE NATURE:
omissions, being patent defects, tainted the special board meetings with nullity. Anent the second issue, it ruled that the

Petition for Review under Rule 45 of the Rules of Court assailing the Decision dated March 4, 2005 and Resolution dated June 6, 2005 of the
purchase of coco lumber from his balae (i.e., mother-in-law of his son) was not covered by the conflict-of-interest provision, for

Court Appeals in CA-G.R. SP No. 64379, which granted the Petition for Certiorari and Prohibition of respondent Philippine Ports Authority and
AUPs Model Statement of Acceptance form mentioned only the members of the immediate family and did not extend to the

set aside the Orders dated December 7, 2000 and February 20, 2001 of the Regional Trial Court (RTC), Branch 44 of the City of Dumaguete in
relationship between him and his balae. On the third issue, it concluded that he was deprived of due process when the Board

LRC Case No. N-201.


of Trustees refused to grant his motion for reconsideration and his request for additional time to produce his evidence, and

instead immediately implemented its decision by relieving him from his position without according him the treatment befitting a
FACTS:
university President.
On October 14, 1998, petitioner City of Dumaguete, through Mayor Felipe Antonio B. Remollo (Remollo), filed before the RTC an Application for

Original Registration of Title over a parcel of land with improvements, located at Barangay Looc, City of Dumaguete (subject property), under
CA:
Nullified and set aside the order of the RTC the Property Registration Decree. The application was docketed as LRC Case No. N-201.
Basis/reason
Cause of dispute:
The Republic and the respondent opposed the application, as it remains in the public domain
The CA rendered its decision nullifying the RTCs writ of preliminary injunction. It rejected the petitioners argument that Article Plaintiff’s argument:
Petitioner alleged in support of its application:
1. That the applicant, City of Dumaguete through its Honorable Mayor Felipe Antonio B. Remollo, is the owner of the land subject of this
IV, Section 3 of AUPs Constitution and By-Laws and Working Policy of the Conference provided a five-year term for him, application with all improvements and buildings comprising the Engineers Compound where it is now situated and has been in continuous
occupation and possession of the same for more than 30 years or from the year 1960 (Affidavit of Ownership executed by Felipe Antonio G.
Remollo, the City Mayor, dated August 21, 1998 herein attached as ANNEX A). The said land consist of 5,410 square meters and is situated
because the provision was inexistent. It ruled that the petitioners term of office had expired on January 22, 2003, or two years and bounded and described as shown on the plan (true and photostatic copies of the original plan marked Psu-07-006805 approved by the
Regional Technical Director of the [Department of Environment and Natural Resources] DENR, Regional Office, Cebu City herein attached as
ANNEX B) and technical descriptions attached hereto (technical description attached as ANNEX C) and made a part hereof;
2. That said land at the last assessment for taxation was assessed at P676,250, Philippine currency, with market value of P1,352,500.00,
from his appointment, based on AUPs amended By-Laws; that, consequently, he had been a mere de facto officer appointed
Philippine currency. (Declaration of Real Property with the assessed and market values attached as ANNEX D);

3.That to the best of my knowledge and belief, there is no mortgage or encumbrance of any kind whatsoever affecting said land, nor another
by the members of the Board of Trustees; and that he held no legal right warranting the issuance of the writ of preliminary person having any estate or interest therein, legal or equitable, in possession, remainder, reversion or expectancy;
4.That the land was acquired by possessory title in open, continuous, adverse occupation and possession in the concept of owner for more
than thirty years since 1960 (please refer to ANNEX A);
injunction. 5. That the land is adjoined by the following:
NorthWest
NorthEast
SouthEast
All along line 1-2-3-4-5-6-7-8-9-10 by Flores Avenue, City Road and the Dumaguete Port Road
SouthWest along line 10-1 by Plan Msi-V-20453
The CA declared that the rule on judicial admissions admitted of exceptions, as held in National Power Corporation v. Court of 8. That the land included is bounded on the West by Flores Avenue and on the North by the City Road, all public highways and on the East by
the Dumaguete Port Road, a private road made part of the Port Zone.

Appeals, where the Court held that admissions were not evidence that prevailed over documentary proof; that the petitioners Defendant’s answer:
Set aside the RTC Order
Basis/reason
The Republic of the Philippines, represented by the Director of Lands, and respondent, represented by the Office of the Government Corporate

The Court of Appeals, in its Decision dated March 4, 2005, found merit in the Petition of respondent and set aside the RTC
Counsel, filed separate Oppositions to the application for registration of petitioner. Both the Republic and respondent averred that petitioner

Orders dated December 7, 2000 and February 20, 2001. The appellate court, in its Resolution dated June 6, 2005, denied the
may not register the subject property in its name since petitioner had never been in open, continuous, exclusive, and notorious possession of

Motion for Reconsideration of petitioner


the said property for at least 30 years immediately preceding the filing of the application; and the subject property remains to be a portion of the
ISSUE:
Wether RTC has jurisdiction over the subject matter
public domain which belongs to the Republic. RULING:

RTC:
RTC has jurisdiction over the subject matter

Dismiss the application for original registration


Jurisdiction over the subject matter of a case is conferred by law and determined by the allegations in the complaint which comprise a concise
Basis/reason
The Court agrees with [herein respondent] Philippine Ports Authority that the basis of the [herein petitioners]
application for original registration of the subject lot is Section 14 of the Presidential Decree No. 1529,
statement of the ultimate facts constituting the plaintiff's cause of action.
otherwise known as the Property Registration Decree. A circumspect scrutiny of said Section readily shows
that it refers to alienable and disposable lands of the public domain as proper subjects of registration, provided
the applicant has met the other requirements such as open, continuous, exclusive and notorious possession
for at least thirty (30) years under a bona fide claim of ownership. Once vested by the allegations in the complaint, jurisdiction also remains vested irrespective of whether or not the plaintiff is entitled to recover

It having been shown by [petitioners] own evidence that the lot subject of the application for original upon all or some of the claims asserted therein.
registration is a foreshore land, and therefore not registerable (Dizon, et al. vs. Bayona, et al., 98 SCRA 942,
944), the application must be denied.
Republic v. Bantigue Point Development
Again as correctly argued by [respondent], [petitioners] reliance on Republic Act 1899 which authorizes all G.R. No. 162322, March 14, 2012
municipalities and chartered cities to undertake and carry out the reclamation by dredging, filling or other TOPIC: Manner of Acquiring Jurisdiction over subject matter
means of any foreshore lands bordering them and which confers ownership on them of the lands so reclaimed, CASE NATURE:
is misplaced, as such has never been alleged in the application. It is fundamental that a party cannot prove Rule 45 Petition for review requires this Court to address the issue of the proper scope of the delegated jurisdiction of municipal trial courts in
what it has not alleged in his complaint or application, as in this case. land registration cases
FACTS:
The admission by Engr. Dorado that there is no formal declaration from the executive branch of government or On 17 July 1997, respondent Bantigue Point Development Corporation filed with the Regional Trial Court (RTC) of Rosario, Batangas an
law passed by Congress that the land in question is no longer needed for public use or special industries x x x application for original registration of title over a parcel of land with an assessed value of ₱4,330, ₱1,920 and ₱8,670, or a total assessed value
further militates against the application. of ₱14,920 for the entire property, more particularly described as Lot 8060 of Cad 453-D, San Juan Cadastre, with an area of more or less
10,732 square meters, located at Barangay Barualte, San Juan, Batangas.
Moreover, the authority granted to municipalities and chartered cities to undertake and carry out at their own On 18 July 1997, the RTC issued an Order setting the case for initial hearing on 22 October 1997. On 7 August 1997, it issued a second Order
expense the reclamation by dredging, filling, or other means, of any foreshore lands bordering them is for the setting the initial hearing on 4 November 1997.
purpose of establishing, providing, constructing, maintaining, and repairing proper and adequate docking and Petitioner Republic filed its Opposition to the application for registration on 8 January 1998 while the records were still with the RTC.
harbor facilities as such municipalities and chartered cities may determine in consultation with the Secretary of Cause of dispute:
Finance and the Secretary of Public Works and Communications. Land registration cases
Plaintiff’s argument:
By its own evidence, [petitioner] has utilized the subject property allegedly reclaimed by it as Office of the City Engineer and First, petitioner argued that the lower court failed to acquire jurisdiction over the application, because the RTC set the date and hour of the
not as docking and harboring facilities. [Petitioner] has failed to show that such reclamation was undertaken by it in initial hearing beyond the 90-day period provided under the Property Registration Decree
consultation with the Secretary of Finance and the Secretary of Public Works and Communications. Second, petitioner contended that since the selling price of the property based on the Deed of Sale annexed to respondents application for
Motion for reconsideration RTC: original registration was ₱160,000, the MTC did not have jurisdiction over the case. Under Section 34 of the Judiciary Reorganization Act, as
Denied the motion for reconsideration amended, the MTCs delegated jurisdiction to try cadastral and land registration cases is limited to lands, the value of which should not
Basis/reason exceed ₱100,000.
RTC:
RTC Clerk of Court transmitted motu proprio the records of the case to the MTC
A thorough review and perusal of the disputed order dated September 7, 2000 and December 7, 2000, whereby this Court Basis/reason
The assessed value of the property was allegedly less than ₱100,000.

dismissed [petitioners] petition for registration of Lot No. 1, Dumaguete Cadastre, and later set aside the Order of September 7,

MTC:
2000, shows that there was honest mistake in declaring said lot 1, as a shoreline. Indeed, the adjoining lots are already titled Awarded the land to Bantigue Point Development
Basis/reason
MTC entered an Order of General Default and commenced with the reception of evidence. Among the documents presented by
and bounded by a City Road. It is not bounded by a sea. The Court wants to correct this error in its findings on the September respondent in support of its application are Tax Declarations, a Deed of Absolute Sale in its favor, and a Certification from the
Department of Environment and Natural Resources (DENR) Community Environment and Natural Resources Office (CENRO)
of Batangas City that the lot in question is within the alienable and disposable zone. Thereafter, it awarded the land to
7, 2000 Order, that Lot No. 1 is situated on the shoreline of Dumaguete City. The Court simply committed an oversight on the respondent Corporation

CA:
Denied the appeal
petitioners evidence that the lot in question is a foreshore land x x x when in fact it is not. And it is for this reason that the court
Basis/reason
Acting on an appeal filed by the Republic, the CA ruled that since the former had actively participated in the proceedings before
the lower court, but failed to raise the jurisdictional challenge therein, petitioner is thereby estopped from questioning the
reconsidered and set aside said September 7, 2000 Order, to correct the same while it is true that said September 7, 2000 jurisdiction of the lower court on appeal. The CA further found that respondent Corporation had sufficiently established the
latters registrable title over the subject property after having proven open, continuous, exclusive and notorious possession and
occupation of the subject land by itself and its predecessors-in-interest even before the outbreak of World War II.
Order had attained its finality, yet this Court cannot in conscience allow injustice to perpetuate in this case and that hearing on
ISSUE:
Wether or not the Municipal Trial Court failed to acquire jurisdiction over the application for original registration of land title
the merits must proceed to determine the legality and truthfulness of its application for registration of title.
RULING:
CA: The Municipal Trial Court propely acquired jurisdiction over the case
The delegated jurisdiction of the MTC over cadastral and land registration cases is indeed set forth in the Judiciary Reorganization Act, which defect that may be raised as an objection anytime even after arraignment, the respondent judge did not err in granting the motion to dismiss
provides: based on this ground. As basis, they cite the case of Villa v. Ibañez, et al.[40] where we held, viz:
Sec. 34. Delegated Jurisdiction in Cadastral and Land Registration Cases. - Metropolitan Trial The defendant had pleaded to an information before he filed a motion to quash, and it is contended that by his plea he waived all objections to
Courts, Municipal Trial Courts, and Municipal Circuit Trial Courts may be assigned by the Supreme Court to the informations. The contention is correct as far as formal objections to the pleadings are concerned. But by clear implication, if not by
hear and determine cadastral or land registration cases covering lots where there is no controversy or express provision of section 10 of Rule 113 of the Rules of Court (now Section 9 of Rule 117), and by a long line of uniform decisions,
opposition, or contested lots where the value of which does not exceed One hundred thousand pesos questions of want of jurisdiction may be raised at any stage of the proceeding. Now, the objection to the respondent’s actuations goes to the
(₱100,000.00), such value to be ascertained by the affidavit of the claimant or by agreement of the respective very foundation of the jurisdiction. It is a valid information signed by a competent officer which, among other requisites, confers jurisdiction on
claimants if there are more than one, or from the corresponding tax declaration of the real property. Their the court over the person of the accused and the subject matter of the accusation. In consonance with this view, an infirmity in the information
decision in these cases shall be appealable in the same manner as decisions of the Regional Trial Courts. (As cannot be cured by silence, acquiescence, or even by express consent.[41] mphasis supplied)
amended by R.A. No. 7691) (Emphasis supplied.) The case of Villa is authority for the principle that lack of authority on the part of the filing officer prevents the court from acquiring jurisdiction
over the case. Jurisdiction over the subject matter is conferred by law while jurisdiction over the case is invested by the act of plaintiff and
Thus, the MTC has delegated jurisdiction in cadastral and land registration cases in two instances: first, where there is no controversy or attaches upon the filing of the complaint or information.[42] Hence, while a court may have jurisdiction over the subject matter, like a violation
opposition; or, second, over contested lots, the value of which does not exceed ₱100,000. of the SSS Law, it does not acquire jurisdiction over the case itself until its jurisdiction is invoked with the filing of the information.

The case at bar does not fall under the first instance, because petitioner opposed respondent Corporations application for registration on 8
January 1998. Lazo v. Republic Surety
However, the MTC had jurisdiction under the second instance, because the value of the lot in this case does not exceed ₱100,000. G.R. No. L-27365, January 30, 1970
Contrary to petitioners contention, the value of the land should not be determined with reference to its selling price. Rather, Section 34 of the
Judiciary Reorganization Act provides that the value of the property sought to be registered may be ascertained in three ways: first, by the TOPIC: Manner of Acquiring Jurisdiction: Over the Issues
affidavit of the claimant; second, by agreement of the respective claimants, if there are more than one; or, third, from the corresponding tax
declaration of the real property. In this case, the value of the property cannot be determined using the first method, because the records are FACTS: Petitioner Lazo is the guarantor of co-petitioner Robles for a loan obtained from the Philippine Bank of Commerce. Lazo executed a
bereft of any affidavit executed by respondent as to the value of the property. Likewise, valuation cannot be done through the second method, real estate mortgage in favor of the defendant Republic Surety & Insurance Co., Inc who was co-debtor in the same loan. Robles failed to pay
because this method finds application only where there are multiple claimants who agree on and make a joint submission as to the value of the despite several extensions until the loan was transferred to the Surety who foreclosed extra-judicially the mortgage on July 1, 1958. Lazo
property. Here, only respondent Bantigue Point Development Corporation claims the property. claims the foreclosure as invalid because plaintiff Jose Robles had paid on the mortgage loan the sum of P13,466.36; he continued to make
The value of the property must therefore be ascertained with reference to the corresponding Tax Declarations submitted by respondent other payments, aggregating P17,250.00.
Corporation together with its application for registration. From the records, we find that the assessed value of the property is ₱4,330, ₱1,920
and ₱8,670, or a total assessed value of ₱14,920 for the entire property. Based on these Tax Declarations, it is evident that the total value of CAUSE OF THE DISPUTE: This is an appeal by the defendants from the decision of the CFI of Manila
the land in question does not exceed ₱100,000. Clearly, the MTC may exercise its delegated jurisdiction under the Judiciary Reorganization
Act, as amended. ARGUMENTS

People of the Philippines v. Hon. Garfin PLAINTIFF:


G.R. No. 153176, March 29, 2004
Plaintiff's prayer: Lazo filed a petition for the defendant company to render an accounting of the payments thus made so that if the loan had
TOPIC: Manner of Acquiring Jurisdiction: Over the Case been paid in full then the real estate mortgage should be cancelled; otherwise the plaintiffs should be allowed to pay, by way of legal
redemption, whatever Balance still remained.
CASE NATURE: SPECIAL CIVIL ACTION in the Supreme Court. Certiorari and Mandamus.
RESPONDENT'S ANSWER: filed a motion to dismiss arguing that the complaint did not state a cause of action and that the claim or demand
FACTS: private respondent was charged with violation of RA No. 8282, otherwise known as the "Social Security Act." The corresponding set forth therein had already prescribed.
information was filed by Prosecutor Tolentino. Garfin was arraigned and pleaded not guilty. Three days thereafter, the accused filed a motion to
dismiss on the ground that the information was filed without the prior written authority or approval of the city prosecutor as required under RTC: went entirely out of the issues submitted to it and chose to decide the case on a point which was not at all litigated, the validity of the
Section 4, Rule 112 of the Revised Rules of Court. The Court thus the case dismissed for lack of jurisdiction. mortgage foreclosure. Consequently, the court concluded, the real estate mortgage in favor of said defendant was extinguished, and the
foreclosure thereof was a nullity.
CAUSE OF THE DISPUTE: the case of People vs. Serafin Saballegue was dismissed for lack of jurisdiction. Hence, this petition
CA: X
ARGUMENTS
ISSUE: WON the trial court erred in deciding the case on an issue not raised in the complaint
PLAINTIFF: In its MR, the People contends that Prosecutor Tolentino, having been duly designated to assist the City Prosecutor in the
investigation and prosecution of all SSS cases by the Regional State prosecutor as alter ego of the Secretary of Justice in Region V, then that RULING: Yes.
authority may be given to him other than the City Prosecutor. The actuation of the trial court was not legally permissible especially because the theory on which it proceeded involved factual considerations
neither touched upon the pleadings nor made the subject of evidence at the trial. Rule 6, Section 1, is quite explicit in providing that "pleadings
Plaintiff's prayer: Petitioner seeks to declare as null and void the Orders issued by the RTC which dismissed for lack of jurisdiction the case of are the written allegations of the parties of their respective claims and defenses submitted to the court for trial and judgment." This rule has
People vs. Serafin Saballegue and denied petitioner’s motion for reconsideration. been consistently applied and adhered to by the courts.
The subject matter of any given case is determined ... by the nature and character of the pleadings submitted by the parties to the court for trial
PRIVATE RESPONDENT'S ANSWER: 1) the instant petition was filed out of time; 2) the special State Prosecutor is only authorized to conduct and judgment. Belandres vs. Lopez Sugar Central Mill Co., Inc., 97 Phil. 100, 103.
preliminary investigation and prosecution of SSS cases and not to sign the information; and 3) the City Prosecutor did not expressly inhibit
himself from handling SSS cases nor signing the information. It is a fundamental principle that judgments must conform to both the pleadings and the proof, and must be in accordance with the theory of the
action upon which the pleadings were framed and the case was tried; that a party can no more succeed upon a case proved. but not alleged,
RTC: dismissed for lack of jurisdiction the case of People vs. Serafin Saballegue and denied petitioner’s motion for reconsideration. than upon one alleged but not proved." (Ramon v. Ortuzar, 89 Phil. 730, 742)

CA: X It is a well-known principle in procedure that courts of justice have no jurisdiction or power to decide a question not in issue." (Lim Toco vs. Go
Pay, 80 Phil. 166)
ISSUE: Whether or not the criminal case was properly dismissed for lack of jurisdiction
A judgment going outside the issues and purporting to adjudicate something upon which the parties were not heard, is not merely irregular, but
RULING: Yes. In sum, we hold that, in the absence of a directive from the Secretary of Justice designating State Prosecutor Tolentino as extrajudicial and invalid." (Salvante v. Cruz, 88 Phil. 236, 244.)
Special Prosecutor for SSS cases or a prior written approval of the information by the provincial or city prosecutor, the information in Criminal The parties here went to court and presented their respective sides on the premise, admitted by both, that the mortgage was valid and
Case No. RTC 2001-0597 was filed by an officer without authority to file the same. As this infirmity in the information constitutes a jurisdictional subsisting. Evidence, therefore, to establish such premise was unnecessary and uncalled for. Indeed, it was for that reason and because in any
defect that cannot be cured, the respondent judge did not err in dismissing the case for lack of jurisdiction. event the record of this case, particularly with respect to the actuations of the parties after the mortgage was foreclosed, shows with
overwhelming preponderance that the said mortgage had not been extinguished
Discussion: After considering the respective arguments raised by the parties, the Court believes and so resolves that the Information has not Redemption issue
been filed in accordance with Section 4, par. 3 of Rule 112 of the 2000 Rules on Criminal Procedure, thus: The plaintiffs rely on Rule 39, Section 34.1 In this connection Section 9 of Act No. 3135, as amended, may also be cited.
‘Rule 112, Section 4 x x x x x x Implicit in the application of these provisions is the premise that the period for redemption of the property sold on execution (on extrajudicial
No complaint or information may be filed or dismissed by an investigating prosecutor without the prior written authority or approval of the foreclosure of mortgage in the present case) has not yet expired. For if the right to redeem has been lost it stands to reason that there is no
provincial or city prosecutor or chief state prosecutor or the Ombudsman or his deputy.’ redemption price to speak of, to which the rents received by the purchasers are to be applied or credited.3
Expresio unius est exclusio alterius. The plaintiffs' position is that since the sheriff's certificate of sale was recorded in the office of the Register of Deeds for Manila on March 28,
The Information will readily show that it has not complied with this rule as it has not been approved by the City Prosecutor. 1963, the one-year period of legal redemption had not yet expired when the action was commenced on December 12 of the same year.
There are, however, certain circumstances peculiar to this case which take it out of the operation of the rule concerning registration in this
The Regional State Prosecutor is not the alter ego of the Secretary of Justice but a mere subordinate official and if ever the former files cases, regard.
it is by virtue of a delegated authority by the Secretary of Justice. Potestas delegada non potesta delegare (sic) – what has been delegated There is, to begin with, the categorical statement in the certificate of sale that "the period of redemption of the said property sold will expire on
cannot be redelegated. The Regional State Prosecutor is clearly vested only with the power of administrative supervision. As administrative the 2nd day of July, 1959." Then there is the fact that no lien or encumbrance, right or claim of any person, other than the mortgage in question,
supervisor, he has no power to direct the city and provincial prosecutors to inhibit from handling certain cases. appeared on the transfer certificate of title of the plaintiff spouses covering the mortgaged property, such that when the defendant company
obtained a new transfer certificate in its name on March 28, 1963, the same was entirely clean. In other words, no third parties who might have
Private respondent and the OSG take the position that the lack of prior authority or approval by the city or provincial prosecutor or chief state an interest in the property, either as possible redemptioners or otherwise, had to be protected by due notice of the sale through its registration.
prosecutor is an infirmity in the information that prevented the court from acquiring jurisdiction over the case. Since lack of jurisdiction is a Second, Beginning July 1958, immediately after the foreclosure sale, the plaintiffs — in some instances in the joint names of Jose Robles and
Felix Lazo and in other instances in the name of Jose Robles alone — started paying rents on the property to the defendant company,
indicating that the former owners, while remaining in occupancy, did so in the concept of tenants. Plaintiff's prayer: Mercader prayed in his complaint that DBP respect the lease period and reimbursement for the improvements and for the
The plaintiffs having reneged on all their repeated promises, the defendant company finally consolidated its title to the property as purchaser at annotation of their interest in the TCT.
the foreclosure sale on March 28, 1963, and obtained the corresponding transfer certificate of title. That was almost five years after the said
sale. DEFENDANT'S ANSWER:
It is clear, in the light of the facts and circumstances above set forth, that the parties had abandoned entirely the concept of legal redemption in
this case and converted it into one of conventional redemption, in which the only governing factor was the agreement between them.Buce vs. RTC: The trial court ruled in favor of the Mercaders.
Court of Appeals 1. It found that the "DBP had unnecessarily and unjustifiably made Mercader understand that his second option [lease-purchase] would be
more or less approved, except that the approval will come from Manila."
2. It also believed "quite firmly" that the "DBP could not have escaped having a foreknowledge of the existence of the prior unrecorded lease"
as the "possession and cultivation of Bernardo Mercader xxx [was] a matter of open, notorious and public knowledge in the area."
332 SCRA 151 , May 12, 2000 3. It first acknowledged that it is a "court of equity and not merely a court of law" and the "DBP is not authorized to keep real propert[y] longer
than ten years or so.”
TOPIC: Manner of Acquiring Jurisdiction: Over the Issues
CA: It found that the trial court erred in treating the lease-purchase option as a controversial issue considering that it was "outside the parties'
CASE NATURE: SPECIAL CIVIL ACTION in the Supreme Court. Certiorari. pleadings." But invoking the Supreme Court's decision in Castro v. Court of Appeals in that "the improvements introduced [into the mortgaged
property] are to be considered so incorporated [in the mortgage] only if so owned by the mortgagor," the Court of Appeals declared that the
FACTS: Petitioner leased a 56-square meter parcel of land for a period of fifteen years to commence on 1 June 1979 and to end on 1 June improvements introduced on Lot No. 2985 had been improperly included in the foreclosure sale since they were not owned by the mortgagors.
1994 "subject to renewal for another ten (10) years, under the same terms and conditions." Petitioner then constructed a building and paid the But since the improvements were already included in the foreclosure sale and the MERCADERs continued the possession and collection of
required monthly rental which periodically increased. income from the lot, the Court of Appeals, as already earlier adverted to, reversed and set aside the appealed judgment. It entered a new one
declaring that the MERCADERs were not entitled to any compensation from the DBP. It also ordered the MERCADERs to immediately turn
CAUSE OF THE DISPUTE: In 1992, the rent was again increased but Buce only tendered checks for a lesser amount which Tiongco, the over the possession of the lot to the DBP.
owner, refused to accept.
ISSUE: WON the CA erred
ARGUMENTS
RULING: This Court agrees with the MERCADERs and finds that the Court of Appeals erred in disregarding as material the lease-purchase
PLAINTIFF: (decision by the CA) by ordering her to vacate the premises, the Appellate Court went beyond the bounds of its authority because option on the ground that it was not raised in the pleadings. If the Court of Appeals adverts to the lack of reference to the lease-purchase option
the case she filed before the RTC was for "Specific Performance" not unlawful detainer. The power to order the lessee to vacate the leased in the initiatory pleadings, this can be simply explained by the fact that the trial court only took cognizance thereof when it became an integral
premises is lodged in another forum. Additionally, private respondents did not pray for the ejectment of petitioners from the leased premises in component of the pre-trial proceedings. That is why the lease-purchase option was included firstly, in the pre-trial order as one of the issues to
their Answer with Counterclaim; well-settled is the rule that a court cannot award relief not prayed for in the complaint or compulsory be resolved at trial and secondly, in the supplemental pleading subsequently filed by the MERCADERs on 7 November 1985. As a
counterclaim. supplemental pleading, it served to aver supervening facts which were then not ripe for judicial relief when the original pleading was filed. As
such, it was meant to supply deficiencies in aid of the original pleading, and not to dispense with the latter. Hence, it was patently erroneous for
Plaintiff's prayer: (filed with the RTC) A prayer for consignation and she prayed that private respondents be ordered to accept the rentals in the Court of Appeals to pronounce that the lease-purchase option was not raised in the pleadings. The DBP was even quite aware and
accordance with the lease contract and to respect the lease of fifteen years, which was renewable for another ten years, at the rate of P200 a knowledgeable of the supplemental pleading because it filed an opposition thereto.
month. The records however reveal that the trial court did not promptly rule on the motion to admit the supplemental pleading. And during trial, the trial
court also failed to rule on the prompt objection interposed by the DBP’s counsel to the MERCADERs’ introduction of evidence relative to said
DEFENDANT'S ANSWER: The increase was pursuant to Republic Act No. 877 or the Rent Control Act and that the phrase in the lease lease-purchase option. But undisputed is the trial court's eventual admission in open court of the MERCADERs’ supplemental pleading
contract authorizing renewal for another ten years does not mean automatic renewal; rather, it contemplates a mutual agreement between the The records also show that not only did the DBP’s counsel began to rigorously cross-examine Bernardo Mercader on the lease-purchase
parties. option, he also subjected his witness Mr. Ruben Carpio, then Chief of the Collection Department, DBP to an intensive direct examination
covering said subject matter. He also offered as evidence the DBP’s letter indicating the three options to the MERCADERs as Exhibit "1" and
RTC: RTC declared the lease contract automatically renewed for ten years and considered as evidence thereof (a) the stipulations in the the lease-purchase option contained therein as Exhibit "1-A."36
contract giving the lessee the right to construct buildings and improvements and (b) the filing by petitioner of the complaint almost one year The DBP is undoubtedly estopped from questioning the trial court’s inclusion of the lease-purchase option as a controversial issue. This action
before the expiration of the initial term of fifteen years. It then fixed the monthly rent. The RTC held that the continuous increase of rent from of the trial court finds anchor on Section 4, Rule 20 of the Rules of Court which reads:
P200 to P250 then P300, P400 and finally P1,000 caused "an inevitable novation of their contract." Section 4. Record of pre-trial results. -- After the pre-trial the court shall make an order which recites the action taken at the conference, the
amendments allowed to the pleadings, and the agreements made by the parties as to any of the matters considered. Such order shall limit the
CA: Court of Appeals reversed the decision of the RTC, and ordered petitioner to immediately vacate the leased premises on the ground that issues for trial to those not disposed of by admissions or agreements of counsel and when entered controls the subsequent course of the
the contract expired on 1 June 1994 without being renewed and to pay the rental arrearages at the rate of P1,000 monthly. According to the action, unless modified before trial to prevent manifest injustice.
Court of Appeals, it is unclear as to who may exercise the option to renew. The stipulation allowing the construction of a building and other Indeed, the pre-trial is primarily intended to make certain that all issues necessary to the disposition of a case are properly raised. The purpose
improvements and the fact that the complaint was filed a year before the expiration of the contract are not indicative of automatic renewal. It is to obviate the element of surprise, hence, the parties are expected to disclose at the pre-trial conference all issues of law and fact which they
applied the ruling in Fernandez v. Court of Appeals13 [166 SCRA 577 (1988)] that without a stipulation that the option to renew the lease is intend to raise at the trial, except such as may involve privileged or impeaching matter.37 [See De la Paz v. Panis, 245 SCRA 242, 248-249
solely for the benefit of one party any renewal of a lease contract must be upon the agreement of the parties. [1995], citing Permanent Concrete Products, Inc. v. Teodoro, 26 SCRA 332, 336 [1968].] In the case at bar, the pre-trial order included as
The Court of Appeals denied petitioner’s motion for reconsideration. integral to the complete adjudication of the case the issue of whether the MERCADERs can demand specific performance from the DBP
relative to the lease-purchase option. Thus, the element of surprise that the provision on pre-trial attempts to preclude was satisfied.
ISSUE: WON the CA erred in its ruling ordering petitioner to vacate the premises Assuming arguendo that the MERCADERs failed to file the supplemental pleading, evidence relative to the lease-purchase option may be
legitimately admitted by the trial court in conformity with Section 5, Rule 10 of the Rules of Court which states:
RULING: Yes. After the lease terminated on 1 June 1994 without any agreement for renewal being reached, petitioner became subject to Section 5. Amendment to conform to or authorize presentation of evidence. -- When issues not raised by the pleadings are tried by express or
ejectment from the premises. It must be noted, however, that private respondents did not include in their Answer with Counterclaim a prayer for implied consent of the parties, they shall be treated in all respects, as if they had been raised in the pleadings. Such amendment of the
the restoration of possession of the leased premises. Neither did they file with the proper Metropolitan Trial Court an unlawful detainer suit pleadings as may be necessary to cause them to conform to the evidence and to raise these issues may be made upon motion of any party at
against petitioner after the expiration of the lease contact. any time, even after judgment; but failure so to amend does not affect the result of the trial of these issues. If evidence is objected to at the trial
Moreover, the issues agreed upon by the parties to be resolved during the pre-trial were the correct interpretation of the contract and the on the ground that it is not within the issues made by the pleadings, the court may allow the pleadings to be amended and shall do so freely
validity of private respondents’ refusal to accept petitioner’s payment of P400 as monthly rental. They later limited the issue to the first, i.e., the when the presentation of the merits of the action will be subserved thereby and the objecting party fails to satisfy the court that the admission of
correct interpretation of the contract.The issue of possession of the leased premises was not among the issues agreed upon by the parties or such evidence would prejudice him in maintaining his action or defense upon the merits. The court may grant a continuance to enable the
threshed out before the court a quo. Neither was it raised by private respondents on appeal. objecting party to meet such evidence. mphasis supplied)
This provision envisions two scenarios -- first, when evidence is introduced on an issue not alleged in the pleadings and no objection was
As correctly contended by the petitioner, the Court of Appeals went beyond the bounds of its authority when after interpreting the questioned interjected and second, when evidence is offered again, on an issue not alleged in the pleadings but this time an objection was interpolated.
provision of the lease contract in favor of the private respondents it proceeded to order petitioner to vacate the subject premises. We are concerned with the second scenario. In Co Tiamco v. Diaz, the Court held that "when evidence is offered on a matter not alleged in the
pleadings, the court may admit it even against the objection of the adverse party, where the latter fails to satisfy the court that the admission of
the evidence would prejudice him in maintaining his defense upon the merits, and the court may grant him a continuance to enable him to meet
Mercader v. DBP the new situation created by the evidence. Of course, the court, before allowing the evidence, as a matter of formality, should allow an
G.R. No. 130699, May 12, 2000 amendment of the pleading, xxx And, furthermore, where the failure to order an amendment does not appear to have caused surprise or
prejudice to the objecting party, it may be allowed as a harmless error. Well-known is the rule that departures from procedure may be forgiven
TOPIC: Manner of Acquiring Jurisdiction: Over the Issues where they do not appear to have impaired the substantial rights of the parties."
the Court reinforces the Co Tiamco ruling on the application of Section 5, Rule 10 of the Rules of Court in this wise:
CASE NATURE: PETITION for review on certiorari of a decision of the Court of Appeals. The failure of a party to amend a pleading to conform to the evidence adduced during trial does not preclude adjudication by the court on the
basis of such evidence which may embody new issues not raised in the pleadings. Although, the pleading may not have been amended to
FACTS: conform to the evidence submitted during trial, judgment may nonetheless be rendered, not simply on the basis of the issues alleged but also
on the issues discussed and the assertions of fact proved in the course of the trial. The court may treat the pleading as if it had been amended
CAUSE OF THE DISPUTE: Mercader leased for 20 years a property owned by Manreal but the property was foreclosed by DBP because to conform to the evidence, although it had not been actually amended. xxx Clearly, a court may rule and render judgment on the basis of the
Manreal defaulted on his loan with the bank. Mercader had already introduced improvements on the property. evidence before it even though the relevant pleading had not been previously amended, so long as no surprise or prejudice is thereby caused
to the adverse party. Put a little differently, so long as the basic requirements of fair play had been met, as where the litigants were given full
ARGUMENTS opportunity to support their respective contentions and to object to or refute each other's evidence, the court may validly treat the pleadings as
if they had been amended to conform to the evidence and proceed to adjudicate on the basis of all the evidence before it.
PLAINTIFF: The MERCADERs assert that in issuing the challenged decision, the Court of Appeals contravened Section 4, Rule 20 and Section As already enunciated, the DBP was not and would not be prejudiced by the incorporation of the lease-purchase option as one of the
5, Rule 10 of the Rules of Court by holding that the trial court should not have taken cognizance of the lease-purchase option as a controversial controverted issues. Moreover, it had been afforded ample opportunity to refute and object to the evidence germane thereto, thus, the
issue since it was not raised in the pleadings. They maintain that the trial court correctly took cognizance of the lease-purchase option because rudiments of fair play had been properly observed.
it was part and parcel of the pre-trial stages, the determination of which will prevent future litigation thereon.
Since we agree with the MERCADERs’ contention that the Court of Appeals contravened Section 4, Rule 20 and Section 5, Rule 10 of the As the sheriff’s return in the present case does not contain any statement with regard to the impossibility of personal service the same is
Rules of Court in promulgating the questioned decision, we have to grant their prayer to refer the matter back to said court for a determination patently defective and so the presumption of regularity in the performance of official functions will not lie.[35]
of the question of whether the lease-purchase option was already consummated and for a complete ascertainment of the rights and obligations Nevertheless, we still hold that jurisdiction was validly acquired by the trial court. Although the substituted service upon him of summons was
of the parties. defective, said defect was cured by his voluntary appearance.
As the records of this case disclose, after private respondent moved for the execution of the trial court’s decision, petitioner filed a motion for a
Dispositive Portion: Petition is GRANTED DUE COURSE. Resolution of CA are set aside. The case is REFERRED BACK to the Court of re-setting of the court’s hearing thereon. In Flores v. Zurbito,[37] we held that an appearance in whatever form without expressly objecting to
Appeals for a determination of whether the lease-purchase option was consummated with the end view of ascertaining the rights and the jurisdiction of the court over the person, is a submission to the jurisdiction of the court over the person of the defendant or respondent, thus:
obligations of the parties. A voluntary appearance is a waiver of the necessity of a formal notice. An appearance in whatever form, without expressly objecting to
the jurisdiction of the court over the person, is a submission to the jurisdiction of the court over the person. While the formal method of entering
an appearance in a cause pending in the courts is to deliver to the clerk a written direction ordering him to enter the appearance of the person
who subscribes it, an appearance may be made by simply filing a formal motion, or plea or answer. This formal method of appearance is not
Cesar v. Hon. Ricafort-Bautista necessary. He may appear without such formal appearance and thus submit himself to the jurisdiction of the court. He may appear by
G.R. No. 136415, October 31, 2006 presenting a motion, for example, and unless by such appearance he specifically objects to the jurisdiction of the court, he thereby gives his
assent to the jurisdiction of the court over his person.[38]
TOPIC: Manner of Acquiring Jurisdiction: Over the Person or Party: Plaintiff Hence, in this case, petitioner’s filing of a Motion for Re-setting of the Hearing effectively cured the defect of the substituted service of
summons. Petitioner’s insistence of lack of jurisdiction over his person is utterly lacking in any legal basis.
CASE NATURE: PETITION for review on certiorari of a decision of the Court of Appeals.

FACTS: After the filing of the complaint, summons9 was issued to petitioner and this was served by Sheriff Juan C. Marquez. As petitioner HERRERA-FELIX VS. COURT OF APPEALS
failed to file his answer to the complaint, private respondent moved that he be declared in default.This motion was favorably acted upon by CITATION: G.R. No. 143736. August 11, 2004.
public respondent TOPIC: JURISDICTION OVER THE PERSON OF THE DEFENDANT; BY VOLUNTARY APPEARANCE

CAUSE OF THE DISPUTE: Petitioner's failure to pay the construction materials it purportedly purchased under a credit line extended by FACTS
private respondent Specified Materials Corporation. Original complainant:
St. Joseph Resource Development, Inc. (now respondent)
ARGUMENTS
Complaint:
PLAINTIFF:Petitioner argues that since the trial court never acquired jurisdiction over his person. He claims that the person who allegedly Sum of money against the Spouses Restituto and Ofelia Felix with a prayer for a writ of preliminary attachment.
received the summons on his behalf, and who was identified in the sheriff's return as Arsenio Robles, was not his employee. He adds that when
he conducted an inquiry, he found out that Robles was a native of Batangas and was merely peddling mango seedlings within the vicinity of his Cause of action:
office when the summons was served. He also maintains that had he been given the opportunity to present his defense, he would have shown During the period from November 16, 1992 to December 14, 1992, the Felix Spouses purchased from the respondent tubs of assorted fish.
that his obligation to private respondent is less than the amount as established by the trial court. Balance payable P1,132,065.50.
RTC
Plaintiff's prayer: seeks the annulment of the decision respondent Honorable Helen Ricafort-Bautista of the RTC of Parañaque City Decision in favor of St. Joseph Resource Development, Inc. RTC also granted the writ of preliminary attachment on a bond of P1,132,065.50
(the amount of balance).
RESPONDENT’S ANSWER: Private respondent retorts that petitioner's insistence that the court a quo did not acquire jurisdiction over him is
belied by the fact that petitioner had actual knowledge of all the proceedings since he was furnished with all the copies of the pleadings and Copies of the writ of preliminary attachment, summons and complaint were served to the spouses at their residence, through the
court orders. Private respondent points out that the Motion to Admit Amended Complaint and the Amended Complaint were personally served sister of Ofelia Herrera-Felix, Ma. Luisa Herrera. (Ofelia was out of the country)
on petitioner himself as shown by his signatures appearing thereon. Moreover, private respondent is of the view that the sheriff who served the
summons upon petitioner enjoys the presumption of regularity in the performance of duty – a presumption which petitioner was unable to Spouses Felix prayed for an extension of time to file their answer – GRANTED!
overcome. Spouses Felix failed to file their answer. So St. Joseph Resource Development, Inc. filed a motion to declare said spouses in default –
GRANTED!
RTC: As petitioner failed to file his answer to the complaint, private respondent moved that he be declared in default. This motion was favorably
acted upon by public respondent through the Order dated 14 March 1997, and private respondent was able to present its evidence. A copy of the resolution was sent to and received by the counsel of the Felix Spouses through registered mail. The copy of the
decision addressed to the spouses was returned to the court after two notices for having been "Unclaimed." However, then counsel
CA: Following the denial of its Motion to Set Aside Decision, petitioner filed before the Court of Appeals a Petition for Annulment of Judgment, for the Felix Spouses received his copy of the decision.
Preliminary Injunction with Prayer for Temporary Restraining Order.[16] This petition was dismissed for “failure to attach an affidavit of merit as
well as the affidavits of witnesses or document supporting the defense. No appeal was made. The motion for a writ of execution was filed and a copy was served to the spouses by registered mail but they failed to
Petitioner then filed a motion for reconsideration but this was denied by the Court of Appeals because an annulment for judgment is proper only oppose the motion.
when there are no remedies available.
The court thereafter issued an order granting the motion and directing the issuance of a writ of execution. The counsel for the Felix Spouses
ISSUE: WON the court a quo acquired jurisdiction over the person of the petitioner by virtue of the substituted service of summons effected by received a copy of the said order.
sheriff Marquez.
Personal properties were levied and sold at a public auction. St. Joseph Resource Development, Inc. won as the highest bidder.
RULING: It is fundamental that courts acquire jurisdiction over the plaintiff once the complaint is filed. On the other hand, there are two ways
through which jurisdiction over the defendant or respondent is acquired – either through the service of summons upon them or through their COURT OF APPEALS
voluntary appearance in court. In the case of Avon Insurance PLC v. Court of Appeals,[28] we discussed the function of summons in court The wife, Ofelia Herrera-Felix, filed a petition under Rule 47 of the Rules of Court for the nullification of the trial court's judgment by default, the
actions, to be – writ of execution issued by the said court, and the sale of her properties at public auction.
Fundamentally, the service of summons is intended to give official notice to the defendant or respondent that an action had been commenced Basis: The complaint and summons were handed over to her sister who was merely a visitor in her house and, as such, was
against it. The defendant or respondent is thus put [on] guard as to the demands of the plaintiff as stated in the complaint. The service of not a valid substituted service under Rule 14, Section 7 of the Rules of Court.
summons upon the defendant becomes an important element in the operation of a court’s jurisdiction upon a party to a suit, as service of
summons upon the defendant is the means by which the court acquires jurisdiction over his person. Without service of summons, or when St. Joseph Resource averred that even if such substituted service on the petitioner was defective, the defect was cured when the
summons are improperly made, both the trial and the judgment, being in violation of due process, are null and void, unless the defendant latter, through her counsel, appeared in court and moved for an extension of time to file her responsive pleading.
waives the service of summons by voluntarily appearing and answering the suit. [29]
Elsewhere, we declared that jurisdiction of the court over the person of the defendant or respondent cannot be acquired notwithstanding his CA decided in favor of St. Joseph.
knowledge of the pendency of a case against him unless he was validly served with summons.[30] Such is the important role a valid service of ISSUE
summons plays in court actions. Whether or not the appearance of the spouses’ counsel constitutes a voluntary submission to the jurisdiction of the court
The Rules of Court[31] requires that, whenever practicable, summons must be served by handing a copy thereof to the defendant in person.
In case the defendant refuses to receive and sign for it, by tendering the summons to him or her. RULING
However, in the event that summons cannot be served within a reasonable time, the Rules permit that substituted service may be resorted to, Yes.
thus:
Sec. 7. Substituted service. - If, for justifiable causes, the defendant cannot be served within a reasonable time as provided in the preceding The court acquires jurisdiction over the person of the defendant:
section, service may be effected (a) by leaving copies of the summons at the defendant’s residence with some person of suitable age and
discretion then residing therein, or (b) by leaving the copies at defendant’s office or regular place of business with some competent person in 1. by service of the complaint and summons on him, either:
charge thereof. a. by personal service or
In this case, the sheriff employed the substituted service of summons. The defect, however, in the manner in which he implemented this mode b. by substituted service or
of service of summons is readily apparent on the face of the return. It must be emphasized that laws providing for modes other than the c. by extra-territorial service thereof; or
personal service of summons must be strictly followed in order for the court to acquire jurisdiction over the person of respondent or defendant. 2. by his voluntary personal appearance before the court or through counsel.
Compliance therewith should appear affirmatively on the return.[32] The essence of this requirement was enunciated in the case of Keister v.
Navarro[33] to be –
The summons must be served to the defendant in person. It is only when the defendant cannot be served personally within a reasonable time In this case, the petitioner appeared before the court, through counsel, and filed a motion for extension of time to file her answer to the
that a substituted service may be made. Impossibility of prompt service should be shown by stating the efforts made to find the defendant complaint which the trial court granted. By filing the said motion, through counsel, the petitioner thereby submitted herself to the jurisdiction of
personally and the fact that such efforts failed. This statement should be made in the proof of service. This is necessary because substituted the trial court.
service is in derogation of the usual method of service. It has been held that this method of service is “in derogation of the common law; it is a
method extraordinary in character, and hence may be used only as prescribed and in the circumstances authorized by statute.” Thus, under A voluntary appearance is a waiver of the necessity of a formal notice.
the controlling decisions, the statutory requirements of substituted service must be followed strictly, faithfully and fully, and any substituted
service other than that authorized by the statute is considered ineffective.[34] ( mphases supplied.)
An appearance in whatever form, without explicitly objecting to the jurisdiction of the court over the person, is a submission to the jurisdiction of After one failed attempt at personal service of summons, the court process server, resorted to substituted service by serving summons upon
the court over the respondents' househelp who did not acknowledge receipt thereof and refused to divulge their names.
person.
Despite substituted service, the Villas failed to file their Answer, prompting RCRDC to fle a "Motion to Declare Defendants in Default". –
Formal method of entering an appearance Granted!

 deliver to the clerk a written direction ordering him to enter the appearance of the person who subscribes it More than eight months thereafter, the Villas filed a Motion to Lift Order of Default, claiming that on they have just received the documents
 an appearance may be made by simply filing a formal motion, or plea or answer. recently and they denied the existence of two women helpers who allegedly refused to sign and acknowledge receipt of the summons. In any
event, they contended that assuming that the allegation were true, the helpers had no authority to receive the documents.

This formal method of appearance is not necessary. He may appear without such formal appearance and thus submit himself to the jurisdiction RTC set aside the Order of Default and gave them 5 days to file their Answer. Respondents just the same did not file an Answer, drawing
of the court. petitioner to again file a Motion to declare them in default, which the trial court again granted.

He may appear by presenting a motion, for example, and unless by such appearance he specifically objects to the jurisdiction of the court, he The Villas filed an Omnibus Motion for reconsideration of the second order declaring them in default and to vacate proceedings, this
thereby gives his assent to the jurisdiction of the court over his person. When the appearance is by motion objecting to the jurisdiction of the time claiming that the trial court did not acquire jurisdiction over their persons due to invalid service of summons. – denied!
court over his person, it must be for the sole and separate purpose of objecting to the jurisdiction of the court. If his motion is for any other
purpose than to object to the jurisdiction of the court over his person, he thereby submits himself to the jurisdiction of the court. Filed certiorari in CA.
COURT OF APPEALS
CA annulled the trial court's Orders declaring respondents in default for the second time.
JAPRL DEV’T CORP., ET AL. VS. SECURITY BANK CORP.
Basis: The Villas focused all their energies on questioning the RTC’s jurisdiction. The latter motion clearly stated prefatorily their counsel's
CITATION: G.R. No. 190107. June 6, 2011.
reservation or "special
TOPIC: JURISDICTION OVER THE PERSON OF THE DEFENDANT; BY VOLUNTARY APPEARANCE
appearance to question jurisdiction" over the persons of the petitioners. "A party
who makes a special appearance in court challenging the jurisdiction of said court based on the ground of invalid service of summons is not
FACTS
deemed to have
JAPRL – domestic corp engaged in fabrication, manufacture and distribution of steel products
submitted himself to the jurisdiction of the court."
JAPRL applied for a credit facility for P50M with Security Bank Corp. (SBC) – approved!
RCRDC filed a petition for review in the SC.
JAPRL Chairman and President (Limson and Arollado) executed a Continuing Suretyship Agreement (CSA) 2 in favor of SBC wherein they
ISSUE
guaranteed the due and full payment and performance of JAPRL's guaranteed obligations under the credit facility.
Whether or not the RTC acquired jurisdiction over the person of the Villas
Subsequently, JAPRL's financial adviser, MRM Management Incorporated (MRM), convened JAPRL's creditors, SBC included, for the purpose
RULING
of restructuring JAPRL's
Yes.
existing loan obligations. Copies of JAPRL's financial statements (FS) from 1998 to 2001 were given for the creditors to study.
It is settled that if there is no valid service of summons, the court can still acquire
SBC soon discovered material inconsistencies in the FS given by MRM vis-Ã -vis those submitted by JAPRL when it applied for a credit facility,
jurisdiction over the person of the defendant by virtue of the latter's voluntary appearance. Thus Section 20 of Rule 14 of the Rules of Court
drawing SBC to conclude that JAPRL committed misrepresentation.
provides:
Sec. 20. Voluntary appearance. — The defendant's voluntary appearance in the action shall be equivalent to service of
As per agreement, any misrepresentation will constitute an event of default committed by JAPRL and its sureties.
summons. The inclusion in a motion to dismiss of other grounds aside from lack of jurisdiction over the person shall not be
deemed a voluntary appearance.
SBC sent a letter of demand to JAPRL and its chairman and president. Because they failed to comply with the demand, SBC filed a complaint
for sum of money with application for issuance of writ of preliminary attachment before the Regional Trial Court (RTC) of Makati City.
Prescinding from the foregoing, it is thus clear that:
RTC (1) Special appearance operates as an exception to the general rule on voluntary appearance;
The Makati RTC at ordered in open court the archiving of SBC's complaint for sum of money until disposition by the Quezon City RTC of (2) Accordingly, objections to the jurisdiction of the court over the person of the defendant must be explicitly made, i.e., set forth
JAPRL's petition for rehabilitation. in an unequivocal manner; and
(3) Failure to do so constitutes voluntary submission to the jurisdiction of
Consequently, RTC dismissed SBC’s complaint without prejudice. the court, especially in instances where a pleading or motion seeking affirmative relief is filed and submitted to the court for
resolution.
SBC filed a motion for reconsideration stating that the suspension of the proceedings should only be with respect to JAPRL but not with respect
to the chairman and president. – Denied! The Villas did not, in their first motion to lift the order of default, allege that their filing thereof was a special appearance for the purpose only to
question the jurisdiction over their persons. Clearly, they had acquiesced to the jurisdiction of the court.
SBC filed another MR. – denied!

Limson and Arollado opposed, claiming that summons were not served on them, hence, the Makati RTC failed to acquire jurisdiction over their
person. Belen Rodriguez and Jose Santos, Jr., Petitioners, v. Hon. Federico Alikpala (CFI of Manila, Branch XXII), Federico Tolentino and Felisa
Tolentino, Respondents.
COURT OF APPEALS
G.R. No. L-38314, June 25, 1974.
The appellate court held that Limson and Arollado voluntarily submitted themselves to the jurisdiction of the Makati RTC, despite the
qualification that the filing of their respective "Opposition[s] Ad Cautelam" and "Manifestation[s] Ad Cautelam," was "by way of special
appearance" they having sought affirmative relief by praying for the archiving of SBC's complaint.
Topic: Manner of acquiring jurisdiction over the person or party by voluntary submission.
Limson and Arollado filed a motion for recon – denied!

ISSUE
Whether or not the court has acquired jurisdiction over Limson and Arollado

RULING Facts:
Yes.

When a defendant's appearance is made precisely to object to the jurisdiction of On August 19, 1971 the petitioner Rodriguez, assisted by her counsel, the petitioner Santos, filed an action, docketed as Civil Case 204601,
the court over his person, it cannot be considered as appearance in court.
with the city court of Manila against the spouses Manuel and Fe Rebollado for recovery of the sum of P5,320 plus interest, attorney's fees and
Limson and Arollado glossed over the alleged lack of service of summons, however, and proceeded to exhaustively discuss why SBC's costs. A writ of preliminary attachment was issued and served on the Rebollados at their store in Divisoria market. Fe Rebollado immediately
complaint could not prosper against them as sureties. They thereby voluntarily submitted themselves to the jurisdiction of the Makati RTC.
communicated with the petitioner Santos, and later with the latter's client, the petitioner Rodriguez, to plead for time before the attachment was

to be effectively enforced. Rodriguez agreed to cause the suspension of the attachment writ on condition that Fe Rebollado's parents, the now
RAPID CITY REALTY & DEV’T CORP. VS. ORLANDO VILLA and LOURDES PAEZ-VILLA
CITATION: G.R. No. 184197. February 11, 2010. respondents Federico and Felisa Tolentino, would bind themselves, jointly and severally with the Rebollado's, to pay the entire obligation
TOPIC: JURISDICTION OVER THE PERSON OF THE DEFENDANT; BY VOLUNTARY APPEARANCE
subject of the suit. Felisa Tolentino who was then present agreed to this proposal, and so the petitioner Santos, at the request of the petitioner
FACTS
Rodriguez, drew up a motion for judgment on a compromise embodying the terms of the agreement of the parties. On the basis of the said
RCRDC filed a complaint for declaration of nullity of subdivision plans, mandamus and damages against several defendants including Spouses
Orlando and Lourdes Villa. motion, the city court, on August 14, 1971, rendered judgment, as follows:
Parties herein submitted the following compromise agreement and prayed that judgment be rendered in accordance there with: There is no question in the mind of the respondent court that the Rebollado's and the Tolentinos freely and voluntarily entered into the

compromise agreement which became the basis of the judgment of the city court. Be it remembered that neither the Rebollado's nor the

COMPROMISE AGREEMENT Tolentino's question the existence of the indebtedness of the Rebollados or the amount thereof. The respondent court heard the testimonies of

xxx xxx xxx the witnesses first hand and accorded no credence to the version of the Rebollado's and the Tolentino's that Manuel and Fe Rebollado and

Felisa Tolentino were made to sign the motion for a judgment on a compromise without being permitted to read its contents and, further, that

1. That the defendants admit all the material allegations in the plaintiff's complaint and acknowledged their indebtedness to the plaintiff Felisa Tolentino was induced to sign, too, the name of her husband without any authority from the latter. The respondent court analyzed the

in the total amount of P5,980.00, which amount includes expenses of litigation; evidence at length and found that the involvement of the Tolentino's in the compromise agreement arose out of their natural filial concern for

their daughter Fe whose inventories at Divisoria market were under imminent threat of levy and seizure. The respondent court, moreover,

2. That in consideration of defendants acknowledging their said indebtedness and confessing judgment therefor, plaintiff has allowed brooks no doubt, and we concur with it, that both the Rebollado's and the Tolentino's understood the plain unequivocal terms of the

defendants some consideration by allowing them to pay their above-stated account in the following manner, to wit: compromise agreement. And by assuming the roles of co-movants in the motion for a judgment on a compromise, the Tolentino's actively

a) the sum of P200.00 shall be paid upon the signing of this compromise agreement; instigated the city court into giving its judicial imprimatur to the said agreement as well as their participation therein. Under the circumstances,

b) the remaining balance shall be paid in installment basis at the rate of P100.00 a week, payable every Saturday beginning the Tolentino's are estopped from denying the very authority they have invoked.3

August 28, 1971 and every Saturday of the week thereafter until fully paid.

Moreover, because they signed and executed the compromise agreement willingly and voluntarily, and, in a manner of speaking, with their eyes

3. That in order to secure the prompt payment of the said obligations of the defendants, Federico Tolentino and Felisa Tolentino hereby wide open, they should be bound by its terms. A person cannot, to paraphrase Justice Alejo Labrador, repudiate the effects of his voluntary acts

bind themselves to pay jointly and severally with the defendants the said obligations, and in the event of default on the part of the simply because they do not suit him. In the very words of Justice Labrador, "in a regime of law and order, repudiation of an agreement validly

defendants to pay any of the said installments when the same is already due, the judgment which may be rendered by virtue hereof as entered into can not be made without any ground or reason in law or in fact for such repudiation."4

to full amount remaining unpaid, may likewise be executed as against the properties of Federico Tolentino and Felisa Tolentino;

And even if we assume that estoppel does not apply in this case, we nonetheless cannot shunt aside the principle of equity that jurisdiction

4. That failure on the part of the defendants to pay any one of the installments as above-scheduled shall render the remaining balance over a person not formally or originally a party to a litigation may nevertheless be acquired, under proper conditions, thru the voluntary

unpaid immediately due and demandable and the plaintiff shall then be entitled to the execution of the judgment which may be appearance of that person before the court. Thus, judgment may be directed against one who, although not a formal party in the case, has

rendered by virtue hereof; assumed or participated in the defense.5 By coming forward with the original litigants in moving for a judgment on a compromise and,

furthermore, by assuming such interest in the final adjudication of the case as would place them in unequivocal liability, together with the

WHEREFORE, judgment by COMPROMISE is hereby rendered pursuant to the foregoing agreement, enjoining strict compliance Rebollado's, to the plaintiff therein, the Tolentino's effectively submitted themselves to the jurisdiction of the city court. They were and are thus

thereto by the parties. subject to its judgment.

The Rebollado's subsequently failed to comply with the terms of the compromise, thus prompting the petitioner Rodriguez to ask the city court

for a writ of execution not only against the Rebollados but as well against the Tolentino's. When this was granted; and later affirmed over the

opposition of the Tolentino's, the latter brought an action for certiorari in the respondent Court of First Instance of Manila, docketed as Civil SOFIA PASTOR DE MIDGELY, Petitioner, v. THE HONORABLE PIO B. FERANDOS, Judge of the Court of First Instance of Cebu, Branch IX
and LEWELYN BARLITO QUEMADA, Special Administrator of the Testate and Intestate Estate of ALVARO PASTOR Y TATO,
Case 85998, to enjoin the city court from enforcing any writ of execution against them. On December 20, 1973, after hearing duly had, the Respondents.

respondent court rendered judgment excluding the Tolentinos from the effects of the writ of execution granted by the city court in Civil Case G.R. No. L-34314 May 13, 1975
204601. It is this judgment that is the subject of the present appeal.

In excluding the Tolentinos from the effects of the judgment on a compromise rendered by the city court, the respondent court invokes two Topic: Manner of acquiring jurisdiction over the res.
reasons: first, the dispositive portion of the judgment quoted above cannot be executed because it does not explicitly enjoin the Tolentino's to

pay, jointly and severally with the Rebollado's, the amount due to the plaintiff; and second, the city court never acquired jurisdiction over the

persons of the Tolentino's and, therefore, the latter cannot be bound by the judgment rendered in Civil Case 204601.
Facts:

Thus, the petitioners Belen S. Rodriguez and Jose S. Santos, Jr., have come to this Court on appeal by certiorari.
Alvaro Pastor, Sr., a Spanish citizen, was allegedly the owner of properties and rights in mining claims located in Cebu and supposedly held in

trust by his son, Alvaro Pastor, Jr., and his daughter-in-law, Maria Elena Achaval-Pastor. Pastor, Sr. died on June 5, 1966. He was survived by

his wife, Sofia Pastor y Bossio (who died on October 21, 1966) and by his two legitimate children, Mrs. Midgely and Alvaro Pastor, Jr.
Issue:
Respondent Quemada claims to be his illegitimate child.

Whether the city court never acquired jurisdiction over the persons of the Tolentino's and, therefore, the latter cannot be bound by

the judgment rendered in Civil Case 204601. Alvaro Pastor, Sr. in his supposed holographic will dated July 31, 1961 devised to LewelynBarlito Quemada thirty percent of his forty-two

percent share in certain mining claims and real properties. In 1970 the alleged will was presented for probate in Special Proceedings No. 3128-

R assigned to Branch I in Cebu City of the Court of First Instance of Cebu. Quemada was appointed special administrator of the decedent's
Ruling:
estate.

The respondent court is in error.


As such administrator and as heir of Alvaro Pastor, Sr., Quemada filed in the Court of First Instance of Cebu at Toledo City a complaint dated

December 7, 1970 against the spouses Alvaro Pastor, Jr. and Maria Elena Achaval, Mrs. Midgely, Atlas Consolidated Mining and Development
Corporation and Caltex (Philippines), Inc. to settle the question of ownership over certain real properties and the rights in some mining claims, In Civil Case No. 274-T the subject matter of the action for reconveyance consists of properties of Alvaro Pastor, Sr. which are located in Cebu.

to obtain an accounting and payment of the royalties and income thereof and for the payment of damages amounting to P25,000. Quemada's Mrs. Midgely claims an actual interest in those properties. She has been receiving a share of the income therefrom. Therefore, the

theory is that those properties and income belong to the estate of Alvaro Pastor, Sr. extraterritorial service of summons upon her was proper.

Allegedly without complying with the requirements of Rule 14 of the Rules of Court, Quemada caused extraterritorial service of summons to be This Court clarified that in a quasi in rem action jurisdiction over the person of the nonresident defendant is not essential. The service of

made in that case through the Department of Foreign Affairs and the Philippine Embassy in Madrid, Spain, which effected the service of the summons by publication is required "merely to satisfy the constitutional requirement of due process". Consequently, the lower court had

summons by registered mail upon Mrs. Midgely and the Pastor, Jr. spouses at their respective address in Alicante and Barcelona, Spain. jurisdiction to try the case even if it had not acquired jurisdiction over the person. The judgment would be confined to the res. No personal

judgment could be rendered against the non-resident.

Alvaro Pastor, Jr. and Mrs. Midgely, in their respective letters to the Philippine Embassy dated February 11 and 12, 1971, acknowledged the

service of summons but reserved the right to contest the courtsjurisdiction over their persons contending that as nonresidents they could be It should be noted that Civil Case No. 274-T is related to the testamentary proceeding (which is a proceeding in rem par excellance) because

summoned only with leave of court and that the requirements laid down in section 17 of Rule 14 should have been observed. the former case was filed by Quemada for the purpose of recovering the properties which, according to his understanding, belong to the estate

of Alvaro Pastor, Sr. and which are held by Mrs. Midgely and the spouses Alvaro Pastor, Jr. and Maria Elena Achaval.

CFI Ruling

Judge Ferandos denied the motion ruling that Mrs. Midgely and Spouses Pastor, Jr. had been properly summoned.Mrs. Midgely's motion for

reconsideration of the order denying her motion to dismiss was denied by Judge Ferandos in his order of September 27, 1971 wherein he ruled IDONAH SLADE PERKINS, Petitioner,v.ARSENIO P. DIZON, Judge of First Instance of Manila, EUGENE ARTHUR PERKINS, and BENGUET
CONSOLIDATED MINING COMPANY, Respondents.
that the action filed by Quemada was for the recovery of real properties and real rights.
G.R. No. 46631, November 16, 1939.

Topic: Manner of acquiring jurisdiction over the res.


Thus,Mrs. Midgely filed the special civil action of certiorari against Judge Ferandos and Quemada in order to set aside the said Order(denying

her motion to dismiss based on lack of jurisdiction).

Respondent Quemada in his answer alleged that inasmuch as his action against Mrs. Midgely concerns property located here in which she Facts:
claims an interest, it is not necessary that jurisdiction over her person be acquired. The service of summons upon her was not for the purpose

of acquiring jurisdiction over her person but merely as a matter of due process. On July 6, 1938, respondent, Eugene Arthur Perkins, instituted an action in the Court of First Instance of Manila against the Benguet

Consolidated Mining Company for dividends amounting to P71,379.90 on 52,874 shares of stock registered in his name, payment of which was
Issue:
being withheld by the company; and, for the recognition of his right to the control and disposal of said shares, to the exclusion of all

others.Benguet Consolidated Mining Companyfiled its answer alleging, by way of defense, that the withholding of such dividends and the non-
Whether Judge Ferandos gravely abused his discretion in denying Mrs. Midgely's motion to dismiss based on the grounds of lack of
recognition of plaintiff's right to the disposal and control of the shares were due to certain demands made with respect to said shares by the
jurisdiction over her person
Petitioner herein, Idonah Slade Perkins, and by one George H. Engelhard. The answer prays that the adverse claimants be made parties to the

action and served with notice thereof by publication.

Ruling:
On September 5, 1938, the trial court ordered respondent Eugene Arthur Perkins to include in his complaint as parties defendant petitioner,

The Court dismissed the petition for certiorari. Idonah Slade Perkins, and George H. Engelhard. The complaint was accordingly amended and in addition to the relief prayed for in the original

complaint, respondent Perkins prayed that petitioner Idonah Slade Perkins and George Engelhard be adjudged without interest in the shares of

Supposing arguendo that the lower court did not acquire jurisdiction over the person of Mrs. Midgely, still her motion to dismiss was properly stock in question and excluded from any claim they assert thereon. Thereafter, summons by publication were served upon the non-resident

denied because Quemada's action against her may be regarded as a quasi in rem action where jurisdiction over the person of the nonresident defendants, Idonah Slade Perkins.

defendant is not necessary and where service of summons is required only for the purpose of complying with the requirement of due process

(Perkins vs. Dizon, 69 Phil. 186; Banco Español-Filipino vs. Palanca, 37 Phil. 921; Mabanag vs. Gallemore, 81 Phil. 254). On December 10, 1938, Petitioner Idonah Slade Perkins, through counsel, filed her pleading entitled "objection to venue, motion to quash, and

demurrer to jurisdiction" wherein she challenged the jurisdiction of the lower court over her person.

An action quasi in rem is an action between parties where the direct object is to reach and dispose of property owned by them, or of some

interest therein (1 Am Jur 2nd 574; State ex rel. South Brevard Drainage Dist. vs. Smith, 170 So. 440, 126 Fla. 72). Quemada's action falls Petitioner Idonah Slade Perkins’ objection, motion and demurrer having been overruled as well as her motion for reconsideration of the order of

within that category. denial, she now brought the present petition for certiorari, praying that the summons by publication issued against her be declared null and void

for lack of jurisdiction over her person.

The service of summons may, with leave of court, be effected out of the Philippines in three ways: (1) by personal service; (2) by publication in

a newspaper of general circulation in such places and for such time as the court may order, in which case a copy of the summons and order of
Issue:
the court should be sent by registered mail to the last known address of the defendant, and (3) service of summons may be effected in any

other manner which the court may deem sufficient. That third mode of extraterritorial service of summons was substantially complied with in this
Whether or not the Court of First Instance of Manila has acquired jurisdiction over the person of the PetitionerIdonah Slade Perkins
case.
as a non-resident defendant, or, notwithstanding the want of such jurisdiction, whether or not said court may validly try the case.
When, however, the action relates to property located in the Philippines, the Philippine courts may validly try the case, upon the principle that a

"State, through its tribunals, may subject property situated within its limits owned by non-residents to the payment of the demand of its own
Ruling:
citizens against them; and the exercise of this jurisdiction in no respect infringes upon the sovereignty of the State where the owners are

domiciled. Every State owes protection to its citizens; and, when non-residents deal with them, it is a legitimate and just exercise of authority to
The present petition for certiorari of Idonah Slade Perkins, praying that the summons by publication issued against her be declared null and
hold and appropriate any property owned by such non-residents to satisfy the claims of its citizens. It is in virtue of the State's jurisdiction over
void for lack of jurisdiction over her person, is denied.
the property of the non-resident situated within its limits that its tribunals can inquire into the non-resident's obligations to its own citizens, and

the inquiry can then be carried only to the extent necessary to control the disposition of the property. If the non-resident has no property in the
Section 398 of our Code of Civil Procedure provides that when a non-resident defendant is sued in the Philippine courts and it appears, by the
State, there is nothing upon which the tribunals can adjudicate." (Pennoyer v. Neff, supra.)
complaint or by affidavits, that the action relates to real or personal property within the Philippines in which said defendant has or claims a lien

or interest, actual or contingent, or in which the relief demanded consists, wholly or in part, in excluding such person from any interest therein,
In the instant case, there can be no question that the action brought by Eugene Arthur Perkins in his amended complaint against the petitioner,
service of summons maybe made by publication.
Idonah Slade Perkins, seeks to exclude her from any interest in a property located in the Philippines. That property consists in certain shares of

stocks of the Benguet Consolidated Mining Company, a sociedad anonima, organized in the Philippines under the provisions of the Spanish
We have fully explained the meaning of this provision in El Banco Español Filipino vs. Palanca, 37 Phil., 921, wherein we laid down the
Code of Commerce, with its principal office in the City of Manila and which conducts its mining activities therein. The situs of the shares is in the
following rules:
jurisdiction where the corporation is created, whether the certificated evidencing the ownership of those shares are within or without that
(1) In order that the court may validly try a case, it must have jurisdiction over the subject-matter and over the persons of the
jurisdiction. (Fletcher Cyclopedia Corporations, Permanent ed. Vol. 11, p. 95). Under these circumstances, we hold that the action thus brought
parties. Jurisdiction over the subject-matter is acquired by concession of the sovereign authority which organizes a court and
is quasi in rem, for while the judgement that may be rendered therein is not strictly a judgment in rem, "it fixes and settles the title to the
determines the nature and extent of its powers in general and thus fixes its jurisdiction with reference to actions which it may entertain
property in controversy and to that extent partakes of the nature of the judgment in rem." (50 C.J., p 503). As held by the Supreme Court of the
and the relief it may grant. Jurisdiction over the persons of the parties is acquired by their voluntary appearance in court and their
United States in Pennoyer v. Neff (supra);
submission to its authority, or by the coercive power of legal process exerted over their persons.

(2) When the defendant is a non-resident and refuses to appear voluntary, the court cannot acquire jurisdiction over his person
It is true that, in a strict sense, a proceeding in rem is one taken directly against property, and has for its object the disposition of the
even if the summons be served by publication, for he is beyond the reach of judicial process. No tribunal established by one State can
property, without reference to the title of individual claimants; but, in a large and more general sense, the terms are applied to actions
extend its process beyond its territory so as to subject to its decisions either persons or property located in another State x xxxthe
between parties, where the direct object is to reach and dispose of property owned by them, or of some interest therein.
proposition that jurisdiction over the person cannot be thus acquired by publication and notice is no longer open to question; and it is

now fully established that a personal judgment upon constructive or substituted service against a non-resident who does not appear is
The action being in quasi in rem, The Court of First Instance of Manila has jurisdiction over the person of the non-resident. In order to satisfy
wholly invalid. This doctrine applies to all kinds of constructive or substituted process, including service by publication and personal
the constitutional requirement of due process, summons has been served upon her by publication. There is no question as to the adequacy of
service outside of the jurisdiction in which the judgment is rendered; and the only exception seems to be found in the case where the
publication made nor as to the mailing of the order of publication to the petitioner's last known place of residence in the United States. But, of
non-resident defendant has expressly or impliedly consented to the mode of service.
course, the action being quasi in rem and notice having be made by publication, the relief that may be granted by the Philippine court must be
(3) The general rule, therefore, is that a suit against a non-resident cannot be entertained by a Philippine court. Where,
confined to the res, it having no jurisdiction to render a personal judgment against the non-resident. In the amended complaint filed by Eugene
however, the action is in rem or quasi in rem in connection with property located in the Philippines, the court acquires jurisdiction over
Arthur Perkins, no money judgment or other relief in personam is prayed for against the petitioner. The only relief sought therein is that she be
the res, and its jurisdiction over the person of the non-resident is non-essential. In order that the court may exercise power over the res,
declared to be without any interest in the shares in controversy and that she be excluded from any claim thereto.
it is not necessary that the court should take actual custody of the property, potential custody thereof being sufficient. There is potential

custody when, from the nature of the action brought, the power of the court over the property is impliedly recognized by law. "An
Petitioner contends that the proceeding instituted against her is one of interpleading and is therefore an action in personam. Section 120 of our
illustration of what we term potential jurisdiction over the res, is found in the proceeding to register the title of land under our system for
Code of Civil Procedure provides that whenever conflicting claims are or may be made upon a person for or relating to personal property, or the
the registration of land. Here the court, without taking actual physical control over the property, assumes, at the instance of some
performance of an obligation or any portion thereof, so that he may be made subject to several actions by different persons, such person may
person claiming to be owner, to exercise a jurisdiction in rem over the property and to adjudicate the title in favor of the petitioner
bring an action against the conflicting claimants, disclaiming personal interest in the controversy, and the court may order them to interplead
against all the world."
with one another and litigate their several claims among themselves, there upon proceed to determine their several claims. Here, The Benguet
(4) As before stated, in an action in rem or quasi in rem against a non-resident defendant, jurisdiction over his person is non-
Consolidated Mining Company, in its answer to the complaint filed by Eugene Arthur Perkins, averred that in connection with the shares of
essential, and if the law requires in such case that the summons upon the defendant be served by publication, it is merely to satisfy the
stock in question, conflicting claims were being made upon it by said plaintiff, Eugene Arthur Perkins, his wife Idonah Slade Perkins, and one
constitutional requirement of due process.
named George H. Engelhard, and prayed that these last two be made parties to the action and served with summons by publication, so that the

three claimants may litigate their conflicting claims and settle their rights among themselves.
The reason for the rule that Philippine courts cannot acquire jurisdiction over the person of a non-resident, as laid down by the Supreme Court

of the United States in Pennoyer v. Neff, supra, may be found in a recognized principle of public law to the effect that "no State can exercise
Had not the complaint been amended, including the herein petitioner as an additional defendant, and had the court, upon the filing of the
direct jurisdiction and authority over persons or property without its territory. Story, Confl. L., ch. 2; Wheat, Int. L., pt. 2, ch. 2. The several
answer of the Benguet Consolidated Mining Company, issued an order under section 120 of the Code of Civil Procedure, calling the conflicting
States are of equal dignity and authority, and the independence of one implies the exclusion of power from all others. And so it is laid down by
claimants into court and compelling them to interplead with one another, such order could not perhaps have validly been served by publication
jurists, as an elementary principle, that the laws of one State have no operation outside of its territory, except so far as is allowed by comity;
or otherwise, upon the non-resident Idonah Slade Perkins, for then the proceeding would be purely one of interpleading. Such proceeding is a
and that no tribunal established by it can extend its process beyond that territory so as to subject either persons or property to its decisions.
personal action, for it merely seeks to call conflicting claimants into court so that they may interplead and litigate their several claims among
"Any exertion of authority of this sort beyond this limit," says Story, "is a mere nullity, and incapable of binding such persons or property in any
themselves, and no specific relief is prayed for against them x x x x Suffice it to say that here the service of the summons by publication was
other tribunals." Story, Confl. L., sec. 539." (Pennoyer v. Neff, 95 U.S., 714; 24 Law. ed., 565, 568-569.).
ordered by the lower court by virtue of an action quasi in rem against the non-resident defendant.
MELINA P. MACAHILIG, Petitioner, v. The Heirs of GRACE M. MAGALIT, Respondents.
because it is needed for forest purposes----so if we deduct 2.3 has. from the 9.9 has. the remaining area for the Petitioner will only be 7.6
G.R. No. 141423, November 15, 2000. has.----that is why the Bureau of Fisheries has to include Lot 4417 and Lot 5216, in order that the area of 10.0 hectares in said order will be

Topic: Manner of acquiring jurisdiction over the res. satisfied."


Facts:

In the Order of June 18, 1993, the trial court adopted the Commissioner’s Report and ruled in favor of Dr. Magalit (1) finding from the
On February 5, 1965, Pepito Magalit, deceased husband of Dr. Grace M. Magalit (now substituted by her heirs as respondents in view of her
Commissioner’s Report and the sketch submitted that the questioned Lot 4417 with an area of 20,805 square meters is actually in the
recent demise), filed with the then Philippine Fisheries Commission -- now Bureau of Fisheries and Aquatic Resources (BFAR) -- Fishpond
possession of Petitioner Melina Macahilig but which was given to the Respondent’s husband, and (2) letting a writ of execution be issued in
Application No. 24400. The application was for eleven (11) hectares of land situated in the Municipality of Batan, Province of Aklan.4 On April
favor of the Respondent and against Petitioner Melina Macahilig for the delivery of Lot 4417.
13, 1972, Bernardo Macahilig, deceased husband of petitioner, filed with the BFAR Fishpond Application No. 29972 for five of the eleven

hectares which Magalit had previously applied for.5 On February 28, 1972, BFAR rejected Macahilig’s application for his failure to submit all the
Petitioner moved for reconsideration, but her motion was denied in the Order of July 14, 1993, which held that she had no valid reason to
requirements.
possess the disputed lot, considering that her husband’s application therefor had been rejected.

Undaunted, Macahilig protested Magalit’s application on November 8, 1976 contending that, for a period of 20 years, he had been in actual
Unfazed by the unfavorable turn of events, Petitioner filed with the CA, on August 12, 1993, a Petition for Certiorari alleging that the trial court
possession of the five-hectare area included in Magalit’s application.
had acted with grave abuse of discretion in issuing the Orders dated June 18 and July 14, 1993.

On August 22, 1979, the Director of BFAR ordered the Committee on Fishpond Claims and Conflict to hear and determine the rights of
The CA ruled that the trial court did not commit grave abuse of discretion when it issued a Writ of Execution ordering the delivery of Lot 4417 to
Macahilig and Magalit over the disputed area. The Committee concluded that the former was merely the latter’s laborer and caretaker. On June
Dr. Magalit. The records show that the fishpond application of petitioner’s husband was rejected by the BFAR, and that petitioner did not
6, 1980, BFAR Director Felix R. Gonzales rendered an Order that: (1) the Letter Protest, dated November 8, 1976, filed by Bernardo Macahilig
present any other evidence to prove her right of possession over the disputed property. On the other hand, Dr. Magalit’s claim was based on
against Pepito Magalit, is dismissed for lack of merit; (2) Fp. A. No. 29972 of Bernardo Macahilig shall remain rejected and Fp. A. No. 24400
the Decision in the Fishpond Case, which upheld her right----as the surviving spouse of the applicant----to possess the ten hectares of land
filed by Pepito Magalit should be, as it is hereby, given due course to contain 10.0 hectares only; and (3) the Regional Director of Iloilo City is
awarded to him, including Lot 4417 which covered an area of more or less 2.0805 hectares. The disputed lot was included in the area awarded
directed to advise Bernardo Macahilig or other occupants to vacate the premises after the finality of the Order.
to Dr. Magalit because of the report of the commissioner appointed by the trial court to settle the issue. Petitioner had not objected either to the

said appointment or to the Report.


Macahilig elevated this disposition to the Office of the President. However, then Presidential Assistant for Legal Affairs Manuel M. Lazaro, by

"authority of the President," denied the motion for reconsideration of Bernardo Macahilig with finality.
The Orders of June 18 and July 14, 1993 were based on the evidence presented before the trial court. Consequently, they cannot be regarded

as capricious and whimsical exercises of judicial power.


Subsequently, Macahilig challenged this action via a "Petition for Review with Prayer for an Issuance of a Writ of Injunction and/or Restraining

Order." He had filed the Petition originally with the Supreme Court, which then referred it to the Intermediate Appellate Court (IAC) where it was
Hence, this Petition for Review under Rule 45 of the Rules of Court, assailing the Decision1 dated October 15, 1999, and the Resolution dated
docketed as AC-GR SP No. 03448. On March 26, 1985, the appellate court rendered a Decision denying and dismissing the petition for review
December 28, 1999, issued by the Court of Appeals (CA) in CA-GR SP No. 31809.
for lack of factual basis, and declaring that Magalit had occupied, cleared and improved the land; and that Macahilig was his mere caretaker

and laborer, and ordering that the petitioner or anybody acting in his behalf is/are to vacate the subject property in question, and to turn it over

to the heirs of Pepito Magalit, considering that the challenged decision has long become final and executory. Issue:

Later on, Magalit instituted Civil Case No. 3517 in the Regional Trial Court of Kalibo, Aklan, for the issuance of a Writ of Execution. On Did the trial court acquire jurisdiction over Lot 4417.
November 29, 1985, Deputy Provincial Sheriff of Aklan Eriberto Taytayon Jr. implemented the Writ issued by Judge Jaime D. Discaya on

October 30, 1985. The heirs of Pepito Magalit, represented by Dr. Magalit, filed on August 6, 1990, a "Motion for Correction of the
Ruling:
Implementation of the Decision of the Court of Appeals in CA-GR SP No. 03448, promulgated on March 26, 1985, and of the Decision of the

Court, dated October 30, 1985, praying that the trial court properly implement said IAC Decision by ordering Spouses Macahilig to turn over to
Petitioner further contends that the trial court gravely abused its discretion in ordering the turnover of Lot 4417 to Dr. Magalit, because of its
her the possession of Lot 4417, which had an area of 2.0805 hectares, more or less, and contending that the Writ of Execution was not
earlier ruling that it had no jurisdiction over said property
satisfied because the spouses had refused to give up the fishpond in question.
We cannot place much weight on this Order. First, the September 9, 1992 Motion for Reconsideration taken up in said Order has not been

attached to or alleged in the herein Petition. Hence, we cannot fully consider the nature of the claim that was denied by this Order or speculate
On September 17, 1992, Judge Maria Carillo-Zaldivar issued the following Order that (1) finding from the Manifestation of counsel on record for
on why the trial court ruled that it had no jurisdiction over the movant’s claim. We cannot
the movant that the two (2) hectares of land she desires to be executed thru an alias writ is outside the ten (10) hectares awarded to her by the
even guess which Order the unidentified movant wanted to be reconsidered.
Fisheries, and (2) this Court has no jurisdiction over her claim.

More important, it is too late in the day for petitioner to challenge the jurisdiction of the trial court. She clearly submitted to its authority by her
On October 9, 1992, Dr. Magalit filed a "Petition for Contempt Against Melina Macahilig," alleging that on November 29, 1985, Bernardo
unqualified participation in Civil Case No. 3517. We cannot allow her to attack its jurisdiction simply because it rendered a Decision prejudicial
Macahilig had refused to turn over Lot 4417 to her. The trial court appointed a commissioner to determine whether Lot 4417 was included in the
to her position. Participation in all stages of a case before a trial court effectively estops a party from challenging its jurisdiction.26 One cannot
parcels of land awarded to the deceased Magalit. Sheriff Nelson R. dela Cruz, the appointed commissioner, submitted his Commissioner’s
belatedly reject or repudiate its decision after voluntarily submitting to its jurisdiction, just to secure affirmative relief against one’s opponent or
Report dated May 13, 1993, pertinent portions of which read (1) Petitioner Dr. Grace M. Magalit is in the actual possession of Lot-A with an
after failing to obtain such relief.27 If, by deed or conduct, a party has induced another to act in a particular manner, estoppel effectively bars
area of 9.9 has., which is a fully developed fishpond, and without any question from the respondent, (2) however, as per Order of the Bureau of
the former from adopting an inconsistent position, attitude or course of conduct that thereby causes loss or injury to the latter.28
Fisheries and Aquatic Resources, dated June 6, 1980, that portion labelled Parcel-A in the sketch with an area of 2.3 has. has to be excluded
purpose. Procedural rules are not to be disdained as mere technicalities. They may not be ignored to suit the convenience of a party.
Adjective law ensures the effective enforcement of substantive rights through the orderly and speedy administration of justice. Rules
Petitioner insists that the trial court had no jurisdiction over the res of Lot 4417 when it issued its September 17, 1992 Order. are not intended to hamper litigants or complicate litigation. But they help provide for a vital system of justice where suitors may be
heard following judicial procedure and in the correct forum. Public order and our system of justice are well served by a
conscientious observance by the parties of the procedural rules.

Again, we disagree. Jurisdiction over the res is acquired either (a) by the seizure of the property under legal process, whereby it is brought into WHEREFORE, the petitions for review on certiorari are DENIED.
actual custody of the law; or (b) as a result of the institution of legal proceedings, in which the power of the court is recognized and made

effective.29 In the latter condition, the property, though at all times within the potential power of the court, may not be in the actual custody of

said court.

The trial court acquired jurisdiction over the disputed lot by virtue of the institution of the Petition for a Writ of Execution filed by the

respondents’ predecessors in interest. Without taking actual physical control of the property, it had an impliedly recognized potential jurisdiction
LIGHT RAIL TRANSIT AUTHORITY, represented by its Administrator MELQUIADES A. ROBLES, Petitioner,
or potential custody over the res. This was the jurisdiction which it exercised when it issued the Writ of Execution directing the surrender of Lot vs.
AURORA A. SALVAÑA, Respondent.
4417 to Dr. Magalit. G.R. No. 192074, June 10, 2014
Topic: Retroactive application of procedural rules
It is true that under the Civil Code of the Philippines, "(l)aws shall have no retroactive effect, unless the contrary is provided. But there
SAMAHAN NG MGA MANGGAGAWA SA HYATT (SAMASAH-NUWHRAIN), Petitioner, are settled exceptions to this general rule, such as when the statute is CURATIVE or REMEDIAL in nature or when it CREATES NEW
vs. RIGHTS.
HOTEL ENTERPRISES OF THE PHILIPPINES, INC., Respondent. On the other hand, remedial or procedural laws, i.e., those statutes relating to remedies or modes of procedure, which do not create new or
G.R. No. 172303 take away vested rights, but only operate in furtherance of the remedy or confirmation of such rights, ordinarily do not come within the legal
Topic: Nature and purpose of procedural law meaning of a retrospective law, nor within the general rule against the retrospective operation of statutes.
FACTS: On May 12, 2006, then Administrator of the Light Rail Transit Authority, Melquiades Robles revoked Atty. Aurora A. Salvaña’s
designation as Officer-in-Charge (OIC) of the LRTA Administrative Department.
Atty. Salvaña was directed to comply with this office order through a memorandum issued on May 22, 2006 by Atty. Elmo Stephen P. Triste, the
Rules of procedure exist for a noble purpose, and to disregard such rules in the guise of liberal construction would be to defeat such purpose. newly designated OIC of the administrative department. Instead of complying, Salvaña questioned the order with the Office of the President.
Procedural rules are not to be disdained as mere technicalities. They may not be ignored to suit the convenience of a party. Adjective law In the interim, Salvaña applied for sick leave of absence on May 12, 2006 and from May 15 to May 31, 2006. In support of her application, she
ensures the effective enforcement of substantive rights through the orderly and speedy administration of justice. Rules are not intended to submitted a medical certificate issued by Dr. Grace Marie Blanco of the Veterans Memorial Medical Center (VMMC).
hamper litigants or complicate litigation. But they help provide for a vital system of justice where suitors may be heard following judicial LRTA discovered that Dr. Blanco did not issue this medical certificate. Dr. Blanco also denied having seen or treated Salvaña. On June 23,
procedure and in the correct forum. Public order and our system of justice are well served by a conscientious observance by the parties of the 2006, Administrator Robles issued a notice of preliminary investigation.
procedural rules. Because of this Salvaña was found guilty of all the charges against her and imposed on her the penalty of dismissal from service with all the
FACTS: Petitioner is a duly registered union and the certified bargaining representative of the rank-and-file employees of Hyatt Regency accessory penalties."
Manila, a five-star hotel owned and operated by respondent Hotel Enterprises of the Philippines, Inc. On January 31, 2001, Hyatt’s General Civil Service Commission:
Manager, David C. Pacey, issued a Memorandum informing all hotel employees that hotel security have been instructed to conduct a thorough Salvaña appealed with the Civil Service Commission. "In her appeal, she claimed that she was denied due process and that there was
bag inspection and body frisking in every entrance and exit of the hotel. no substantial evidence to support the charges against her."
From February 3, 2001 to March 2, 2001 Angelito Caragdag, a waiter at the hotel’s Cafe Al Fresco restaurant and a director of the union, The Civil Service Commission found that Salvaña was guilty only of simple dishonesty.
committed multiple infraction such as refusing to be frisked by the security personnel, interrupting and intimidating his superior, and leaving CA:
work assignment during official working hours On November 11, 2009, the Court of Appeals dismissed the petition and affirmed the Civil Service Commission’s finding that Salvaña was
Because of the succession of infractions he committed, An investigation board was formed, and the matter was set for hearing on May 19, only guilty of simple dishonesty.
2001. However, despite notice of the scheduled hearing, both Caragdag and the Union President failed to attend. Thereafter, the investigating The appellate court also ruled that Administrator Robles had no standing to file a motion for reconsideration before the Civil Service
board resolved on the said date to dismiss Caragdag. Caragdag appealed but the investigating board affirmed its resolution. Commission because that right only belonged to respondent in an administrative case. LRTA moved for reconsideration of this decision
Caragdag’s dismissal was questioned by petitioner, and the dispute was referred to voluntary arbitration upon agreement of the parties. but was denied.
Arbitrator’s ruling: Party arguments:
The Arbitrator affirmed the decision of the investigating board reasoning that the union officers and members had no right to Petitioner argues that it has the legal personality to appeal the decision of the Civil Service Commission before the Court of Appeals. It
breach company rules and regulations on security and employee discipline on the basis of certain suspicions against management cites Philippine National Bank v. Garcia as basis for its argument that it can be considered a "person adversely affected" under the
and an ongoing CBA negotiation standoff. The Voluntary Arbitrator also found that when Caragdag advised Lacambacal and Alvaro pertinent rules and regulations on the appeal of administrative cases. It also argues that respondent’s falsification of the medical
not to give any statement, he threatened and intimidated his superior while the latter was performing his duties. Moreover, there is certificate accompanying her application for sick leave was not merely simple but serious dishonesty.
no reason why he did not arrange his time-off with the Department Head concerned. Respondent argues the opposite.
Petitioner sought reconsideration of the decision while respondent filed a motion for partial reconsideration. However, the Fact relative to the case:
Voluntary Arbitrator denied both motions on May 26, 2003. Previous definition: PARTY ADVERSELY AFFECTED refers to the respondent against whom a decision in a disciplinary case has been
CA’s ruling: On the Petition for Certiorari of petitioners rendered or to the disciplining authority in an appeal from a decision exonerating(only) the said employee. (emphasis supplied)
The CA dismissed the petition outright for being the wrong remedy. The CA explained: During the pendency of this decision, or on November 18, 2011, the Revised Rules on Administrative Cases in the Civil Service or
Final orders or resolution of voluntary arbitrators is through a Petition for Review which should be filed within fifteen (15) days RACCS was promulgated. The Civil Service Commission modified the definition of a "party adversely affected" for purposes of appeal.
from the receipt of notice of judgment, order or resolution of the voluntary arbitrator. Considering that petitioner intends this petition to It now reads: PARTY ADVERSELY AFFECTED refers to the respondent against whom a decision in an administrative case has been
be a Petition for Certiorari, the Court hereby resolves to dismiss the petition outright for being an improper mode of appeal. rendered or to the disciplining authority in an appeal from a decision reversing or modifying the original decision.
Petitioner duly filed a motion for reconsideration of the dismissal, but the motion was denied by the CA. Note: so the question now stands is if LRTA, in a non-exonerating case, may appeal as a PARTY ADVERSELY AFFECTED as
Petitioner filed before the SC a petition for review on certiorari. newly defined despite that this change in definition was done after the case was brought into court.
Party arguments: ISSUE:
Whether the LRTA, as represented by its Administrator, has the standing to appeal the modification by the Civil Service Commission of its
decision
Petitioner points out, that Rule 43 expressly provides "shall not apply to judgments or final orders issued under the Labor Code RULING:
of the Philippines." Hence, a petition for certiorari under Rule 65 is the proper remedy for questioning the decision of the Voluntary In previous cases, or as the rule that stood during the time that the case was tried was that disciplining authority were only allowed to appeal
Arbitrator. from a decision exonerating an employee. In this case, respondent was not exonerated; she was found guilty, but the finding was modified.
On the other hand, respondent maintains that Section 1 of Rule 43 clearly states that it is the governing rule with regard to The LRTA had standing to appeal the modification by the Civil Service Commission of its decision
appeals from awards, judgments, final orders or resolutions of voluntary arbitrators. The employer has the right "to select honest and trustworthy employees." When the government office disciplines an employee based on
ISSUE: causes and procedures allowed by law, it exercises its discretion. This discretion is inherent in the constitutional principle that "[p]ublic officers
WON the CA erred in dismissing outright the petition for certiorari filed before it on the ground that the same is an improper mode of appeal; and employees must, at all times, be accountable to the people, serve them with utmost responsibility, integrity, loyalty, and efficiency; act with
patriotism and justice, and lead modest lives." This is a principle that can be invoked by the public as well as the government office employing
RULING: the public officer.
The question on the proper recourse to assail a decision of a voluntary arbitrator has already been settled in Luzon Development Bank v. Here, petitioner already decided to dismiss respondent for dishonesty. Dishonesty is a serious offense that challenges the integrity of the public
Association of Luzon Development Bank Employees, where the Court held that the decision or award of the voluntary arbitrator or panel of servant charged. To bar a government office from appealing a decision that lowers the penalty of the disciplined employee prevents it from
arbitrators should likewise be appealable to the Court of Appeals by a petition for certiorari under Rule 43, just like those of the quasi-judicial ensuring its mandate that the civil service employs only those with the utmost sense of responsibility, integrity, loyalty, and efficiency.
agencies, boards and commissions enumerated therein, and consistent with the original purpose to provide a uniform procedure for the Honesty and integrity are important traits required of those in public service. If all decisions by quasi-judicial bodies modifying the penalty of
appellate review of adjudications of all quasi-judicial entities. dismissal were allowed to become final and unappealable, it would, in effect, show tolerance to conduct unbecoming of a public servant. The
quality of civil service would erode, and the citizens would end up suffering for it.
On the application of the amendment to the present case:
Procedural laws have retroactive application. In Zulueta v. Asia Brewery:
Hence, upon receipt on May 26, 2003 of the Voluntary Arbitrator’s Resolution denying petitioner’s motion for reconsideration, petitioner should
As a general rule, laws have no retroactive effect. But there are certain recognized exceptions, such as when they are remedial or procedural in
have filed with the CA, within the fifteen (15)-day reglementary period, a petition for review, not a petition for certiorari.
nature. This Court explained this exception in the following language:
Petitioner insists on a liberal interpretation of the rules but we find no cogent reason in this case to deviate from the general rule. Verily, rules
of procedure exist for a noble purpose, and to disregard such rules in the guise of liberal construction would be to defeat such
It is true that under the Civil Code of the Philippines, "(l)aws shall have no retroactive effect, unless the contrary is provided. But there FACTS: On April 15, 2000, petitioner SLR Builders (then known as Violago Builders, Inc), as seller, and respondent Ma. Cristina F. Bayang
are settled exceptions to this general rule, such as when the statute is CURATIVE or REMEDIAL in nature or when it CREATES NEW (Cristina), as buyer, entered into a "contract to sell" of a sixty (60)-square meter lot in Violago Homes Parkwoods Subdivision, located in
RIGHTS. Barangay Payatas, Quezon City.
On the other hand, remedial or procedural laws, i.e., those statutes relating to remedies or modes of procedure, which do not create new or
take away vested rights, but only operate in furtherance of the remedy or confirmation of such rights, ordinarily do not come within the legal Upon full payment of the monthly amortizations on the purchased lot, Cristina demanded from SLR Builders the execution of the deed of
meaning of a retrospective law, nor within the general rule against the retrospective operation of statutes. absolute sale and the lot's certificate of title but the latter failed to deliver, prompting Cristina to file a complaint for specific performance and
Thus, procedural laws may operate retroactively as to pending proceedings even without express provision to that effect. Accordingly, rules of damages against SLR Builders and its President, Oscar Violago (petitioners) before the Housing and Land Use Regulatory Board (HLURB).
procedure can apply to cases pending at the time of their enactment. In fact, statutes regulating the procedure of the courts will be applied on
actions undetermined at the time of their effectively. Procedural laws are retrospective in that sense and to that extent. (Emphasis supplied) HLURB Ruling:
Remedial rights are those rights granted by remedial or procedural laws. These are rights that only operate to further the rules of procedure or Ruled in favor of respondent.
to confirm vested rights. As such, the retroactive application of remedial rights will not adversely affect the vested rights of any person.
Considering that the right to appeal is a right remedial in nature, we find that Section 4, paragraph (k), Rule I of the RACCS applies in this OP (Office of the President) ruling:
case. Petitioner, therefore, had the right to appeal the decision of the Civil Service Commission that modified its original decision of The OP dismissed the petitioners' appeal for having been filed out of time. The OP's resolution stated:
dismissal. . . .Arbiter's decision was received by the respondents/appellants ( referring to the petitioners) on July 27, 2005. On that date, the
Thus, we now hold that the parties adversely affected by a decision in an administrative case who may appeal shall include the 15-day prescriptive period within which to file an appeal began to run. Instead of preparing an appeal, respondents-appellants
disciplining authority whose decision dismissing the employee was either overturned or modified by the Civil Service Commission. opted to file a Motion for Reconsideration on August 10, 2005. Their filing of the said motion interrupted the period of appeal by that
time, however, fourteen (14) days had already elapsed.
JUDITH YU, Petitioner, On April 17, 2006, respondents-appellants received the Resolution denying their Motion for Reconsideration. Following the above
vs. rules, respondents-appellants have only one (1) day left, or until April 18, 2006, within which to file their notice of appeal.
HON. ROSA SAMSON-TATAD, Presiding Judge, Regional Trial Court, Quezon City, Branch 105, and the PEOPLE OF THE Unfortunately, they were able to do so only on April 27, 2006, or nine (9) days late.
PHILIPPINES, Respondents.
G.R. No. 170979, February 9, 2011 The petitioners moved to reconsider and argued that the "fresh period rule" enunciated in the case of Domingo Neypes, et at. v. Court of
Appeals, et al. should be applied to their case.
Topic: Fresh Period Rule; Applies in Criminal Procedure
The OP, in a resolution dated July 26, 2007, denied the petitioners' motion with finality, stating that the "fresh period rule" applies only to judicial
While Neypes involved the period to appeal in civil cases, the Court’s pronouncement of a "fresh period" to appeal should equally apply appeals and not to administrative appeals, such as in petitioners' case.
to the period for appeal in criminal cases. CA Ruling:
Were we to strictly interpret the "fresh period rule" in Neypes and make it applicable only to the period to appeal in civil cases, we shall Denied the petitioners' petition for review. The CA, likewise, denied the petitioners' motion for reconsideration; hence, the filing of the present
effectively foster and encourage an absurd situation where a litigant in a civil case will have a better right to appeal than an accused in a petition for review on certiorari with this Court.
criminal case – a situation that gives undue favor to civil litigants and unjustly discriminates against the accused-appellants. It suggests a
double standard of treatment when we favor a situation where property interests are at stake, as against a situation where liberty stands to be ISSUE:
prejudiced. We must emphatically reject this double and unequal standard for being contrary to reason.
FACTS: Whether the "fresh period rule" in Neypes applies to administrative appeals, such as an appeal filed from a decision of the HLURB Board of
Based on the complaint of Spouses Sergio and Cristina Casaclang, an information for estafa against the petitioner was filed with the RTC. Commissioners to the Office to the President.
In a May 26, 2005 decision, the RTC convicted the petitioner as charged.
Fourteen (14) days later, or on June 9, 2005, the petitioner filed a motion for new trial with the RTC, alleging that she discovered new and RULING:
material evidence that would exculpate her of the crime for which she was convicted. We DENY the petition. It is settled that the "fresh period rule" in Neypes applies only to judicial appeals and not to administrative appeals.
In an October 17, 2005 order, respondent Judge denied the petitioner’s motion for new trial for lack of merit. The "fresh period rule" in Neypes declares:
On November 16, 2005, the petitioner filed a notice of appeal with the RTC, alleging that pursuant to our ruling in Neypes v. Court of Appeals,
she had a "fresh period" of 15 days from November 3, 2005, the receipt of the denial of her motion for new trial, or up to November 18, 2005, To standardize the appeal periods provided in the Rules and to afford litigants fair opportunity to appeal their cases, the Court deems it practical
within which to file a notice of appeal. to allow a fresh period of 15 days within which to file the notice of appeal in the Regional Trial Court, counted from receipt of the order
On December 8, 2005, the prosecution filed a motion to dismiss the appeal for being filed 10 days late, arguing that Neypes is inapplicable to dismissing a motion for a new trial or motion for reconsideration.
appeals in criminal cases.
The Petition Henceforth, this "fresh period rule" shall also apply to Rule 40 governing appeals from the Municipal Trial Courts to the Regional Trial Courts;
The petitioner argues that the RTC lost jurisdiction to act on the prosecution’s motions when she filed her notice of appeal within the 15-day Rule 42 on petitions for review from the Regional Trial Courts to the Court of Appeals; Rule 43 on appeals from quasi-judicial agencies to the
reglementary period provided by the Rules of Court, applying the "fresh period rule" enunciated in Neypes. Court of Appeals; and Rule 45 governing appeals by certiorari to the Supreme Court. The new rule aims to regiment or make the appeal period
The Case for the Respondents uniform, to be counted from receipt of the order denying the motion for new trial, motion for reconsideration (whether full or partial) or any final
In their comment, the Spouses Casaclang aver that the petitioner cannot seek refuge in Neypes to extend the "fresh period rule" to criminal order or resolution.
cases because Neypes involved a civil case, and the pronouncement of "standardization of the appeal periods in the Rules" referred to the Obviously, these Rules cover judicial proceedings under the 1997 Rules of Civil Procedure.
interpretation of the appeal periods in civil cases, i.e., Rules 40, 41, 42 and 45, of the 1997 Rules of Civil Procedure among others; nowhere in
Neypes was the period to appeal in criminal cases, Section 6 of Rule 122 of the Revised Rules of Criminal Procedure, mentioned.
ISSUE:
The core issue boils down to whether the "fresh period rule" enunciated in Neypes applies to appeals in criminal cases.
RULING: What was the proper action?
In Neypes, the Court modified the rule in civil cases on the counting of the 15-day period within which to appeal. The Court categorically set a
fresh period of 15 days from a denial of a motion for reconsideration within which to appeal, thus: Petitioner's present case is administrative in nature involving an appeal from the decision or order of the DENR regional office to the DENR
The Supreme Court may promulgate procedural rules in all courts. It has the sole prerogative to amend, repeal or even establish new rules for Secretary. Such appeal is indeed governed by Section 1 of Administrative Order No. 87, Series of 1990. As earlier quoted, Section 1 clearly
a more simplified and inexpensive process, and the speedy disposition of cases. In the rules governing appeals to it and to the Court of provides that if the motion for reconsideration is denied, the movant shall perfect his appeal "during the remainder of the period of appeal,
Appeals, particularly Rules 42, 43 and 45, the Court allows extensions of time, based on justifiable and compelling reasons, for parties to file reckoned from receipt of the resolution of denial;" whereas if the decision is reversed, the adverse party has a fresh 15-day period to perfect his
their appeals. These extensions may consist of 15 days or more. appeal.
Henceforth, this "fresh period rule" shall also apply to Rule 40 governing appeals from the Municipal Trial Courts to the Regional Trial
Courts; Rule 42 on petitions for review from the Regional Trial Courts to the Court of Appeals; Rule 43 on appeals from quasi-judicial In this case, the subject appeal, i.e., appeal from a decision of the HLURB Board of Commissioners to the OP, is not judicial but administrative
agencies to the Court of Appeals and Rule 45 governing appeals by certiorari to the Supreme Court . The new rule aims to regiment or in nature; thus, the "fresh period rule" in Neypes does not apply.
make the appeal period uniform, to be counted from receipt of the order denying the motion for new trial, motion for reconsideration (whether
full or partial) or any final order or resolution. As aptly pointed out by the OP, the rules and regulations governing appeals from decisions of the HLURB Board of Commissioners to the OP
While Neypes involved the period to appeal in civil cases, the Court’s pronouncement of a "fresh period" to appeal should equally apply are Section 2, Rule XXI of HLURB Resolution No. 765, series of 2004, in relation to Paragraph 2, Section 1 of Administrative Order No. 18,
to the period for appeal in criminal cases under Section 6 of Rule 122 of the Revised Rules of Criminal Procedure. series of 1987:
Were we to strictly interpret the "fresh period rule" in Neypes and make it applicable only to the period to appeal in civil cases, we shall Section 2, Rule XXI of the HLURB Resolution No. 765, series of 2004, prescribing the rules and regulations governing appeals from decisions
effectively foster and encourage an absurd situation where a litigant in a civil case will have a better right to appeal than an accused in a of the Board of Commissioners to the Office of the President, pertinently reads:
criminal case – a situation that gives undue favor to civil litigants and unjustly discriminates against the accused-appellants. It suggests a
double standard of treatment when we favor a situation where property interests are at stake, as against a situation where liberty stands to be Section 2. Appeal. - Any party may, upon notice to the Board and the other party, appeal a decision rendered by the Board of Commissioners to
prejudiced. We must emphatically reject this double and unequal standard for being contrary to reason. Over time, courts have recognized with the Office of the President within fifteen (15) days from receipt thereof, in accordance with P.D. No. 1344 and A.O. No. 18 Series of 1987.
almost pedantic adherence that what is contrary to reason is not allowed in law – Quod est inconveniens, aut contra rationem non permissum
est in lege. The pendency of the motion for reconsideration shall suspend the running of the period of appeal to the Office of the President.
WHEREFORE, the petition for prohibition is hereby GRANTED.
Corollary thereto, paragraph 2, Section 1 of Administrative Order No. 18, series of 1987, provides that in case the aggrieved party files a
motion for reconsideration from an adverse decision of any agency/office, the said party has the only remaining balance of the
prescriptive period within which to appeal, reckoned from receipt of notice of the decision denying his/her motion for
reconsideration.
San Lorenzo Ruiz Builders and Developers Group, Inc. vs Ma. Cristina F. Bayang
G.R. No. 194702, April 20, 2015 Thus, in applying the above-mentioned rules to the present case, we find that the CA correctly affirmed the OP in dismissing the petitioners'
appeal for having been filed out of time.
Topic: Fresh Period Rule; not apply to administrative appeals
It is settled that the "fresh period rule" in Neypes applies only to judicial appeals and not to administrative appeals. WHEREFORE, we DENY the present petition for review on certiorari and AFFIRM.
On December 15, 2009, Jocson filed a Supplemental Complaint alleging that, during the pendency of the present suit, San Miguel commenced
to plant corn on the subject landholding which violated their Contract.
FORTUNE LIFE INSURANCE COMPANY, INC., Petitioner,
vs. In his Answer, San Miguel maintained that he had religiously complied with all the terms and conditions of their Contract and that Jocson has
COMMISSION ON AUDIT (COA) PROPER; COA REGIONAL OFFICE NO. VI-WESTERN VISAYAS; AUDIT GROUP LGS-B, PROVINCE OF no valid ground to eject him from the disputed landholding.
ANTIQUE; AND PROVINCIAL GOVERNMENT OF ANTIQUE, Respondents.
G.R. No. 213525, January 27, 2015 PARAD Decision
Topic: Fresh Period Rule; not apply to Rule 64 The PARAD (Provincial Agrarian Reform Adjudicator) ruled in favor of petitioner.
We ruled in Pates v. Commission on Elections that the belated filing of the petition for certiorari under Rule 64 on the belief that the fresh period
rule should apply was fatal to the recourse. As such, the petitioner herein should suffer the same fate for having wrongly assumed that the
fresh period rule under Neypes applied. Rules of procedure may be relaxed only to relieve a litigant of an injustice that is not
commensurate with the degree of his thoughtlessness in not complying with the prescribed procedure. Absent this reason for liberality, San Miguel filed a Motion for Reconsideration but it was denied in an Order.
the petition cannot be allowed to prosper. On June 15, 2011, San Miguel filed his Notice of Appeal. It was subsequently denied because it was filed out of time according to PARAD
FACTS: applying the DARAB Rules of Procedure which only gives a petitioner five (5) days to file an appeal.
Respondent Provincial Government of Antique (LGU) and the petitioner executed a memorandum of agreement concerning the life insurance
coverage of qualified barangay secretaries, treasurers and tanod, the former obligating ₱4,393,593.60 for the premium payment, and Note: The PARAD found that San Miguel, through his counsel, received his copy of Decision dated January 26, 2011 on February
subsequently submitting the corresponding disbursement voucher to COA Antique for pre-audit. The latter office disallowed the payment for 3, 2011 and thereafter filed his MR on February 15, 2011, thus, he could have only three (3) days within which to file his Notice of
lack of legal basis. Respondent LGU appealed but its appeal was denied. Appeal upon its denial. The MR was denied on May 31, 2011 and San Miguel, through his counsel, received his copy of the Order
COA Ruling: on June 2, 2011 and he filed his Notice of Appeal on June 15, 2011 or after twelve (12) days, which, following the rules
Consequently, the petitioner filed its petition for money claim in the COA. On November 15, 2012, the COA issued its decision denying the abovementioned, is already beyond the period allowed.
petition, holding that only municipal or city governments are expressly vested with the power to secure group insurance coverage for barangay
workers. Undaunted, San Miguel filed a Petition for Certiorari with the CA praying that the new 2009 DARAB Rules of Procedure which adopted the
The petitioner received a copy of the COA decision on December 14, 2012, and filed its motion for reconsideration on January 14, 2013. "fresh period rule" should apply to the case.
However, the COA denied the motion, the denial being received by the petitioner on July 14, 2014.
CA Ruling: Note: In September 1, 2009 or during the pendency of the case the New 2009 DARAB Rules of Procedure was enacted hence the
The petition for certiorari was dismissed because of prayer.
(a) the late filing of the petition;
(b) the non-submission of the proof of service and verified declaration; and CA Ruling:
(c) the failure to show grave abuse of discretion on the part of the respondents.
The CA issued a decision granting San Miguel's petition and remanding the case to the DARAB-PARAD for further proceedings. The CA held
that the "fresh period rule" enunciated in Neypes should be applied in the instant case.

ISSUE:
ISSUES: WON the new DARAB Rules of Procedure should apply.
In its motion for reconsideration, the petitioner submits that it filed the petition for certiorari within the reglementary period following the fresh WON the “fresh period rule” should apply.
period rule enunciated in Neypes v. Court of Appeals.
RULING: RULING:
The petitioner posits that the fresh period rule applies because its Rule 64 petition is akin to a petition for review brought under Rule 42 of the
Rules of Court; hence, conformably with the fresh period rule, the period to file a Rule 64 petition should also be reckoned from the receipt of Application of the 2003 DARAB Rules of Procedure
the order denying the motion for reconsideration or the motion for new trial.
The petitioner’s position cannot be sustained. San Miguel alleged that due to the effectivity of the 2009 DARAB Rules of Procedure, its provisions should be applied instead of the 2003
There is no parity between the petition for review under Rule 42 and the petition for certiorari under Rule 64. DARAB Rules of Procedure.
The reglementary periods under Rule 42 and Rule 64 are different. In the former, the aggrieved party is allowed 15 days to file the petition for
review from receipt of the assailed decision or final order, or from receipt of the denial of a motion for new trial or reconsideration (fresh period The SC ruled in the negative.
rule). In the latter, the petition is filed within 30 days from notice of the judgment or final order or resolution sought to be reviewed. The filing of a
motion for new trial or reconsideration, if allowed under the procedural rules of the Commission concerned, interrupts the period; hence, should It must be noted that Section 1, Rule XXIV of the 2009 DARAB Rules of Procedure explicitly states that:
the motion be denied, the aggrieved party may file the petition within the remaining period, which shall not be less than five days in any event,
reckoned from the notice of denial. (fresh period rule not applied) Sec. 1. Transitory Provisions. These Rules shall govern all cases filed on or after its effectivity. All cases pending with the Board and the
The petitioner filed its motion for reconsideration on January 14, 2013, which was 31 days after receiving the assailed decision of the COA on Adjudicators, prior to the date of effectivity of these Rules, shall be governed by the DARAB Rules prevailing at the time of their
December 14, 2012. Pursuant to Section 3 of Rule 64, it had only five days from receipt of the denial of its motion for reconsideration to file the filing.
petition. Considering that it received the notice of the denial on July 14, 2014, it had only until July19, 2014 to file the petition. However, it filed
the petition on August 13, 2014, which was 25 days too late. Thus, pursuant to the above-cited rule, the applicable rule in the counting of the period for filing a Notice of Appeal with the Board is governed
We ruled in Pates v. Commission on Elections that the belated filing of the petition for certiorari under Rule 64 on the belief that the fresh period by Section 12, Rule X of the 2003 DARAB Rules of Procedure, which states that:
rule should apply was fatal to the recourse. As such, the petitioner herein should suffer the same fate for having wrongly assumed that the fresh
period rule under Neypes applied. Rules of procedure may be relaxed only to relieve a litigant of an injustice that is not commensurate The filing of the Motion for Reconsideration shall interrupt the period to perfect an appeal. If the motion is denied, the aggrieved party shall
with the degree of his thoughtlessness in not complying with the prescribed procedure. Absent this reason for liberality, the petition have the remaining period within which to perfect his appeal. Said period shall not be less than five (5) days in any event, reckoned from the
cannot be allowed to prosper. receipt of the notice of denial.

Milagrosa Jocson vs Nelson San Miguel Application of the "fresh period rule" enunciated in the Neypes ruling
G.R. No. 206941, March 9, 2016
This Court likewise finds no merit to San Miguel's contention that the "fresh period rule" laid down in Neypes is applicable in the instant case.
Topic: Fresh Period Rule; not apply to administrative rules
In Panolino, this Court held that the "fresh period rule" only covers judicial proceedings under the 1997 Rules of Civil Procedure, to wit:
In the present case, the appeal from a decision of the Provincial Adjudicator to the DARAB as provided for under Section 1, Rule XIV of the
2003 DARAB Rules of Procedure, is not judicial but administrative in nature. As such, the "fresh period rule" in Neypes finds no application The "fresh period rule" in Neypes declares:
therein.
xxxx
As a final note, it is worthy to emphasize that the right to appeal is not a natural right or a part of due process, but is merely a statutory privilege As reflected in the above-quoted portion of the decision in Neypes, the "fresh period rule" shall apply to Rule 40 (appeals from the
that may be exercised only in the manner prescribed by law. The right is unavoidably forfeited by the litigant who does not comply with the Municipal Trial Courts to the Regional Trial Courts); Rule 41 (appeals from the Regional Trial Courts to the [CA] or Supreme Court);
manner thus prescribed. In addition, the liberal application of rules of procedure for perfecting appeals is still the exception, and not the rule; Rule 42 (appeals from the Regional Trial Courts to the [CA]); Rule 43 (appeals from quasi-judicial agencies to the [CA]); and Rule 45
and it is only allowed in exceptional circumstances to better serve the interest of justice. [36] This exceptional situation, however, does not obtain (appeals by certiorari to the Supreme Court). Obviously, these Rules cover judicial proceedings under the 1997 Rules of Civil
in this case. Procedure.

FACTS: On September 10, 2008, Milagrosa C. Jocson (Jocson) filed with the DARAB-PARAD, Region III of San Fernando City, Pampanga, a In the present case, the appeal from a decision of the Provincial Adjudicator to the DARAB as provided for under Section 1, Rule XIV of the
Complaint for ejectment with damages against respondent Nelson San Miguel (San Miguel) and all persons claiming rights under him. 2003 DARAB Rules of Procedure, is not judicial but administrative in nature. As such, the "fresh period rule" in Neypes finds no application
therein.
In the Complaint, Jocson alleged that she is the registered owner of a parcel of agricultural land, located in Magalang, Pampanga. She
asserted that 56,000 sq m thereof became the subject of an Agricultural Leasehold Contract (Contract) between her and San Miguel, with the As correctly observed by PARAD, San Miguel should perfect his appeal during the remainder of the period of appeal, but not less than five (5)
latter as tenant-lessee. As part of the contract, they agreed that the subject landholding shall be devoted to sugar and rice production. days, reckoned from receipt of the resolution of denial of his MR or until June 7, 2011.

According to Jocson, San Miguel, however, occupied the entire landholding and refused to vacate the portion not covered by their Contract As a final note, it is worthy to emphasize that the right to appeal is not a natural right or a part of due process, but is merely a statutory privilege
despite repeated demands. that may be exercised only in the manner prescribed by law. The right is unavoidably forfeited by the litigant who does not comply with the
manner thus prescribed. In addition, the liberal application of rules of procedure for perfecting appeals is still the exception, and not the rule;
and it is only allowed in exceptional circumstances to better serve the interest of justice. This exceptional situation, however, does not obtain in
this case.

WHEREFORE, in consideration of the foregoing disquisitions, the petition is hereby GRANTED.


Party List Ang Pamilya Party List Gabriela

Party List Akbayan

THE UNITED STATES, Plaintiff-Appellee, v. JOSE TAMPARONG ET AL., Defendants-Appellants.


G.R. No. 9527, August 23, 1915 Party List Bayan Muna

Topic: Philippine Courts Party List Anak Pawis

Under the Spanish criminal procedure, appeals from justices’ courts were allowed only to Courts of First Instance. By section 43 of General
Orders No. 58, this procedure has been so amended that appeals can be taken to the Supreme Court in such cases when the validity or
constitutionality of a statute is involved. This amendment of the procedure does not carry with it the right of review of the facts, but is confined
to the purpose stated — that is, of determining the validity or constitutionality of the statute or ordinance upon which the judgment was Those who voted for the passing of the law were classified by petitioners as comprising "Team Patay," while those who voted against it form
predicated. Former cases reviewed, showing that such has uniformly been the interpretation of section 43 by this court. "Team Buhay"
FACTS: On February 27, 2013, COMELEC Law Department issued a letter ordering the immediate removal of the tarpaulin; otherwise, it will be
The defendants were convicted by the justice of the peace of Baguio for having played the game of chance called "monte" in violation of constrained to file an election offense against petitioners.
Ordinance No. 35. They appealed to the Court of First Instance, where they were again tried and convicted upon the same charge. An appeal Petitioners initiated this case through this petition for certiorari and prohibition with application for preliminary injunction and temporary
was allowed to this court because the validity of Ordinance No, 35 was drawn in question during the trial of the cause in the court below. restraining order questioning the constitutionality of the COMELEC’s order.
Two questions are raised by this appeal: (1) Is Ordinance No. 35 valid, and (2) is this court required under the law to examine the evidence for
the purpose of determining the guilt or innocence of the defendants?
ISSUES:
Is the court required under the law to examine the evidence for the purpose of determining the guilt or innocence of the defendants? On March 13, 2013, respondents filed their comment arguing that petitioners violated the doctrine of hierarchy of courts in directly filing their
RULING: petition before this court.
In a long line of cases the Court had constantly ruled that upon appeal the only question to be considered will be that of the validity or invalidity
of an ordinance. The Court cannot review the evidence nor pass upon any other question of law which may appear in the record."
The court has not, since its organization, held in any case that it has the power to review the facts touching the guilt of an accused person in ISSUE:
cases of the character of the one under consideration. WON petitioners violated the doctrine of hierarchy of courts in directly filing their petition before this court.

Some discussion has arisen in regard to the language we should use in the final disposition of cases wherein the statute or ordinance has been
upheld. Sometimes we say, "The judgment is affirmed," and at other times we have said "the appeal is dismissed," etc. The result is the same
RULING:
and it is of little importance which expression we use. But, as the case comes to us on appeal for the purpose of testing the legality of the
Hierarchy of courts
statute or ordinance upon which the judgment rests and as the judgment cannot be executed without the sanction of this court, it is perfectly
This brings us to the issue of whether petitioners violated the doctrine of hierarchy of courts in directly filing their petition before this court.
legal to "affirm" or "reverse" the judgment as the case may be.
Respondents contend that petitioners’ failure to file the proper suit with a lower court of concurrent jurisdiction is sufficient ground for the
For the foregoing reasons the judgment appealed from is affirmed, with costs against the defendants. So ordered.
dismissal of their petition. They add that observation of the hierarchy of courts is compulsory, citing Heirs of Bertuldo Hinog v. Melicor. While
respondents claim that while there are exceptions to the general rule on hierarchy of courts, none of these are present in this case.
The doctrine that requires respect for the hierarchy of courts was created by this court to ensure that every level of the judiciary performs its
designated roles in an effective and efficient manner.
Trial courts do not only determine the facts from the evaluation of the evidence presented before them. They are likewise competent to
determine issues of law which may include the validity of an ordinance, statute, or even an executive issuance in relation to the Constitution.
The Court of Appeals is primarily designed as an appellate court that reviews the determination of facts and law made by the trial courts.
THE DIOCESE OF BACOLOD, et al Petitioners,
This court, on the other hand, leads the judiciary by breaking new ground or further reiterating — in the light of new circumstances or in the light
vs.
of some confusions of bench or bar — existing precedents.
COMMISSION ON ELECTIONS, et al Respondents.
In other words, the Supreme Court’s role to interpret the Constitution and act in order to protect constitutional rights when these become
G.R. No. 205728, January 21, 2015
exigent should not be emasculated by the doctrine in respect of the hierarchy of courts. That has never been the purpose of such doctrine.
Topic: Heirarchy of Courts; Exceptions
Thus, the doctrine of hierarchy of courts is not an iron-clad rule. This court has "full discretionary power to take cognizance and assume
Thus, the doctrine of hierarchy of courts is not an iron-clad rule. This court has "full discretionary power to take cognizance and assume
jurisdiction [over] special civil actions for certiorari . . .filed directly with it for exceptionally compelling reasons or if warranted by the nature
jurisdiction [over] special civil actions for certiorari . . .
of the issues clearly and specifically raised in the petition."
As correctly pointed out by petitioners, we have provided exceptions to this doctrine:
First, a direct resort to this court is allowed when there are genuine issues of constitutionality that must be addressed at the most
FACTS: On February 21, 2013, petitioners posted two (2) tarpaulins within a private compound housing the San Sebastian Cathedral of immediate time.
Bacolod. The first tarpaulin contains the message "IBASURA RH Law" referring to the Reproductive Health Law of 2012. The second tarpaulin Second is when the issues involved are of transcendental importance. In these cases, the imminence and clarity of the threat to
is the subject of the present case. This tarpaulin contains the following: fundamental constitutional rights outweigh the necessity for prudence.
Third, cases of first impression warrant a direct resort to this court. In cases of first impression, no jurisprudence yet exists that
will guide the lower courts on this matter.
TEAM BUHAY TEAM PATAY Fourth, the constitutional issues raised are better decided by this court.
Fifth, the time element presented in this case cannot be ignored .
Sixth, the filed petition reviews the act of a constitutional organ . COMELEC is a constitutional body.
Estrada, JV Angara, Juan Edgardo Seventh, petitioners rightly claim that they had no other plain, speedy, and adequate remedy in the ordinary course of law that
could free them from the injurious effects of respondents’ acts in violation of their right to freedom of expression.
Eighth, the petition includes questions that are " dictated by public welfare and the advancement of public policy, or demanded by
Honasan, Gregorio Casiño, Teddy the broader interest of justice, or the orders complained of were found to be patent nullities, or the appeal was considered as clearly
an inappropriate remedy."
It is not, however, necessary that all of these exceptions must occur at the same time to justify a direct resort to this court. While generally, the
hierarchy of courts is respected, the present case falls under the recognized exceptions and, as such, may be resolved by this court directly.
Magsaysay, Mitos Cayetano, Alan Peter After a long discussion on the subject the Court later admitted that the tarpaulin in question is a form of free speech by stating:
xxx
This is a form of speech hopeful of a quality of democracy that we should all deserve. It is protected as a fundamental and primordial right by
Pimentel, Koko Enrile, Jackie our Constitution. The expression in the medium chosen by petitioners deserves our protection.
WHEREFORE, the instant petition is GRANTED. The temporary restraining order previously issued is hereby made permanent. The act of the
COMELEC in issuing the assailed notice dated February 22, 2013 and letter dated February 27, 2013 is declared unconstitutional.
Trillanes, Antonio Escudero, Francis

Villar, Cynthia Hontiveros, Risa

Party List Buhay Legarda, Loren


EDITHA PADLAN, Petitioner, regarding the projects, wherein the DPWH would give an advance payment even before project completion. Petitioner then gave the blank LBP
vs. check to respondent to guarantee payment of 15,698 bags of Portland cement. However a year later respondent filled up the blank LBP Check
ELENITA DINGLASAN and FELICISIMO DINGLASAN, Respondents. by placing P2,062,000.00. The petitioners then filed a case against the respondent.
G.R. No. 180321, March 20, 2013 RTC
Topic: Jurisdiction; determined on the allegations Ruled in favor of the petitioner
In order to determine which court has jurisdiction over the action, an examination of the complaint is essential. Basic as a hornbook principle is Respondent filed a Notice of Appeal
that jurisdiction over the subject matter of a case is conferred by law and determined by the allegations in the complaint which comprise a Petitioners filed a Motion for Reconsideration, to Dismiss Appeal, and for Issuance of Writ of Execution, stating that respondent’s appeal should
concise statement of the ultimate facts constituting the plaintiff's cause of action. The nature of an action, as well as which court or body be dismissed as the same was not perfected due to non-payment of docket and other lawful fees
has jurisdiction over it, is determined based on the allegations contained in the complaint of the plaintiff, irrespective of whether or not The trial court dismissed respondent's appeal and directed the issuance of a writ of execution to implement the RTC Decision
the plaintiff is entitled to recover upon all or some of the claims asserted therein. CA
FACTS: Elenita Dinglasan (Elenita) was the registered owner of a parcel of land. While on board a jeepney, Elenita’s mother, Lilia Baluyot Rendered a decision in favor of the respondent
(Lilia), had a conversation with one Maura Passion (Maura) regarding the sale of the said property. Believing that Maura was a real estate
agent, Lilia borrowed the owner’s copy of the TCT from Elenita and gave it to Maura. Maura then subdivided the property into several lots
under the name of Elenita and her husband Felicisimo Dinglasan (Felicisimo). Petitioner’s averments:
Through a falsified deed of sale bearing the forged signature of Elenita and her husband Felicisimo, Maura was able to sell the lots to different Petitioners allege that since respondent failed to pay the docket and other legal fees at the time he filed the Notice of Appeal, his appeal was
buyers. On April 26, 1990, Maura sold a parcel to one Lorna Ong (Lorna), who later sold the lot to petitioner Editha Padlan. deemed not perfected in contemplation of the law.
After learning what had happened, respondents demanded petitioner to surrender possession of the bought portion of lot, but the latter refused. Respondent’s:
Respondents were then forced to file a case. Summons was, thereafter, served to petitioner through her mother, Anita Padlan. Respondent maintains that his appeal has been perfected by the mere filing of the notice of appeal. Respondent theorizes that with the
Petitioner claimed that the court did not acquire jurisdiction over her, because the summons was not validly served upon her person, but only perfection of his appeal, the trial court is now divested of jurisdiction to dismiss his appeal and, therefore, only the CA has jurisdiction to
by means of substituted service through her mother. Petitioner maintained that she has long been residing in Japan after she married a determine and rule on the propriety of his appeal.
Japanese national and only comes to the Philippines for a brief vacation once every two years.
On April 5, 2001, the RTC issued an Order denying petitioner’s motion to dismiss and declared her in default. Thereafter, trial ensued.
RTC Ruling:
ISSUE:
On July 1, 2005, the RTC rendered a Decision finding petitioner to be a buyer in good faith and, consequently, dismissed the complaint.
WON the CA acquired jurisdiction over the case.
CA Ruling:
On June 29, 2007, the CA rendered a Decision in favor of the respondent.
RULING:
Aggrieved, petitioner filed a Motion for Reconsideration. Petitioner argued that not only did the complaint lacks merit, the lower court failed to
acquire jurisdiction over the subject matter of the case and the person of the petitioner.
Petitioner posits amongst others that the court lacks jurisdiction of the subject matter, considering that from the complaint, it can be inferred that
the value of the property was only ₱4,000.00, which under the BP 129 the MTC has jurisdiction to resolve. The petition is meritorious.
Note: Respondents filed their Complaint with the RTC; hence, before proceeding any further with any other issues raised by the Concomitant with the filing of a notice of appeal is the payment of the required appeal fees within the 15-day reglementary period set forth in
petitioner, it is essential to ascertain whether the RTC has jurisdiction over the subject matter of this case based on the above-quoted Section 4 of Rule 41.
provisions. Respondent’s claim that his non-payment of docket and other lawful fees should be treated as mistake and excusable negligence, attributable
to the RTC Branch Clerk of Court, is too superficial to warrant consideration. This is clearly negligence of respondent's counsel, which is not
excusable. Negligence to be excusable must be one which ordinary diligence and prudence could not have guarded against.
ISSUE: The CA took cognizance over the case, based on the wrong premise that when the RTC issued the Order giving due course to respondent’s
WON the RTC has jurisdiction over the subject matter. Notice of Appeal and directing the Branch Clerk of Court to transmit the entire records of the case to the CA, it ipso facto lost jurisdiction over
the case. Section 9, Rule 41 of the Rules explains that the court of origin loses jurisdiction over the case only upon the perfection of
RULING: the appeal filed in due time by the appellant and the expiration of the time to appeal of the other parties. Withal, prior to the transmittal
of the original records of the case to the CA, the RTC may issue orders for the protection and preservation of the rights of the prevailing party,
as in this case, the issuance of the writ of execution because the respondent’s appeal was not perfected.
Since respondent’s appeal was not perfected within the 15-day reglementary period, it was as if no appeal was actually taken. Therefore, the
In order to determine which court has jurisdiction over the action, an examination of the complaint is essential. Basic as a hornbook principle is RTC retains jurisdiction to rule on pending incidents lodged before it, such as the petitioner’s Motion for Reconsideration, to Dismiss Appeal,
that jurisdiction over the subject matter of a case is conferred by law and determined by the allegations in the complaint which comprise a and for Issuance of Writ of Execution. Having no jurisdiction over the case, the prudent thing that the CA should have done was to dismiss the
concise statement of the ultimate facts constituting the plaintiff's cause of action. The nature of an action, as well as which court or body respondent’s appeal for failure to pay the appeal fees, and declare that the RTC Decision dated June 28, 2002 has now become final and
has jurisdiction over it, is determined based on the allegations contained in the complaint of the plaintiff, irrespective of whether or not executory.
the plaintiff is entitled to recover upon all or some of the claims asserted therein. To recapitulate, one who seeks to avail of the right to appeal must strictly comply with the requirements of the rules, and failure to do so leads
Respondents’ Complaint narrates: to the loss of the right to appeal. The rules require that from the date of receipt of the assailed RTC order denying one’s motion for
- That they are the duly registered owners of the disputed lot. reconsideration, an appellant may take an appeal to the CA by filing a notice of appeal with the RTC and paying the required docket and other
lawful fees with the RTC Branch Clerk of Court, within the 15-day reglementary period for the perfection of an appeal. Otherwise, the
- Without their knowledge and consent, the land was divided into several lots under their names through the fraudulent appellant's appeal is not perfected, and the CA may dismiss the appeal on the ground of non-payment of docket and other lawful fees. As a
manipulations of Maura. consequence, the assailed RTC decision shall become final and executory and, therefore, the prevailing parties can move for the issuance of a
writ of execution.
While every litigant must be given the amplest opportunity for the proper and just determination of his cause, free from the constraints of
- One of the lots was by Maura to Lorna who later sold the lot to petitioner for ₱4,000.00. technicalities, the failure to perfect an appeal within the reglementary period is not a mere technicality. It raises jurisdictional problem, as it
deprives the appellate court of its jurisdiction over the appeal. After a decision is declared final and executory, vested rights are acquired by the
winning party. Just as a losing party has the right to appeal within the prescribed period, the winning party has the correlative right to enjoy the
finality of the decision on the case.
- That despite demands from the respondents, petitioner refused to surrender possession of the subject property.
WHEREFORE, the petition is GRANTED.

Where the ultimate objective of the plaintiffs is to obtain title to real property, it should be filed in the proper court having jurisdiction over the
assessed value of the property subject thereof. Since the amount alleged in the Complaint by respondents for the disputed lot is only
₱4,000.00, hence under Sec. 33 of BP 129 the MTC and not the RTC has jurisdiction over the action. Therefore, all proceedings in the RTC are
null and void. TITLE: HERRERA-FELIX VS. COURT OF APPEALS
Consequently, the remaining issues raised by petitioner need not be discussed further. CITATION: G.R. No. 143736. August 11, 2004.
WHEREFORE, the petition is GRANTED. The Decision of the Court of Appeals in CA-G.R. CV No. 86983, dated June 29, 2007, and its TOPIC: JURISDICTION OVER THE PERSON OF THE DEFENDANT; BY VOLUNTARY APPEARANCE
Resolution dated October 23, 2007, are REVERSED and SET ASIDE. The Decision of the Regional Trial Court, dated July I, 2005, is declared
NULL and VOID. The complaint in Civil Case No. 438-ML is dismissed without prejudice. FACTS
Original complainant:
St. Joseph Resource Development, Inc. (now respondent)
UGUSTUS GONZALES and spouses NESTOR victor and MA. LOURDES RODRIGUEZ, Petitioners,
vs. Complaint:
QUIRICO PE, Respondent. Sum of money against the Spouses Restituto and Ofelia Felix with a prayer for a writ of preliminary attachment.
G.R. No. 167398, August 9, 2011
Topic: Jurisdiction; Docket Fee Cause of action:
Section 9, Rule 41 of the Rules explains that the court of origin loses jurisdiction over the case only upon the perfection of the appeal filed in During the period from November 16, 1992 to December 14, 1992, the Felix Spouses purchased from the respondent tubs of assorted fish.
due time by the appellant and the expiration of the time to appeal of the other parties. Balance payable P1,132,065.50.
While every litigant must be given the amplest opportunity for the proper and just determination of his cause, free from the constraints of
RTC
technicalities, the failure to perfect an appeal within the reglementary period is not a mere technicality. It raises jurisdictional problem, as it Decision in favor of St. Joseph Resource Development, Inc. RTC also granted the writ of preliminary attachment on a bond of P1,132,065.50
deprives the appellate court of its jurisdiction over the appeal. After a decision is declared final and executory, vested rights are acquired by the
(the amount of balance).
winning party. Just as a losing party has the right to appeal within the prescribed period, the winning party has the correlative right to enjoy the
finality of the decision on the case.
Copies of the writ of preliminary attachment, summons and complaint were served to the spouses at their residence, through the
sister of Ofelia Herrera-Felix, Ma. Luisa Herrera. (Ofelia was out of the country)
FACTS:
Respondent Quirico Pe was engaged in the business of construction materials and had been in the business with petitioner spouses Spouses Felix prayed for an extension of time to file their answer – GRANTED!
Rodriguez. Petitioners were then awarded by DPWH two contracts and availed of DPWH’s pre-payment program for cement requirement
Spouses Felix failed to file their answer. So St. Joseph Resource Development, Inc. filed a motion to declare said spouses in default – The Makati RTC at ordered in open court the archiving of SBC's complaint for sum of money until disposition by the Quezon City RTC of
GRANTED! JAPRL's petition for rehabilitation.

A copy of the resolution was sent to and received by the counsel of the Felix Spouses through registered mail. The copy of the Consequently, RTC dismissed SBC’s complaint without prejudice.
decision addressed to the spouses was returned to the court after two notices for having been "Unclaimed." However, then counsel
for the Felix Spouses received his copy of the decision. SBC filed a motion for reconsideration stating that the suspension of the proceedings should only be with respect to JAPRL but not with respect
to the chairman and president. – Denied!
No appeal was made. The motion for a writ of execution was filed and a copy was served to the spouses by registered mail but they failed to
oppose the motion. SBC filed another MR. – denied!

The court thereafter issued an order granting the motion and directing the issuance of a writ of execution. The counsel for the Felix Spouses Limson and Arollado opposed, claiming that summons were not served on them, hence, the Makati RTC failed to acquire jurisdiction over their
received a copy of the said order. person.
COURT OF APPEALS
Personal properties were levied and sold at a public auction. St. Joseph Resource Development, Inc. won as the highest bidder. The appellate court held that Limson and Arollado voluntarily submitted themselves to the jurisdiction of the Makati RTC, despite the
COURT OF APPEALS qualification that the filing of their respective "Opposition[s] Ad Cautelam" and "Manifestation[s] Ad Cautelam," was "by way of special
The wife, Ofelia Herrera-Felix, filed a petition under Rule 47 of the Rules of Court for the nullification of the trial court's judgment by default, the appearance" they having sought affirmative relief by praying for the archiving of SBC's complaint.
writ of execution issued by the said court, and the sale of her properties at public auction.
Basis: The complaint and summons were handed over to her sister who was merely a visitor in her house and, as such, was Limson and Arollado filed a motion for recon – denied!
not a valid substituted service under Rule 14, Section 7 of the Rules of Court.
ISSUE
St. Joseph Resource averred that even if such substituted service on the petitioner was defective, the defect was cured when the Whether or not the court has acquired jurisdiction over Limson and Arollado
latter, through her counsel, appeared in court and moved for an extension of time to file her responsive pleading.
RULING
CA decided in favor of St. Joseph. Yes.
ISSUE
Whether or not the appearance of the spouses’ counsel constitutes a voluntary submission to the jurisdiction of the court When a defendant's appearance is made precisely to object to the jurisdiction of
the court over his person, it cannot be considered as appearance in court.
RULING
Yes. Limson and Arollado glossed over the alleged lack of service of summons, however, and proceeded to exhaustively discuss why SBC's
complaint could not prosper against them as sureties. They thereby voluntarily submitted themselves to the jurisdiction of the Makati RTC.
The court acquires jurisdiction over the person of the defendant:
3. by service of the complaint and summons on him, either:
a. by personal service or TITLE: RAPID CITY REALTY & DEV’T CORP. VS. ORLANDO VILLA and LOURDES PAEZ-VILLA
b. by substituted service or CITATION: G.R. No. 184197. February 11, 2010.
c. by extra-territorial service thereof; or TOPIC: JURISDICTION OVER THE PERSON OF THE DEFENDANT; BY VOLUNTARY APPEARANCE
4. by his voluntary personal appearance before the court or through counsel.
FACTS
In this case, the petitioner appeared before the court, through counsel, and filed a motion for extension of time to file her answer to the RCRDC filed a complaint for declaration of nullity of subdivision plans, mandamus and damages against several defendants including Spouses
complaint which the trial court granted. By filing the said motion, through counsel, the petitioner thereby submitted herself to the jurisdiction of Orlando and Lourdes Villa.
the trial court.
After one failed attempt at personal service of summons, the court process server, resorted to substituted service by serving summons upon
A voluntary appearance is a waiver of the necessity of a formal notice. respondents' househelp who did not acknowledge receipt thereof and refused to divulge their names.

An appearance in whatever form, without explicitly objecting to the jurisdiction of the court over the person, is a submission to the jurisdiction of Despite substituted service, the Villas failed to file their Answer, prompting RCRDC to fle a "Motion to Declare Defendants in Default". –
the court over the Granted!
person.
More than eight months thereafter, the Villas filed a Motion to Lift Order of Default, claiming that on they have just received the documents
Formal method of entering an appearance recently and they denied the existence of two women helpers who allegedly refused to sign and acknowledge receipt of the summons. In any
 deliver to the clerk a written direction ordering him to enter the appearance of the person who subscribes it event, they contended that assuming that the allegation were true, the helpers had no authority to receive the documents.
 an appearance may be made by simply filing a formal motion, or plea or answer.
RTC set aside the Order of Default and gave them 5 days to file their Answer. Respondents just the same did not file an Answer, drawing
This formal method of appearance is not necessary. He may appear without such formal appearance and thus submit himself to the jurisdiction petitioner to again file a Motion to declare them in default, which the trial court again granted.
of the court.
The Villas filed an Omnibus Motion for reconsideration of the second order declaring them in default and to vacate proceedings, this
He may appear by presenting a motion, for example, and unless by such appearance he specifically objects to the jurisdiction of the court, he time claiming that the trial court did not acquire jurisdiction over their persons due to invalid service of summons. – denied!
thereby gives his assent to the jurisdiction of the court over his person. When the appearance is by motion objecting to the jurisdiction of the
court over his person, it must be for the sole and separate purpose of objecting to the jurisdiction of the court. If his motion is for any other Filed certiorari in CA.
purpose than to object to the jurisdiction of the court over his person, he thereby submits himself to the jurisdiction of the court. COURT OF APPEALS
CA annulled the trial court's Orders declaring respondents in default for the second time.

Basis: The Villas focused all their energies on questioning the RTC’s jurisdiction. The latter motion clearly stated prefatorily their counsel's
TITLE: JAPRL DEV’T CORP., ET AL. VS. SECURITY BANK CORP.
reservation or "special
CITATION: G.R. No. 190107. June 6, 2011.
appearance to question jurisdiction" over the persons of the petitioners. "A party
TOPIC: JURISDICTION OVER THE PERSON OF THE DEFENDANT; BY VOLUNTARY APPEARANCE
who makes a special appearance in court challenging the jurisdiction of said court based on the ground of invalid service of summons is not
deemed to have
FACTS
submitted himself to the jurisdiction of the court."
JAPRL – domestic corp engaged in fabrication, manufacture and distribution of steel products
RCRDC filed a petition for review in the SC.
JAPRL applied for a credit facility for P50M with Security Bank Corp. (SBC) – approved!
ISSUE
JAPRL Chairman and President (Limson and Arollado) executed a Continuing Suretyship Agreement (CSA) 2 in favor of SBC wherein they
Whether or not the RTC acquired jurisdiction over the person of the Villas
guaranteed the due and full payment and performance of JAPRL's guaranteed obligations under the credit facility.
RULING
Subsequently, JAPRL's financial adviser, MRM Management Incorporated (MRM), convened JAPRL's creditors, SBC included, for the purpose
Yes.
of restructuring JAPRL's
existing loan obligations. Copies of JAPRL's financial statements (FS) from 1998 to 2001 were given for the creditors to study.
It is settled that if there is no valid service of summons, the court can still acquire
jurisdiction over the person of the defendant by virtue of the latter's voluntary appearance. Thus Section 20 of Rule 14 of the Rules of Court
SBC soon discovered material inconsistencies in the FS given by MRM vis-Ã -vis those submitted by JAPRL when it applied for a credit facility,
provides:
drawing SBC to conclude that JAPRL committed misrepresentation.
Sec. 20. Voluntary appearance. — The defendant's voluntary appearance in the action shall be equivalent to service of
summons. The inclusion in a motion to dismiss of other grounds aside from lack of jurisdiction over the person shall not be
As per agreement, any misrepresentation will constitute an event of default committed by JAPRL and its sureties.
deemed a voluntary appearance.
SBC sent a letter of demand to JAPRL and its chairman and president. Because they failed to comply with the demand, SBC filed a complaint
Prescinding from the foregoing, it is thus clear that:
for sum of money with application for issuance of writ of preliminary attachment before the Regional Trial Court (RTC) of Makati City.
(1) Special appearance operates as an exception to the general rule on voluntary appearance;
RTC (2) Accordingly, objections to the jurisdiction of the court over the person of the defendant must be explicitly made, i.e., set forth
in an unequivocal manner; and
(3) Failure to do so constitutes voluntary submission to the jurisdiction of Respondent’s Answer
the court, especially in instances where a pleading or motion seeking affirmative relief is filed and submitted to the court for Respondent, by way of special appearance through counsel, filed a Motion to Dismiss on grounds of lack of jurisdiction over the case and over
resolution. the person of the respondent.

The Villas did not, in their first motion to lift the order of default, allege that their filing thereof was a special appearance for the purpose only to Respondent alleged that only the courts of London, United Kingdom or Rome, Italy, have jurisdiction over the complaint for damages pursuant
question the jurisdiction over their persons. Clearly, they had acquiesced to the jurisdiction of the court. to the Warsaw Convention, 5 Article 28 (1) of which provides:
An action for damages must be brought at the option of the plaintiff, either before the court of domicile of the carrier or his
principal place of business, or where he has a place of business through which the contract has been made, or before the court
TITLE: Allan Go v. Cordero of the place of destination.
CITATION: G.R. No. 164703, May 4, 2010
TOPIC: Jurisdiction over the defendant - by voluntary appearance Thus, since a) respondent is domiciled in London; b) respondent's principal place
of business is in London; c) petitioner bought her ticket in Italy; and d) Rome, Italy is petitioner's place of destination, then it follows that the
FACTS complaint should only be filed in the proper courts of London, United Kingdom or Rome, Italy.
Mortimer F. Cordero, Vice-President of Pamana Marketing Corporation (Pamana), ventured into the business of marketing inter-island
passenger vessels. Likewise, it was alleged that the case must be dismissed for lack of jurisdiction over the person of the respondent because the
summons was erroneously served on Euro-Philippine Airline Services, Inc. which is not its resident agent in the Philippines. –
After contacting various overseas fast ferry manufacturers from all over the world, he came to meet Tony Robinson, an Australian national Granted! Basis: Our Courts have to apply the principles of international law, and are bound by treaty stipulations entered into by the Philippines
based in Brisbane, Australia, who is the Managing Director of Aluminium Fast Ferries Australia (AFFA). which form part of the law of the land. One of this is the Warsaw Convention.
Petitioner filed an MR – denied! ; filed a certiorari in SC.
Robinson signed documents appointing Cordero as the exclusive distributor of AFFA catamaran and other fast ferry vessels in the Philippines.
As such exclusive distributor, Cordero offered for sale to prospective buyers the 25-meter Aluminium Passenger catamaran known as the ISSUE
SEACAT 25. Whether or not the court has jurisdiction over the person of the defendant/respondent (British Airways)
After negotiations with the lawyers of Allan Go (owner/operator of ACG Express Liner of Cebu City), Cordero was able to close a deal for the RULING
purchase of two (2) SEACAT 25. No.
However, Cordero later discovered that Go was dealing directly with Robinson when he was informed by Dennis Padua of Wartsila Philippines Respondent, in seeking remedies from the trial court through special appearance of counsel, is not deemed to have voluntarily submitted itself
that Go was canvassing for a second catamaran engine from their company which provided the ship engine for the first SEACAT 25. to the jurisdiction of the trial court.
Cordero filed a case seeking to hold Robinson, Go, Tecson and Landicho liable jointly and solidarily for conniving and conspiring together in The second sentence of Sec. 20, Rule 14 of the Revised Rules of Civil Procedure clearly provides:
violating his exclusive distributorship in bad faith and wanton disregard of his rights, thus depriving him of his due commissions. Sec. 20. Voluntary appearance. — The defendant's voluntary appearance in the action shall be equivalent to service of
summons. The inclusion in a motion to dismiss of other grounds aside from lack of jurisdiction over the person of the defendant
Robinson filed a motion to dismiss grounded on lack of jurisdiction over his person, among others. – denied! shall not be deemed a voluntary appearance.
RTC decided in favor of Cordero. In this case, the special appearance of the counsel of respondent in filing the Motion to Dismiss and other pleadings before the trial court
cannot be deemed to be voluntary submission to the jurisdiction of the said trial court.
Defendants filed a petition for certiorari in the CA.
TITLE: Phil. Women’s Christian Temperance Union v. Yangco
ISSUE
Whether or not the court has jurisdiction over the person of Robinson
CITATION: G.R. No. 199595, April 2, 2014.
TOPIC: Payment of filing/docket fees- jurisdictional
RULING
Yes.
FACTS
Teodoro R. Yangco (2nd and 3rd Generation Heirs) Foundation, Inc. (TRY Foundation) filed before the RTC of Quezon City, acting as a Land
Courts acquire jurisdiction over the plaintiffs upon the filing of the complaint, while jurisdiction over the defendants in a civil case is acquired
Registration Court, a Petition for the Issuance of New Title in Lieu of Transfer Certificate of Title (TCT) No. 20970 T-22702 of the Office of the
either through the service of summons upon them in the manner required by law or through their voluntary appearance in court and their
Register of Deeds of Quezon City.
submission to its authority.
The property was previously donated to PWCTUI with a condition that should the property be used for any other purpose/s not specified, the
A party who makes a special appearance in court challenging the jurisdiction of said court based on the ground of invalid service of summons is
gift shall become ipso facto null and void and property given shall automatically revert to the donor, his heirs and assigns.
not deemed to have submitted himself to the jurisdiction of the Court.
PWCTUI’s corporate term expired in September 1979. TRY Foundation claimed that the expiration of PWCTUI’s corporate term in 1979
In this case, however, although the Motion to Dismiss filed by Robinson specifically stated as one of the grounds the lack of "personal
effectively rescinded the donation.
jurisdiction," it must be noted that he had earlier filed a Motion for Time to file an appropriate responsive pleading even beyond the time
provided in the summons by publication. (guys sorry wa jud ni sya sa facts ang summons by publication) Such motion did not state that it was a
PWCTUI claimed, among others, that the RTC has no jurisdiction over the petition because PWCTUI is unaware of its publication. – denied!
conditional appearance entered to question the regularity of the service of summons, but an appearance submitting to the jurisdiction of the
court by acknowledging the summons by publication issued by the court and praying for additional time to file a responsive pleading.
CA affirmed the RTC’s decision.
Consequently, Robinson having acknowledged the summons by publication and also having invoked the jurisdiction of the trial court to secure
PWCTUI sought recourse with the Court thru a petition for review on certiorari.
affirmative relief in his motion for additional time, he effectively submitted voluntarily to the trial court's jurisdiction. He is now estopped from
asserting otherwise, even before this Court.
ISSUE
TTITLE: Lhuillier v. British Airways Whether or not the RTC has acquired jurisdiction over the case

RULING
CITATION: G.R. No. 171092. March 15, 2010. No.
TOPIC: Jurisdiction over the defendant - by voluntary appearance
RTC judgment in the land registration case and all proceedings taken in relation thereto were void because the RTC did not acquire jurisdiction
FACTS over the fundamental subject matter of TRY Foundation’s petition for the issuance of a title which was in reality, a complaint for revocation of
Petitioner Edna Diago Lhuillier took respondent's flight 548 from London to Rome. Once on board, she allegedly requested Julian Halliday donation. No judgment proclaiming TRY Foundation as the absolute owner of the property can be arrived at without declaring the deed of
(Halliday), one of the respondent's flight attendants, to assist her in placing her hand-carried luggage in the overhead bin. However, Halliday donation revoked.
allegedly refused to help and assist her, and even sarcastically remarked that "If I were to help all 300 passengers in this flight, I would have a
broken back!" The issues embroiled in revocation of donation are litigable in an ordinary civil proceeding which demands stricter jurisdictional requirements
than that imposed in a land registration case.
Petitioner further alleged that when the plane was about to land in Rome another flight attendant, Nickolas Kerrigan (Kerrigan), singled her out
from among all the passengers in the business class section to lecture on plane safety. Allegedly, Kerrigan made her appear to the other The payment of docket fees is another jurisdictional requirement for an action for revocation which was absent in the suit filed by
passengers to be ignorant, uneducated, stupid, and in need of lecturing on the safety rules and regulations of the plane. Affronted, petitioner TRY Foundation. On the other hand, Section 111 of P.D. No. 1529 merely requires the payment of filing fees and not docket fees.
assured Kerrigan that she knew the plane's safety regulations being a frequent traveler. Thereupon, Kerrigan allegedly thrust his face a mere
few centimeters away from that of the petitioner and menacingly told her that "We don't like your attitude." Filing fees are intended to take care of court expenses in the handling of cases in terms of cost of supplies, use of equipment, salaries and
fringe benefits of personnel, etc., computed as to man hours used in handling of each case. Docket fees, on the other hand, vest the trial court
Upon arrival in Rome, petitioner complained to respondent's ground manager and jurisdiction over the subject matter or nature of action.
demanded an apology. However, the latter declared that the flight stewards were "only doing their job."
The absence of the jurisdictional requirements for ordinary civil actions thus prevented the RTC, acting as a land registration court, from
Complaint acquiring the power to hear and decide the underlying issue of revocation of donation in the land registration case.
Petitioner filed a Complaint for damages against respondent British Airways before the RTC of Makati City.

Summons, together with a copy of the complaint, was served on the respondent through Violeta Echevarria, General Manager of Euro-
Philippine Airline Services, Inc.

You might also like